Income Tax Planning CFP

¡Supera tus tareas y exámenes ahora con Quizwiz!

Which of the following would be true for items subject to the 2%-of-AGI floor? A: Required safety shoes purchased by a factory employee. B: Union dues paid by a union employee. C: Miscellaneous itemized deductions subject to the 2% floor are no longer deductible. D: Tax return preparation fee paid by a non-employed retiree.

Solution: The correct answer is C. TCJA eliminated the miscellaneous itemized deduction subject to the 2% of AGI so there is no longer a 2% floor calculation. The other miscellaneous itemized deductions remain, not subject to the 2% floor, ie. gambling losses, IRD, deduction for remaining basis in an annuity at death, expenses for employment incurred for a handicapped individual.

In the case of a below-market loan between family members, if the imputed interest rules apply: I: The borrower must recognize interest income. II: The lender has interest income. III: The lender is deemed to have made a gift. IV: The borrower has interest expense. A: Only I is true. B: II, III, and IV are true but I is false. C: I and II are false but III and IV are true. D: All of the above are true.

B

Mary purchased a new five-year class asset on March 7, of the current year. The asset was listed property (not an automobile). It was used 60% for business and the rest of the time for personal use. The asset cost $12,000. Mary made the § 179 election. The income from the business before the § 179 deduction was $200,000. Determine the total deductions with respect to the asset for 2022. A: $7,200 B: $12,000 C: $4,800 D: $6,000

Solution: The correct answer is A. Business Use (60%) Personal Use (40%) Cost $7,200 $4,800 § 179 expense (7,200) Basis for cost recovery $-0-

Tao's moving expense that is reimbursed by MedEx, Inc. will be treated as taxable income to Tao. A: True B: False

Solution: The correct answer is A. Moving expenses are not deductible for AGI and, if reimbursed, will be taxable income after 12/31/17

In the case of a gift loan of greater than $10,000 but less than $100,000, the imputed interest rules apply if the donee has net investment income of over $1,000. A: True B: False

Solution: The correct answer is A. The imputed interest rules apply to gift loans. However, if the amount of the loan is for $100,000 or less, the imputed interest cannot exceed the borrower's net investment income for the tax year. If net investment is $1,000 or less, it is considered to be $0.

Abigail is 63 years old, single, and in excellent health. What is her standard deduction for 2023? A- $13,850 B- $15,350 C- $15,700 D- $27,700

Solution: The correct answer is A. The standard deduction for a single individual under the age of 65 in 2023 is $13,850. The additional standard deduction does not apply until 65.

Billy, single (he was divorced in 2010) and age 42, has the following items of income and expense for the current tax year. Wages: $60,000 Interest: $1,200 Inheritance: $50,000 Alimony paid: $10,000 Child support paid: $8,000 Federal taxes paid: $5,000 State income taxes paid: $2,000 Medical expenses: $7,500 Tickets from his employer for one basketball game: $100 What is the medical expense deduction that will actually be utilized for 2023? A- $0 B- $2,380 C- $3,660 D- $7,500

Solution: The correct answer is A. Wages $60,000 Interest $1,200 less Alimony <$10,000> AGI $51,200 Medical expenses = $7,500 - ($51,200 × 7.5%) = $3,660 Itemized deductions would be: Medical expenses + SALT ($3,660 + $2,000) is less than $13,850 therefore, you will use the standard deduction as it is higher than the itemized deduction. A taxpayer cannot use the standard deduction plus itemized deductions, it is one or the other. Alimony from divorces settled after 12/31/2018 will follow the new rules. Divorces settled on or prior to that date will be grandfathered into the old rules. Taxpayer Certainty Disaster Tax Relief Act of 2020 permanently set the medical expense deduction to 7.5% of AGI. SALT stands for State and Local Taxes.

Which of the following assets, purchased during the current tax year, will be depreciable over a 7-year useful life under the Modified Accelerated Cost Recovery System (MACRS)? A- Business automobiles. B- Office furniture. C- Land. D- Cattle.

Solution: The correct answer is B. A is incorrect. Automobiles are considered 5-year property under MACRS. C is incorrect. Land is not depreciable under MACRS. D is incorrect. Cattle is generally depreciable over 5-years under MACRS.

Gee and Bea Lee have a very diverse family. Which of the following children does not meet the relationship test for the purpose of Gee and Bea qualifying them as dependent for that child? A- Dan, Bea's 8-year-old son from a prior marriage. B- Fran, Gee's 10-year-old cousin. C- Stan, a 5-year-old foster child placed with Gee and Bea by a state agency. D- Xan, Bea's 12-year-old niece.

Solution: The correct answer is B. Because Fran is from the same generation as Gee (even though she is presumably much younger than Gee), she does not meet the relationship test. A qualifying child must be from a lower generation than the taxpayer. All of the other options do meet the relationship test for being a qualifying child for the purpose of the dependency exemption.

Dina loans $24,000 to her daughter Erin and does not charge any interest. Erin has investment income of $1,400 and investment expenses of $300. The applicable federal rate is 5%. How much interest must be imputed on the loan? A- $1,000. B- $1,100. C- $1,200. D- $1,400.

Solution: The correct answer is B. Erin has net investment income of $1,100. Therefore, the amount of imputed interest is the lesser of net investment income or interest calculated using the AFR less interest calculated using the stated rate of the loan. Since the stated rate of interest on the loan is 0%, the amount of imputed interest is the lesser of $1,100 or $1,200 ($24,000 × 0.05). Therefore, $1,100 of interest must be imputed on the loan.

Which of the following qualify for the medical expense deduction? A- Maternity clothes for a celebrity. B- The cost of non-prescribed insulin. C- A weight reduction program designed to avoid diabetes. D- A breast implant for an aging starlet.

Solution: The correct answer is B. Non-prescribed medicine is generally not deductible, but an exception to this is insulin. Prescribed or non-prescribed insulin is considered a deductible medical expense. All others including clothing, weight loss programs or elective surgery are not deductible medical expenses but A and D could be business expense deductions.

Proposed Regulations carry more weight than Temporary Regulations. A: True B: False

Solution: The correct answer is B. Proposed Regulations have no legal precedence and are not binding on taxpayers until the regulation becomes final.

Stuart is the sole owner and a material participant in a business in which he has $50,000 at risk. If the business incurs a loss of $80,000 from operations, Stuart will be allowed the full amount of the loss as a deduction. A: True B: False

Solution: The correct answer is B. The at-risk provisions limit the deductibility of losses from business and income-producing activities for individuals and closely held corporations. The $80,000 loss reduces Stuart's at-risk amount to $0 and produces a $50,000 loss that is currently deductible. The $30,000 balance of the loss is suspended under the at-risk rules. The passive loss rules do not apply because Stuart is a material participant.

Martha gives her niece a machine to use in her business with a fair market value of $4,200 and a basis of $4,400. What is the niece's basis for depreciation (cost recovery)? A- $0; Gift property is not depreciable. B- $4,200 C- $4,300 D- $4,400

Solution: The correct answer is B. Your basis for depreciation is the lower of FMV or adjusted basis for depreciation

Mark, Inc placed into service a machine in 2022 that cost $2,715,000. What is the maximum Section 179 expense that Mark, Inc could take for 2022? A: $615,000 B: $915,000 C: $1,065,000 D: $1,080,000

Solution: The correct answer is C. As of 2022, $1,080,000 - ($2,715,000 - $2,700,000) = $1,065,000

Paul and Heather are getting divorced. As part of the divorce settlement, Heather receives the vacation home worth $3,000,000. The couple originally purchased the vacation home five years ago for $1,000,000. Which of the following statements is true? A- If Heather sells the vacation home for $2,500,000, she will have a $500,000 loss. B- In any future sale of the vacation home, Heather and Paul will each have a basis of $500,000. C- If Heather sells the vacation home for $3,200,000, she will have a gain of $2,200,000. D- If Heather sells the vacation home only six months after receiving it in the divorce settlement, any gain or loss that she has will be short-term.

Solution: The correct answer is C. Option "D" is incorrect because Heather has a carryover holding period, so if she sells the vacation home six months after receiving it in the divorce settlement, any gain or loss that she has will be long-term. Option "A" is incorrect because Heather has a carryover basis in the property. Therefore, her basis is $1,000,000. Option "B" is incorrect because Paul no longer has an interest in the property.

Our tax laws encourage taxpayers to: A- Sell investment assets that have appreciated, but keep investment assets that have declined in value. B- Sell investment assets that have appreciated, as well as those investment assets that have declined in value. C- Sell investment assets that have declined in value, but keep those investment assets that have appreciated. D- Keep those investment assets that have declined in value as well as those investment assets that have appreciated.

Solution: The correct answer is C. Selling assets which have declined in value may result in a loss. Selling appreciated assets will most always result in a taxable gain. The desire of the taxpayer is to pay the least amount of tax. Selling declined assets will assure that.

Under current law, what is the maximum Child Tax Credit for 2023 per child for a couple with an AGI of $300,000? A- $500 B- $1,000 C- $2,000 D- $1,600

Solution: The correct answer is C. The Child Tax Credit is $2,000. TCJA increased the credit from 1,000 for tax years 2018 - 2025. This credit is not fully refundable. It is refundable up to $1,600 per child.

Sally and Kaleb have 2 dependent children, ages 6 and 9. They file their income tax return jointly. They are both in great health, and in their mid-30s. Assuming their AGI is $192,000, how much should they expect as a credit for the child tax credit in 2023, if anything? A- $2,800 B- $12,700 C- $4,000 D- $20,000

Solution: The correct answer is C. The child tax credit is $2,000 per qualified child, so $4,000 in this instance. Up to $3,200 can be refundable ($1,600 per child 2023)

Bonnie and Manuel are married. He paid $100,000 for their home five years ago. Its fair market value was $150,000 when Manuel died. What is Bonnie's basis in the home after Manuel's death if the home was held as community property and Manuel left his half to Bonnie? A- $50,000 B- $75,000 C- $125,000 D- $150,000

Solution: The correct answer is D. Upon the death of either spouse in a community property state, both halves of community property are stepped to the fair market value regardless of who inherits the decendant's half.

If an income tax return is not filed by a taxpayer, there is no statute of limitations on assessments of tax by the IRS. A: True B: False

A

Which of the following statements regarding cafeteria plans is not correct? A- A cafeteria plan must offer at least three nontaxable benefits. B- A cafeteria plan is a written plan under which the employee may choose to receive either cash or taxable benefits as compensation or qualified fringe benefits that are excludable from wages. C- Cafeteria plans are authorized by Section 125 of the Internal Revenue Code. D- A cafeteria plan is appropriate when employee benefit needs vary within the employee group.

Solution: The correct answer is A. A cafeteria plan must offer at least one taxable benefit, usually cash, and one qualified nontaxable benefit. All of the other statements regarding cafeteria plans are correct.

Which of the following is a "trade or business" expense? A- QNEC contribution to 401K plans to retain qualification. B- Charitable contributions by a partnership. C- Gifts to contract; letting officials get preferential treatment in contracts. D- Prepaid parking fines paid at a discount to the city to avoid ticketing in high traffic areas.

Solution: The correct answer is A. Charitable contributions by a partnership are personal expenses. Fines are not deductible. Gifts (bribes) are not deductible. QNEC, qualified non-elective contributions, to a 401K plan to maintain qualification is an ordinary/necessary business expense.

Romeo is starting a new business. His major concern is to limit his liability, but he is also concerned with the ability of the company to raise capital. At some point, he would like to be able to easily sell partial interests in the business to other investors or perhaps to even take the company public, so he does not want to limit the number of potential owners. Which of the following entities would best suit Romeo's needs? A- A C corporation. B- A S corporation. C- A LLC. D- A General Partnership.

Solution: The correct answer is A. Option "B" is incorrect because Romeo does not want to limit the number of potential owners, and a S corporation cannot have more than 100 owners. Option "C" is incorrect; if Romeo is interested in taking the company public in the future, the LLC form would not be appropriate. Option "D" is incorrect because a partnership would not provide the limited liability that Romeo is looking for.

Chris works for a company where he travels constantly. The company does not reimburse him for all of his expenses so he has $14,000 of unreimbursed employee business expenses. If Chris' adjusted gross income is $75,000, how much of these expenses may he deduct? A- $0 B- $7,000 C- $12,500 D- $14,000

Solution: The correct answer is A. The $14,000 of unreimbursed employee business expenses are miscellaneous itemized deductions subject to 2% of AGI floor and this deduction was eliminated for years after 12/31/17.

Due to a merger, Elizabeth transfers from New York City to Albuquerque. Under a new job description, she is reclassified from employee to independent contractor status. Her moving expenses, which are not reimbursed, are as follows: Transportation $1,200 Meals 300 Lodging 400 Cost of moving household goods 3,000 Penalty for breaking New York City apartment lease 5,000 Elizabeth's deductible moving expense is: A: $0 B: $4,600 C: $4,750 D: $4,900

Solution: The correct answer is A. Under TCJA, all moves, even qualified moves, are not deductible or excludible from income for 2018 - 2025. Moving is not considered a business expense. Self-employed individuals could previously deduct moving expenses above-the-line under the same rules as employees, but the deduction is not a Schedule C deduction, it is a Form 1040 ATL deduction. The suspension of that deduction applies to both employees and self-employed. The only exception to the suspension of the deduction is for military.

Which of the following statements regarding administrative tax law issued by the IRS are true? I- Revenue rulings are based on a set of facts that are common to many taxpayers. II- Private letter rulings are issued at the request of an individual taxpayer. III- Determination letters are issued prior to the completion of a transaction. A- I only. B- I and II only. C- II and III only. D- I, II and III.

Solution: The correct answer is B. Determination letters are issued at the request of a taxpayer by the district director of the IRS when the taxpayer has already engaged in a transaction and would like to know how to report the transaction for tax purposes.

What is the major advantage of the cash method of accounting? I- Income is counted at the time it is earned. II- Income may be deferred. III- Deductible expenses may be accelerated. IV- Deductible expenses may be used as carry-backs. A- I and II only. B- II and III only. C- I and IV only. D- III and IV only. E- None of the above.

Solution: The correct answer is B. Income may be deferred until cash is received and deductible expenses accelerated if paid.

Which of the following taxes generates the largest percentage of gross collections for the Internal Revenue Service? A; Corporate income tax. B: Individual income tax. C: Estate tax. D: Employment tax.

Solution: The correct answer is B. Individual income taxes make up nearly 50% of the gross collections by the Internal Revenue Service.

Jay owns an office building that burned down. The basis of the building was $125,000. The insurance policy paid Jay $195,000. What can Jay do that will defer the recognition of any gain? A- Jay must use the money to buy similar property. B- Jay must replace the property with similar property of equal or greater value within two years from December 31st of the year of realization. C- Jay must replace the property with any kind of business property within two years from the date of the fire. D- Jay must replace the property with similar property that costs at least as much as what the policy paid within 3 years of the fire.

Solution: The correct answer is B. Insurance proceeds which exceed the current basis of destroyed property will not be taxable if the taxpayer replaces that property with similar property within a two-year period from the end of the year in which realization resumed if a natural disaster (fire) or three years from the end of the year in which realization occurred in the event of a government taking (emminent domain).

During the current year, Martin, an unmarried taxpayer residing in Montgomery County, collected $200 interest on U.S. government bonds, $600 on Montgomery County school bonds, and $500 interest on CDs held by the taxpayer. He also received $60 in dividends on Ford Co. Common stock. His gross taxable income from the above is: A- $1,360 B- $760 C- $560 D- $260

Solution: The correct answer is B. Interest on Montgomery County school bonds is excluded under Section 103. He resides in the county of issuance, making it federal, state and local tax free. $200 of interest on a federal bond will be taxed federally (taxed by the issuing entity). $500 of interest from a CD will be taxed. $60 of dividends will be taxed (but at capital gain rates once gross income is down to taxable income). Therefore, gross income is $200 + $500 + $60 = $760.

In June of this year, Mary's office building was damaged during a sudden storm. The fair market value of the building is $225,000 and her basis is $100,000. The storm caused $30,000 in damage. The insurance company awards Mary $20,000 in a settlement. What is Mary's includible gain or deductible loss? A- $30,000 deductible loss. B- $10,000 deductible loss. C- $20,000 includible gain. D- $0; this is a partial loss and not deductible.

Solution: The correct answer is B. Mary's net loss is $10,000 ($30,000 in damage less $20,000 in insurance proceeds) because this was business property.

Tommy Vasquez has taxable income of $200,000. He is concerned about being subject to the alternative minimum tax (AMT). The following income and deductions were included in computing his taxable income. Select one item which may be added to (or subtracted from) regular taxable income in calculating the AMT. A- A long-term capital gain of $100,000. B- A state income tax deduction of $5,000. C- Dividend income of $100,000. D- Personal Residence interest deduction of $25,000.

Solution: The correct answer is B. Options "A", "C" and "D" are incorrect as they are neither added nor subtracted from the regular income to calculate AMT. State income taxes deducted as an itemized deduction will be added back into regular income to calculate Alternative Minimum Taxable Income.

Personal use property that is subject to wear and tear is eligible for cost recovery. A: True B; False

Solution: The correct answer is B. Personal use property is not eligible for cost recovery.

This year, Gail had a Section 179 deduction carryover of $7,800 from the prior year. This year, she elected Section 179 for an asset acquired at a cost of $10,000. Her net income is $135,000 for the current year. Determine Gail's Section 179 deduction for this year. A- $10,000 B- $17,800 C- $13,900 D- $7,800

Solution: The correct answer is B. Section 179 carryover to subsequent years will be limited by the business income before the deduction and the year's Section 179 limitation. In 2023, the limit for Section 179 is $1,160,000. Therefore, the entire amount of $17,800 ($7,800 carryover + $10,000 current year's deduction) is allowable.

Lauren purchased a home worth $1.5 million with an interest-only mortgage of $1.2 million on 12/20/17. She is currently only paying interest on the mortgage in the amount of $60,000 per year. What amount may she deduct as home mortgage interest on Schedule A of her individual income tax return? A- $0 B- $37,500 C- $55,000 D- $60,000

Solution: The correct answer is B. The calculation is calculated by dividing the qualified mortgage over the total mortgage times the interest paid. (750,000/1,200,000) × 60,000 = 37,500 Per TCJA, home mortgages are limited to qualified residential interest and a maximum indebtedness of $750,000 if financed after 12/15/17. (For debt prior to 12/15/17, the $1 million limit applies.)

Which of the following is not a requirement that must be satisfied in order for a legally married taxpayer to use the head of household filing status? I- The taxpayer must file a separate tax return from the spouse. II- The taxpayer must furnish over one-half of the cost of maintaining the household. III- The spouse must not be a member of the household during the last six months of the tax year. IV - The taxpayer must be legally separated from the spouse. A- II only. B- IV only. C- III and IV only. D- I, II and III only.

Solution: The correct answer is B. The taxpayer is not required to be legally separated from the spouse in order to qualify as an abandoned spouse and use the head of household filing status. All of the other options are requirements for qualifying as an abandoned spouse.

Contributions to charity are limited to a certain percentage of income. How long is the carry-over period for individuals to use any excess current charitable deduction? A- One year. B- Five years. C- Seven years. D- Fifteen years.

Solution: The correct answer is B. There is a five-year carry-over provision for charitable deductions. The total years are 6: The initial year plus five carry-over years.

For the year 2022, personal casualty loss deductions are never allowed on Form 1040. A: True B: False

Solution: The correct answer is B. Watch the use of absolutes. NEVER is an absolute. TCJA provides for the use of the casualty loss rules if the area is deemed a federal disaster, making the fact pattern a false statement.

Which would require income recognition by a taxpayer in Year 1? A- John, a cash basis taxpayer, owns apartment building and charges $800 per month rent. Susan, his tenant, offers $600 but John doesn't accept it. Susan then pays John $800 in Year 2. B- Chris, a cash basis physician, performs medical services for a patient who is in a Medicaid program. The state doesn't set the value of the services until Year 2. C- Terri, an accrual basis taxpayer, performs services in Year 1, but doesn't collect cash until Year 2 because the client didn't have money to pay. D- Ron, a cash basis taxpayer, receives restricted stock worth $5,000 in Year 1. The restriction lapses if Ron continues to work for his employer until Year 3.

Solution: The correct answer is C. A is incorrect. A taxpayer will not have constructive receipt if the taxpayer refuses to accept less money than what is owed to him or her. B is incorrect. A taxpayer is not required to report income until the income is known. D is incorrect. Restricted stock is not taxed until the employee no longer has a substantial risk of forfeiture.

Charlie's daughter Deborah is enrolled as a full-time student at Northwest University. Her enrollment began this past summer and she is in her freshman year of studies. Charlie is paying Deborah's tuition. What credits are available to help Charlie off-set the tuition expense? I- American Opportunity credit. II- Lifetime Learning credit. III- Education IRA credit. IV- College Cost credit. A- I only. B- II and IV only. C- I and II only. D- II and III only.

Solution: The correct answer is C. Both ATOC and LLC are available to him for his daughter's freshman year, though he cannot use both at the same time for the same person. It makes sense for Charlie to use the American Opportunity credit for Deborah's first four years as a full-time student. After that, Charlie can switch to the Lifetime Learning credit. Choices III and IV do not exist.

Tara is single and her taxable income is $45,750, which puts her in the 22% tax bracket. How much is her income tax liability for 2023? A- $1,325.50 B- $5,147.00 C- $5,372.50 D- $10,065.00

Solution: The correct answer is C. Have a copy of your CFP Board provided tax tables handy for these types of questions. The calculation is as follows: You are given the taxable income = $45,750 Taxes = $5,147* + [($45,750 - $44,725*) × 22%] = $5,372.50 *Based on the provided tax tables for 2023 for income below $44,725 plus 22% of any income above $44,725. A copy of the tax tables are in the front of your Income Tax Pre-study book or posted online in your learning platform or CFP Board's website. Remember the tax formula: Gross income - Above the line deductions = AGI - Below the line deductions (standard deduction or itemized) - Personal and dependency exemptions (after TCJA expires) = Taxable Income Calculate tax based on filing status - Credits + Other taxes - Prepayments = refund or additional tax due

Which of the following credentials permit an individual to appear before the IRS on behalf of a client? I- An enrolled agent. II- A Certified Public Accountant. III- An attorney IV- A CERTIFIED FINANCIAL PLANNER™. A- II and III only. B- I and II only. C- I, II and III only. D- I, III and IV only.

Solution: The correct answer is C. Only an attorney, enrolled agent, certified public accountant and the taxpayer can represent the taxpayer before the IRS. A CFP® Professional, unless they are also an enrolled agent, CPA or attorney, may NOT represent a third party taxpayer. EDITORS NOTE: Third party is defined as a person or group besides the two primarily involved in a situation, especially in a dispute. Primary and Secondary parties would be the IRS, and the taxpayer's representative.

Payton owns farm land in west Texas where he raises cattle. In March of this year Austin approaches Payton about renting 25% of Payton's land for purposes of growing wheat. Payton and Austin agree that Austin will only pay 3 months of rent at an amount of $8,000 per month if Austin will build a barn on the land, which is the equivalent to 6 months of rent. How much will Payton recognize as rental income this year? A- $8,000 B- $24,000 C- $72,000 D- $96,000

Solution: The correct answer is C. Payton's rental income is the cash received of $24,000 ($8,000 × 3) plus the fair market value of any property received. Since they agreed that 6 months of rent would equal the fair market value of the barn, the additional value is $48,000 ($8,000 × 6).

Lucy, a single mother, has four children, ages 4, 8, 12, and 17. How much will her child tax credit be for the current tax year assuming her AGI is $150,000? A- $2,000 B- $4,200 C- $6,000 D- $8,000

Solution: The correct answer is C. The 17 year old does not qualify since the cut off for children is under age 17 (not including 17). Therefore Lucy will get $2,000 credit for each of her 3 children = $6,000.

Which of the following statements regarding cost recovery periods is/are correct? I- Most cars are 5-year class life assets. II- Most office furniture is a 7-year class life asset. III- Residential rental real estate is depreciated over 27 1/2 years. A- I only. B- I and II only. C- I and III only. D= I, II and III.

Solution: The correct answer is D. All of the above statements regarding recovery periods are correct.

Which of the following are below the line deductions? I- Medical expenses. II- Alimony paid. III- Moving expenses. IV- Penalties for early withdrawal of savings. V- Tax preparation fees. A- I, and V only. B- III, IV and V only. C- II and IV only. D- I only

Solution: The correct answer is D. Below the line deductions include all itemized deductions. Medical expenses are a below the line deduction for itemized taxpayers in the amount exceeding 7.5% of their AGI. Alimony paid is an above the line deductions found in the Adjustment section of the 1040. Moving expenses are not deductible (itemized miscellaneous deduction subject to the 2% floor are suspended based on TCJA). Penalties for early withdrawal of savings are above the line deductions found in the Adjustment section of the 1040. Tax preparation fees are not deductible for the individual taxpayer (suspended based on TCJA). Self-employed can deduct as a business expense, but it will be above the line on their schedule

Capital recoveries include: A- The initial outlay for capital improvements. B- Repair and maintenance expenditures. C- Salvage value. D- Amortization of bond premium.

Solution: The correct answer is D. Capital recovery is the expensing of certain acquisition costs. Bonds purchased at a premium are amortized over their life to expense the premium paid. The theory is that when they mature, their basis will be equal to their face value and not the face plus premium. Bond expenditures are, therefore, a recovery of capital. Instructor Note: Salvage value is the estimated value of an asset at the end of its useful life. It represents the amount that a company could sell the asset for after it has been fully depreciated.

A cash basis taxpayer includes income from a service business when: A- The services are performed. B- The client is invoiced for the services. C- The client's check is deposited in the bank. D- The client's check is received by the taxpayer.

Solution: The correct answer is D. Cash basis is recognized as income when received.

Child support: A- Is deductible when the paying ex-spouse pays the support. B- Is deductible when the child or custodial parent uses the money in support of the child. C- Is deductible when the decree stipulates the payments will be deductible to the payor and includible to the payee. D- Is not deductible.

Solution: The correct answer is D. Child support is not deductible under any circumstances when identified as child support. It is also not taxable to the recipient.

Which of the following are requirements for satisfying the bona fide resident test necessary for excluding foreign earned income? I: The taxpayer must establish permanent quarters in the foreign country for himself and his family. II: The taxpayer may not return to the United States during the year. III: The taxpayer must intend to work in the foreign country for an indefinite period of time. A: I only. B: I and II only. C: II and III only. D: I and III only.

Solution: The correct answer is D. Occasional trips back to the United States for vacation or other purposes will not usually prevent a taxpayer from meeting the bona fide resident test.

Which of the following statements is correct? A: If the tentative AMT is $10,000 and the regular income tax liability is $12,000, the AMT is $2,000. B: If the tentative AMT is $12,000 and the regular income tax liability is $10,000, the AMT is $12,000. C: If the tentative AMT is $10,000 and the regular income tax liability is $12,000, the AMT is a negative $2,000. D: If the tentative AMT is $12,000, and the regular income tax liability is $10,000, the AMT is $2,000.

Solution: The correct answer is D. The AMT is the excess of the tentative AMT over the regular income tax liability.

Which of the following is/are a required test(s) for the deduction of a business expense? A- Ordinary. B- Necessary. C- Reasonable if it is compensation. D- All of the above.

Solution: The correct answer is D. The general rule in determining whether an expense is deductible is whether the expense was Ordinary (usual for the particular trade or business), Necessary (required to assist in the generation of revenues) and Reasonable if it is compensation.

Which of the following is not a disallowed loss? A: Losses on the sale of personal use assets. B: Losses on the subsequent sale of property gifted or sold to a related party when its fair market value is less than the original owner's adjusted basis. C: Loss from a wash sale. D: Capital losses in excess of $3,000

Solution: The correct answer is D. Up to $3,000 of capital losses may be recognized against forms of income other than capital gains in any one tax year. However, if a taxpayer has capital losses in excess of $3,000, these losses are not disallowed, but are carried over to future tax years. A, B and C are not deductible losses.

Which of the following is not an above-the-line deduction? A- Medical expenses. B- A contribution to an HSA. C- A contribution to a traditional IRA. D- Student loan interest.

Solution: The correct answer is A. Medical expenses in excess of 7.5% (Taxpayer Certainty and Disaster Relief Tax Act 2020 made 7.5% permanent) of AGI are deductible below-the-line. All of the other options are above-the-line deductions.

Refundable credits are those that result in a payment to the taxpayer even when the amount of the credit (or credits) exceeds the taxpayer's tax liability. A: True B: False

Solution: The correct answer is A. Most credits are not refundable. Some refundable tax credits include: estimated tax payments, excess Social Security taxes withheld, and earned income credit.

Mike rents his personal residence out each year for 2 weeks during the national golf championship hosted by his neighborhood golf club. The rental income received is $12,000 each year. Mike's rental related expenses are $1,200. How much will Mike recognize as taxable income from the rental activity? A- None B- $10,800 C- $11,954 D- $12,000

Solution: The correct answer is A. No income is required to be reported when renting a personal residence for less than 15 days. Expenses are also disallowed in this situation.

Which of the following represents an addback for alternative minimum tax that would potentially result in a minimum tax credit? A- Bargain element of Incentive Stock Options. B- Casualty loss deduction. C- Interest income from a private activity bond. D- Real estate tax deduction.

Solution: The correct answer is A. The bargain element of an incentive stock option is an addback for alternative minimum tax purposes. In addition, since the addback is considered a deferral item for AMT, it would be eligible for the minimum tax credit in subsequent years. B is incorrect. Casualty loss deductions are not added back for alternative minimum tax purposes. C is incorrect. Although interest income from a private activity bond is an addback for alternative minimum tax purposes, it is considered an exclusion item. Exclusion items are not eligible for the minimum tax credit. D is incorrect. Although real estate taxes are added back for alternative minimum tax purposes, they are considered an exclusion item. Exclusion items are not eligible for the minimum tax credit.

Your client Bebe Rebozo is contemplating the exchange of two parcels of investment land for two similar parcels in two separate transactions. Given the following details of the proposed transaction, compute the amount of recognized gain and loss (if any) on both parcels if your client completes the exchanges: Parcel A: Ten acres of land acquired 15 years ago with a current basis of $50,000. In exchange your client will receive eight acres of land (FMV = $80,000) and $20,000 in cash. Parcel B: Twenty acres of land acquired two years ago with a current basis of $100,000. In exchange, your client will receive twelve acres of land (FMV = $75,000) and $10,000 in cash. A- Parcel A Recognized Gain = $20,000; Parcel B Recognized Loss = $0 B- Parcel A Recognized Gain = $20,000; Parcel B Recognized Loss = $10,000 C- Parcel A Recognized Gain = $50,000; Parcel B Recognized Loss = $10,000 D- Parcel A Recognized Gain = $20,000; Parcel B Recognized Loss = $15,000

Solution: The correct answer is A. This question pertains to like kind exchanges where boot is involved. The rule is that any realized gain will be recognized to the extent of the lesser of realized gain or boot received. In this case, there was a realized gain of $50,000 ($50,000 basis for $100,000 market value). The boot of $20,000 is recognized as gain since it is the lesser of boot or realized gain. Parcel B will have no gain in that there is no realized gain between the basis of the property given up and the fair market value of the property received. There is a realization but it is not recognized. Losses in like kind exchanges are not recognized. Note: A gain or loss is realized when an asset is disposed of, or sold. Recognition is what a like kind exchange is trying to avoid (payment of tax).

Which of the following is deemed passive income? A- Investment income. B- Rental income. C- Self-employment income. D- Alimony.

Solution: The correct answer is B. Alimony from agreements dated prior to 2018 will continue to be considered earned income as is self-employment income. Investment income is portfolio income. Rental income is deemed passive income subject to "active participant" rules.

Josie is 17 years old and qualifies as a dependent for her parents. Josie earned $4,200 in wages and $900 in interest income during the current year. What is Josie's basic standard deduction for the current year? A- $1,250. B- $4,600. C- $5,100. D- $13,850.

Solution: The correct answer is B. Josie is a dependent subject to Kiddie Tax rules due to having unearned income. Kiddie tax rules state the dependent's standard deduction is the greater of $1,250 or earned income plus $400, which ever is greater. Josie's basic standard deduction is equal to her earned income plus $400. Therefore, her basic standard deduction is $4,600.

Section 1245 recapture applies when: A- A gain on sale of real property occurs due to the depreciated basis. B- All or a portion of gain on tangible personal business property resulted from depreciation taken. C- Tangible personal business property is sold regardless of whether there is a gain. D- A gain on any class of property occurs due to cost reduction.

Solution: The correct answer is B. Section 1245 recapture is treated as ordinary income for the gain realized resulting from depreciation taken that was greater than economic reality.

Ginger, age 21 and a full-time student for a degree at State University, qualifies as a dependent on her parents' return. During the summer, she earned $5,500 from a part-time job. Her only other income consisted of $950 interest on a savings account. What is Ginger's taxable income for 2023? A- $0 B- $550 C- $1,250 D- $5,500

Solution: The correct answer is B. The standard deduction for Ginger is the greater of $1,250 or $400 plus earned income but not to exceed the normal standard deduction. Therefore $400 + $5,500 = $5,900 so it is not limited in 2023 (limited to standard deduction for single taxpayer). The total income is $5,500 + $950 = $6,450. Taxable income is $6,450 - $5,900 = $550. Since the unearned income does not exceed $1,250, there is no need to calculate it separately.

Chelsea had to put more money into her rental property this year. She had the exterior of the rental home painted and the roof replaced at a cost of $12,500 and $18,000, respectively. How much is depreciable? A- $0 B- $12,500 C- $18,000 D- $30,500

Solution: The correct answer is C. Painting, inside or out, is considered a repair, which is immediately expensed. The roof replacement is an improvement that substantially prolongs the asset's life, which is capitalized and depreciated over the useful life.

Under the accrual method of accounting, the taxpayer (buyer) recognizes expenses when: A- The goods leave the seller's warehouse. B- The seller accepts the buyer's purchase order. C- The buyer receives the seller's invoice. D- The buyer first uses the goods.

Solution: The correct answer is C. The accrual accounting method recognizes expenses when the legal liability to pay arises. This usually occurs when the invoice is received.

Which of the following can own stock in an S corporation? A- Non-resident alien. B- Publicly-traded corporation. C- Partnership D- Grantor retained annuity trust.

Solution: The correct answer is D. A grantor trust can own stock in an S Corporation. The number of shareholders of an S corporation is limited to 100 and the S corporation can only have one class of stock. LLCs, partnerships, and other corporations are prohibited from becoming S corporation shareholders. Additionally, nonresident aliens and most trusts may not be S corporation shareholders.

Mira is currently working on completing her income taxes. She has calculated her Federal Income Tax before addressing any Add Ons and Credits (FITBAC) to be $10,000. She still needs to factor in the following: Federal Income Taxes withheld: $8,360 Tuition paid towards her masters degree: $5,000 What is her tax due (or refund)? A- $860 refund B- $3,360 refund C- $1,640 due D- $640 due

Solution: The correct answer is D. After calculating the Federal Income Tax Before Add-ons and Credits, credits are backed out, including any federal income tax withheld. Item Amount Mira had tax due of $10,000 Subtract tax withheld- $8,360 Subtract any credits *- $1,000 $640 due *She paid tuition for a masters degree, which would allow her to claim the Lifetime Learning Credit. $5,000 in tuition x 20% = $1,000 credit.

Which of the following statements regarding the adoption expenses credit is/are true? A: The adoption expenses credit is a nonrefundable credit. B: The adoption expenses credit is subject to an AGI phase-out limitation. C: The adoption expenses credit is limited to no more than $14,890 per eligible child in 2022. D: All of these statements are true.

Solution: The correct answer is D. All statements are true

Examples of audit triggers include: A- Unusual or suspicious entries. B- Evidence of unreported income. C- Low reported income for normally high-paying work. D- All of the above.

Solution: The correct answer is D. Among other flags, all of the above can trigger an audit.

Which of the following is not an add-back item for purposes of calculating the Alternative Minimum Tax (AMT)? A- Depreciation of property placed in service after 1986. B- The standard deduction, if taken in lieu of itemized deductions. C- Passive activity losses. D- Installment sales undertaken before March 1, 1986.

Solution: The correct answer is D. Installment sales executed after March 1, 1986 but not before, are add-back items.

Janice is single, had gross income of $38,000, and incurred the following expenses: Charitable contribution $2,500 Taxes and interest on home 9,000 Legal fees incurred in a tax dispute 1,000 Medical expenses 4,000 Student Loan Interest 200 What is Janice's AGI? A: $21,300 B: $28,800 C: $35,500 D: $37,800

Solution: The correct answer is D. Janice's AGI is calculated as follows: Gross income $38,000 Deductions for AGI: Student Loan Interest (200) AGI $37,800 Medical expense, charitable contributions, SALT are all FROM AG

Reese and Jake engage in a like-kind exchange. Reese transfers real estate with a fair market value of $500,000 and an adjusted basis of $200,000 to Jake. Jake transfers real estate worth $700,000 and an adjusted basis of $250,000, plus a $200,000 mortgage on the property, to Reese. What is Jake's potential or deferred gain before and after the transaction? A- $450,000 potential gain before the transaction; $50,000 potential gain after the transaction. B- $250,000 potential gain before the transaction; $50,000 potential gain after the transaction. C- $450,000 potential gain before the transaction; $250,000 potential gain after the transaction. D- $250,000 potential gain before the transaction; $200,000 potential gain after the transaction.

C

Steve has been adjusting his portfolio to meet his target asset allocation and has realized several capital gains and losses this year. $13,000 in short-term capital gains. $9,000 in short-term capital losses. $4,000 in long-term capital gains. $7,000 in long-term capital losses. What is Steve's net gain or loss? A- $1,000 net short-term capital gain. B- $1,000 net long-term capital loss. C- $1,000 net short-term capital loss. D- $1,000 net long-term capital gain.

Solution: The correct answer is A. Because the $4,000 NSTCG and the $3,000 NLTCL have different signs, they can be combined. The resulting $1,000 is a NSTCG because $4,000 is larger than $3,000.

In which of the following situations, if any, may the individual NOT be deemed a dependent of the taxpayer: A- A cousin who does not live with the taxpayer. B- A former brother-in-law who does not live with the taxpayer. The taxpayer is divorced. C- A nephew who does not live with the taxpayer. D- A legally adopted child who does not live with taxpayer.

Solution: The correct answer is A. All other parties either satisfy the relationship or member of the household test. Cousin does not qualify. IRS Publication 501: Relatives who don't have to live with you. A person related to you in any of the following ways doesn't have to live with you all year as a member of your household to meet this test. Your child, stepchild, foster child, or a descendant of any of them (for example, your grandchild). (A legally adopted child is considered your child.) Your brother, sister, half brother, half sister, stepbrother, or stepsister. Your father, mother, grandparent, or other direct ancestor, but not foster parent. Your stepfather or stepmother. A son or daughter of your brother or sister (niece or nephew). A son or daughter of your half brother or half sister. A brother or sister of your father or mother. Your son-in-law, daughter-in-law, father-in-law, mother-in-law, brother-in-law, or sister-in-law. Any of these relationships that were established by marriage aren't ended by death or divorce.

Hansel and Gretel, a married couple, manage apartments and they are required to live in the managers' apartment as a condition of their employment. Instead of providing the apartment to Hansel and Gretel rent-free, the owner of the apartment building gives Hansel and Gretel a housing allowance of $600, which they use to pay rent on the managers' apartment. Hansel and Gretel pay $600 per month in rent. If they did not live in the managers' apartment, Hansel and Gretel could live in another apartment building where they would only pay $500 in rent. What amount, if any, must be included in Hansel and Gretel's gross income? A: $0 B: $100 C: $500 D: $600

Solution: The correct answer is A. An employee is allowed to exclude from gross income the value of lodging furnished by an employer to the employee if the lodging is furnished (1) on the employer's business premises, (2) for the convenience of the employer, and (3) the employee is required to accept the lodging as a condition of employment. It does not matter that Hansel and Gretel were paid a housing allowance, which they were then required to pay back to the employer for rent. Hansel and Gretel can exclude the entire value of their housing from their gross income.

Romo is starting a new business. His top priority is limited liability, but he is also concerned with the ability of the company to raise capital. At some point, he would like to be able to easily sell interests in the business to other investors or perhaps to even take the company public, so he does not want to limit the number of potential owners. Which of the following entities would best suit Romo's needs? A: C corporation B: S corporation C: LLC D: Partnership

Solution: The correct answer is A. Answer B is incorrect because Romo does not want to limit the number of potential owners, and an S corporation cannot have more than 100 owners. Answer C is incorrect; if Romo is interested in taking the company public in the future, the LLC form would not be appropriate. Answer D is incorrect because a partnership would not provide the limited liability that Romo is looking for.

Janice had a car accident in which the other car's driver was cited for reckless driving. An appraisal established that the fair market value of Janice's car declined by $4,000 as a result of the accident. To have the car repaired, Janice paid $5,000. Her insurance company reimbursed her $500. If Janice's AGI for the year was $20,000, determine her deductible casualty loss on the car. A- $0 B- $1,400 C- $1,500 D- $2,400

Solution: The correct answer is A. Casualty loss expense for non-business losses are no longer deductible after 12/31/17.

Albert purchased a tract of land for $140,000 in the beginning of the current year when he heard that a new highway was going to be constructed through the property and that the land would soon be worth $200,000. Highway engineers surveyed the property and indicated that he would probably get $180,000. The highway project was abandoned later in the year and the value of the land fell to $100,000. What is the amount of loss Albert can claim based on this information? A- $0 B- $40,000 C- $60,000 D- $80,000

Solution: The correct answer is A. Changes to the value of land are not taxable events. The question indicates several changes to the fair market value of the property based on proposed local improvements. Because no disposition or sale ever occurred, there is no realized gain or loss.

Which of the following statements is true regarding the education tax credits? A: The American Opportunity credit is not available for an individual that has a felony drug conviction. B: The American Opportunity credit permits a maximum credit of 20% of qualified expenses up to $10,000 per year. C: The American Opportunity credit is calculated per taxpayer, while the lifetime learning credit is available per eligible student. D: Continuing education expenses do not qualify for either education credit.

Solution: The correct answer is A. Choice B reflects the computation of the lifetime learning credit, not the American Opportunity credit. The exact opposite of choice C is correct. Choice D is incorrect since some continuing education expenses can qualify for the lifetime learning credit.

Which of the following statements concerning hobby activities is correct? A: Any activity which generates a profit in the first three years is presumed to be a for profit venture, not a hobby activity. B: The IRS must prove that the taxpayer does not have a profit motive to treat an activity as a hobby activity. C: Expenses associated with the hobby activity can offset, without limitation, the income generated from the activity. D: Hobby income is included in gross income above the line, while hobby expenses are deducted below the line and are subject to a 2% hurdle.

Solution: The correct answer is A. Income generated from a hobby activity is included in gross income, and expenses associated with the hobby will no longer be deductible as a miscellaneous itemized deduction. If the activity earns a profit in three out of five years, it is presumed to be a for profit venture and the IRS has the burden of proof of showing that there is no profit motive avoiding taxpayer's above the line deductible business expense. However, if there has not been a profit in three out of the last five years, the taxpayer has the burden of proof to prove it is a "for profit" venture. There are extended timelines for certain specialty businesses, BUT this is the general rule. Keep in mind - There is no "bright line test" that requires an activity to be treated as a hobby activity based on the income trend of an activity, however for exam purposes the 3 out of 5 year is generally followed.

Five years ago, Linda, a single taxpayer, purchased her first home which was her primary residence. This past year, Linda sold her home. Her realized gain was $27,500. One month later, she purchased a new home for $15,000 less than the old one sold for. How much will Linda have to report as capital gain income? A- None of it. B- All of it. C- The difference between the sale and replacement price and the realized gain ($27,500 - $15,000 = $12,500). D- All of it, but it may be averaged out over five years.

Solution: The correct answer is A. Linda has 1) owned and 2) lived in her home for two years. Under Section 121, Linda, a taxpayer filing single, will be allowed up to $250,000 in excluded gain. Linda's income due to gain was less than the maximum, therefore, she will have no tax liability

Which of the following are permitted deductions for purposes of the AMT for an individual taxpayer? A: Medical expenses to the extent that they exceed 7.5% of AGI. B: Home mortgage interest on a principal residence and two other residences. C: Charitable contributions to the extent they exceed 7.5% of AGI. D: Miscellaneous itemized deductions to the extent they exceed 2% of AGI.

Solution: The correct answer is A. Medical expenses are deductible to the extent that they exceed 7.5% of AGI. Home mortgage interest (i.e., acquisition indebtedness) is deductible on the principal residence and on one other residence subject to $1,000,000 of debt for mortgages prior to 12/15/17 or $750,000 for mortgages after 12/15/17. The charitable contribution is the same as that for regular income tax purposes. Miscellaneous itemized deductions are not deductible for either AMT or regular taxes. SECURE Act 2019 made the 7.5% medical expense floor retroactive for 2019 and extended it to 2020. Taxpayer Certainty and Disaster Tax Relief Act of 2020 extends 7.5% of AGI to taxable years beginning after December 31, 2020.

In Year 1, Kipp invested $65,000 for a 30% interest in a partnership conducting a passive activity. The partnership reported losses of $200,000 in Year 1 and $100,000 in Year 2, Kipp's share being $60,000 in Year 1 and $30,000 in Year 2. How much of the losses from the partnership can Kipp deduct assuming he owns no other investments? A: $0 in Year 1, $0 in Year 2 B: $60,000 in Year 1, $30,000 in Year 2 C: $60,000 in Year 1, $5,000 in Year 2 D: $60,000 in Year 1, $0 in Year 2

Solution: The correct answer is A. No losses are deductible in either year because the activity is passive and he owns no passive income-producing investments. Had Kipp been a material participant, the at-risk rules would have limited the deduction to $65,000 over the two-year period. Therefore, he has a suspended loss for passive activity in Year 1 of $60,000 and an additional suspended passive loss of $5,000 in Year 2 plus a suspended at-risk loss of $25,000 in Year 2.

Which of the following miscellaneous itemized deductions is/are not deductible? A- Losses on IRAs. B- Gambling losses to the extent of gambling income. C- Equity in an annuitized contract at the annuitants death. D- Repayments of income.

Solution: The correct answer is A. Of the above options, only losses on IRAs were subject to the 2% floor under the old law and all miscellaneous itemized deductions subject to the 2% AGI limit were eliminated under TCJA. All of the other options are miscellaneous itemized deductions not subject to the 2% floor.

Which is the best source for obtaining a plain language understanding about the current tax law? A- Commerce Clearing House Federal Tax Guide. B- Congressional Tax Committee Reports. C- Treasury Regulations. D- Tax Court Reports.

Solution: The correct answer is A. Option "A" is correct because Commerce Clearing House (CCH) provides plain language interpretation of tax law. Option "B" is incorrect as the Congressional Committee Reports (sometimes known as the Blue Book) provides congressional reasoning for enacting tax law. This language is often very technical and difficult to understand. Option "C" is incorrect as treasury regulations are official interpretations of the Internal Revenue Code (IRC), and give taxpayers insight into how the Code provisions will be enforced by the IRS. Provided that the Treasury regulations are consistent with the literal provisions of the Internal Revenue Code and Congressional intent at the time the law was passed, the regulations are deemed by the courts to have the full force and effect of law. Option "D" is incorrect because Tax Court Reports provide rulings of the U.S. Tax Court in the form of case law.

Which of the following individuals can make a deductible contribution to a traditional IRA in the current year? A- Jack, who is married, has an AGI of $180,000, and his spouse is an active participant in her employer's defined contribution plan, but he is not an active participant. B- Kelly, who is single, has an AGI of $85,000, and is an active participant in her employer's defined benefit plan. C- Leo, who is married, has an AGI of $145,000, and is an active participant in his employer's defined contribution plan. D- Marni, who is single with income from a family trust.

Solution: The correct answer is A. Option "A" is correct because although Jack's spouse is an active participant, Jack is not. Therefore, Jack may make a deductible contribution to a traditional IRA as long as their joint AGI does not exceed $228,000 in 2023. Option "B" is incorrect because Kelly's AGI exceeds the phaseout range for single individuals who are active participants. Option "C" is incorrect because Leo's AGI exceeds the phaseout range for married individuals who are active participants. Option "D" is incorrect because Marni must have earned income in order to contribute to a traditional IRA. Trust income is investment income. MFJ taxpayers follow one of two phase outs. MFJ 116k-136k for active participants OR if one spouse is not an active participant, the non-active participant follows the spousal IRA phase out, 218k - 228k.

Under what circumstances is a taxpayer required to use a calendar year tax period? A- If the taxpayer does not keep books or accounting records. B- If the taxpayer opens a new business. C- If the taxpayer has an initial tax year of less than 12 months. D-If the taxpayer receives reporting documents such as Forms W-2 and 1099.

Solution: The correct answer is A. Option "B" is incorrect because there is no requirement for new businesses to use a calendar year tax period. Option "C" is incorrect; a taxpayer may use a fiscal year tax period and have a tax year of less than 12 months in the first year. Option "D" is incorrect; although most taxpayers who receive such documents use the calendar year tax period, the receipt of such documents does not in and of itself require them to do so.

Under what circumstances is a taxpayer required to use a calendar year tax period? A: If the taxpayer does not keep books or accounting records. B: If the taxpayer just opened a new business. C: If the taxpayer has a tax year of less than 12 months. D: If the taxpayer receives reporting documents such as Forms W-2 and 1099.

Solution: The correct answer is A. Option B is incorrect because there is no requirement for new businesses to use a calendar year tax period. Option C is incorrect; a taxpayer may use a fiscal year tax period and have a tax year of less than 12 months in the first year. Option D is incorrect; although most taxpayers who receive such documents use the calendar year tax period, the receipt of such documents does not in and of itself require them to do so.

Michael purchased a mutual fund with $25,000 five years ago. During the four years, $6,000 in dividends and capital gains were reinvested in the fund. Today, the fund is valued at $40,000. If Michael sells shares equal to $25,000, which statement(s) is/are correct? I- The taxable gain can be based on an average cost per share. II- The client can choose which shares to sell, thereby controlling the taxable gain. III- To minimize the taxable gain today, the client would sell shares with the higher cost basis. IV- The client will NOT have a gain as long as he or she sells less than what he or she invested. A- I, II and III only. B- I and III only. C- II and IV only. D- IV only.

Solution: The correct answer is A. Option I is correct because it is permissible to determine the per share price of mutual funds using the average cost per share. Option II is correct because it is permissible to select specific identifiable shares of mutual funds to sell. Option III is correct because the higher cost basis will minimize the gain as long as the taxpayer uses the specific identification method. Option IV is incorrect because the sales proceeds will be matched against the basis of the shares sold not the original plus adjusted investment.

Jesse's valuable gun collection was stolen from his home. He purchased the guns for $100,000 and they were worth $125,000 at the time they were stolen. He did not endorse them on his homeowner's insurance policy so he only received $1,000 from insurance. How much may he deduct on his tax return if his AGI is $100,000? A- $0 B- $83,900 C- $88,900 D- $113,900

Solution: The correct answer is A. Personal casualty losses were eliminated for years after 12/31/17 except in cases where a federally declared disaster occurred. Business casualty losses still exist.

Kim and Warren are both 67 years old and healthy. They also have two dependent parents living with them, his mother and her father. They are filing their tax return and want to know how many personal exemptions they may take. You correctly inform them that they can take: A- 0 B- 2 C- 3 D- 4

Solution: The correct answer is A. Personal exemption were eliminated for years after 12/31/17 by TCJA.

Hal is an investment counselor who is employed by Richardson Investments. While he attended a conference on the impact of the new tax laws in investment choices, he incurred the following unreimbursed expenses: Airfare = $350; Lodging = $450; Meals = $330; Tuition and fees = $410. Without regard to his AGI, how much can Hal deduct on his return? A- $0 B- $975 C- $1,150 D- $1,375

Solution: The correct answer is A. Post 12/31/17, TCJA eliminated the deduction for unreimbursed employee business expenses.

Treasury regulations classified by their stage of adoption include all of the following, except: A- Procedural Regulations. B- Proposed Regulations. C- Temporary Regulations. D- Final Regulations.

Solution: The correct answer is A. Procedural regulations are a type of Treasury regulation classified by the function of the regulation. All of the other types of regulations are classified by their stage of adoption.

Jordan was granted a nonqualified stock option by his employer six years ago, giving him the right to purchase 1,000 shares of employer stock at $7 per share. He exercised the option two years ago, when the value of the stock was $12 per share. He sold the shares today at a price of $32 per share. Assuming an ordinary tax rate of 24%, what is the tax due on the sale? A- $3,000. B- $3,750. C- $4,800. D- $6,000.

Solution: The correct answer is A. Read carefully what the question is asking. It is asking about the taxes at the point of the stock sale, not at the time of exercise when the ordinary income (W-2) tax would be due. Jordan's basis in his employer stock is $12 per share (the fair market value at the date of exercise). The grant was at $7 and the FMV at $12 at the time of exercise, and Jordan paid W-2 on the bargain element ($12 - $7 = $5). He sold the stock for $32 per share, resulting in a gain of $20,000 [(32-12) x 1,000 shares]. The gain will be taxed as a long-term capital gain (holding period begins at the date of exercise). Therefore, the rate applied to the gain will be 15%, resulting in a tax of $3,000 (20,000 gain x 15%). Note: Use your CFP Board provided tax tables to see the range of the 24% tax bracket then, apply that to the capital gains tax table. The range falls in to the 15% cap gain rate.

Cindy owned her home six months prior to moving into it. She had lived in her home for 18 months when her employer required that she move to another state to manage its sales office. Her realized gain from the sale of her home is $149,000. Does Cindy have to report income, and if so, how much? A- Cindy does not have to report income because she has owned and used her home for 18 months and her gain is less than the pro rata exemption. B- Cindy reports a gain of $149,000 because she did not meet the owned and used rule. C- Cindy may exclude up to 75% of her actual gain under the allowable exclusion because the move was job related. D- Cindy may exclude up to 75% of the allowable exclusion but must purchase a replacement home within two years at a cost equal to or exceeding the selling price.

Solution: The correct answer is A. Section 121 requires that to qualify for the exemption she must have 1) owned and 2) used as principal residence for two years out of the past 5 years. An exception to the rules exists where a taxpayer moved because of employment transfer. In this case, Cindy owned and lived in the home for 18 of the 24 months. Cindy is entitled to 75% of the allowable $250,000 gain exemption or $187,500. Note: The Section 121 amount is what is pro-rated, not the amount of the gain from the sale of the property. A single taxpayer has a $250,000 exemption, MFJ has a $500,000

Kelly owns a potato chip manufacturing business. The demand for her gourmet potato chips is on the rise. This year her business had a record taxable income year of $750,000. Kelly has decided to invest in an upgraded potato slicing machine that will slice potatoes four times as fast as her previous machine. The cost of the new potato slicer is $2,940,000 and will be the only depreciable property that Kelly places into service during the current year. What is the amount of her Section 179 deduction for the current year? A- $750,000 B- $1,110,000 C- $1,160,000 D- $2,890,000

Solution: The correct answer is A. Section 179 deductions are limited to $2,890,000 of equipment placed into service. Placing the full 2.89 million would give you 1.08 million in deductions. Amounts over the 2.7 million will reduce the maximum deduction dollar for dollar. Kelly's Potato Chip manufacturing business' taxable income was $750,000. Kelly cannot deduct more than her taxable income. $2,940,000 - $2,890,000 = $50,000; $1,160,000 - $50,000 = $1,110,000 limited to the $750,000 of taxable business income.

What is the primary advantage of using the Section 179 Deduction over other cost recovery methods? A- By deducting more currently, total tax liability is reduced and the present value of cash flows is increased. B- The $1,160,000 Section 179 limit allows a businesses to deduct more up front. C- Section 179 reduces the depreciation on most assets to only three years. D- Depreciation applies only to business assets, whereas Section 179 applies to business and personal assets.

Solution: The correct answer is A. Section 179 is an upfront business deduction, now at $1,160,000 (2023) that can be used by businesses to reduce tax liabilities. B is incorrect. It's possible to reduce Section 179 deduction to zero, depending how much is placed into service. If too much is placed into service, Section 179 would not have any advantages over other methods of depreciation. Any asset value placed into service over $2,890,000 reduces the deductible maximum of $1,160,000 dollar for dollar. C is incorrect. Section 179 is one year deduction. D is incorrect. Section 179 does not apply to personal assets.

On December 1, Andrea reviews her investment portfolio and finds out that she has had a very profitable year. To offset some of her gains, Andrea sells 100 shares of Big Bear Corporation for $10,000. She purchased those shares for $15,000 two years earlier. On December 25 of the same year, Andrea reads a newspaper article indicating that the price of Big Bear Corporation is expected to increase substantially. Second-guessing the wisdom of selling her previous shares of Big Bear stock, she purchases 100 shares of Big Bear Corporation for $8,000. What are the tax consequences to Andrea this year? A- A $5,000 realized, but not recognized loss. B- An $8,000 realized and recognized loss. C- A $5,000 realized and recognized loss. D- A $7,000 realized, but not recognized loss.

Solution: The correct answer is A. Since Andrea purchased and sold substantially identical securities within 30 days, a wash sale occurs. Her realized loss on the sale of the original shares is calculated as follows: Amount Realized: $10,000 Less: Adjusted Basis -$15,000 Equals: Gain or (Loss) ($5,000) Due to the wash sale transaction, however, Andrea will not be permitted to recognize the loss in the year it was incurred. Instead, the realized but unrecognized loss of $5,000 will be added to the basis of the replacement securities. Andrea purchased the replacement securities for $8,000 so adding the unrecognized loss increases her basis to $13,000. By increasing basis in the amount of the unrecognized loss, Andrea will receive that back tax free when she ultimately sells the stock.

Ian, a single taxpayer, received $15,000 of Social Security retirement benefits in the current year. He also received $16,000 of interest income. How much of Ian's Social Security benefits must be included in his gross income? A- $0. B- $7,500. C- $12,750. D- $15,000.

Solution: The correct answer is A. Since the total of Ian's MAGI ($16,000) and one-half of his Social Security benefits (0.50 × $15,000 = $7,500) is less than the base amount ($25,000), none of his Social Security benefits are included in gross income.

Ian, a single taxpayer, received $15,000 of Social Security retirement benefits this year. He also received $16,000 of interest income. How much of Ian's Social Security benefits must be included in his gross income? A: $0 B: $7,500 C: $12,750 D: $15,000

Solution: The correct answer is A. Since the total of Ian's MAGI ($16,000) and one-half of his Social Security benefits (0.50 × $15,000 = $7,500) is less than the base amount ($25,000), none of his Social Security benefits are included in gross income.

Rita earns a salary of $150,000, and invests $40,000 for a 20% interest in a passive activity. Operations of the activity result in a loss of $250,000, of which Rita's share is $50,000. How is her loss characterized? A: $40,000 is suspended under the passive loss rules and $10,000 is suspended under the at-risk rules. B: $40,000 is suspended under the at-risk rules and $10,000 is suspended under the passive loss rules. C: $50,000 is suspended under the passive loss rules. D: $50,000 is suspended under the at-risk rules.

Solution: The correct answer is A. Step 1 - Rita invested $40,000 this is her at risk amount. Her share of the passive loss is $50,000, only $40,000 can be used due to at risk rules. $10,000 is suspended here. Step 2 - $40,000 of loss made it passed the at risk rules, now look for any passive income to offset. Rita has no passive income, so the $40,000 loss is suspended under the passive loss rules.

In the current year, Susan and Tom had three preschool-aged children who require daycare so that Susan and Tom work. Their total daycare costs for all three children was $6,500. While at daycare, 1/3 of the time is education while the remainder is custodial care. Assuming that Susan and Tom have an AGI of $110,000, what is the dependent care credit amount? A- $1,200 B- $1,300 C- $4,800 D- $6,500

Solution: The correct answer is A. Sue and Tom will be able to take a credit of $1,200 ($6,000 × 20%). Although not normally allowable as deductible child care, education expenses for children preschool through kindergarten are qualified expenses. The amount of costs that qualify is the lesser of actual costs or $3,000 for one qualified individual, and $6,000 for two or more qualified individuals. If AGI is greater than $43,000 the allowed credit is 20%.

This year, Roberto's daughter Juanita started her freshman year at a local university. Jaunita's intent is to obtain a degree in Biology and possibly enter medical school. Roberto intends to pay for Juanita's tuition. What credit should Roberto use during Juanita's first year? A- The American Opportunity Tax credit. B- The Lifetime Learning credit. C- Both the combined American Opportunity Tax and Lifetime Learning credit. D- None, since it is the taxpayer's daughter who is attending the university and he is paying.

Solution: The correct answer is A. The American Opportunity Tax credit would make the most sense in that it is designed for the first four years of a post secondary degree program and offers the highest maximum credit of $2,500 versus $2,000 for the Lifetime Learning credit. Also, a taxpayer cannot take both the Lifetime and American Opportunity credits in the same year for the same student.

This year, a client purchased an expensive piece of equipment for his small business, costing $2,500,000. The client's net income is estimated to hit $1,000,000 this year. Which of the following depreciation methods should be used if the client wishes to maximize tax depreciation? A: Modified Accelerated Cost Recovery System (MACRS). B: Accelerated Cost Recovery System (ACRS). C: Section 179. D: Sum of the year's digits.

Solution: The correct answer is A. The Section 179 expensing will be limited to the client's net income. Using MACRS will allow the client to depreciate over the equipment's life expectancy. Option "A" is correct because it is an acceptable method of tax depreciation as well as one which will yield the greatest expense in the early portion of the asset's life. Option "B" is incorrect because ACRS application was discontinued after 1986. Option "C" is incorrect because section 179 would limit the expensing to one year and $1,160,000 maximum deduction in 2023 (further limited by net income). Option "D" is incorrect because it is not a recognized depreciation method for tax purposes.

Kara owns two assets that she is considering selling. One has appreciated in value by $3,000 and the other has declined in value by $3,000. Both assets are held for personal use. Kara believes that she should sell both assets in the same taxable years so that the loss of $3,000 can offset the gain of $3,000. Advise Kara regarding the tax consequences. I- In this case, the appreciated property will cause capital gain to be recognized. II- In this case, the personal use property from which the loss is realized is not deductible. III-In this case, the personal use property from which the loss is realized is deducted from capital gains. IV- In this case, Kara has no immediate net tax consequences. A- I and II only. B- II and IV only. C- I, III and IV only.

Solution: The correct answer is A. The appreciated property will cause a capital gain to be recognized. Because the property realizing a loss was for personal use, it will not be deductible. Capital gain on personal use property is taxable, but losses on personal use property (property not used in business) are not deductible. Therefore, the $3,000 in gains will be taxed at the appropriate capital gains rates but the $3,000 loss can not be used as a deduction to income or to capital gains.

The constructive receipt doctrine: A- Applies to a secular trust used for deferred compensation. B- Does NOT apply to cash basis taxpayers. C- Is used to distinguish unearned income from earned income. D- Means that a taxpayer cannot plan transactions to defer the recognition of income.

Solution: The correct answer is A. The cash method of accounting recognizes income when received. Receipt may be either actual or constructive. Constructive receipt is when the taxpayer has the right to the money although they are not in possession thereof. A secular trust constructively belongs to the beneficiary therefore constructive receipt applies.

Albert holds two jobs, a full-time job with Stork Company and a part-time job with Swan Corporation. Albert uses his personal car to get to work, and the mileage involved is as follows: Albert's home to Stork = 30 miles; Stork to Swan = 20 miles; Swan to Albert's home = 35 miles. Albert's deductible mileage for each work day is: A- 0 miles. B- 25 miles. C- 35 miles. D- 55 miles.

Solution: The correct answer is A. The deduction is eliminated for years after 12/31/17.

Maria Blue spent $12,000 in day care services for her 4 children to allow her to work. If her adjusted gross income is $100,000, how much is her dependent care credit? A- $1,200 B- $2,400 C- $6,000 D- $12,000

Solution: The correct answer is A. The dependent care credit is not phased out and provides a credit of 20% on up to $3,000 of expenses per qualifying child, with a maximum of $6,000 for two or more children. She spent $12,000 for 4 children, or $3,000 on each child. The maximum expenses she can use for 2 or more children is $6,000. Her expenses are limited. The $6,000 in expenses x 20% = $1,200 in child care credit.

Melissa is an employee and regional manager of "Burger Heaven," a national chain of fast-food outlets. On a particular day, she uses her personal car for the following trips: Melissa's home to post office to get Burger Heaven mail = 10 miles; Post office to her office = 30 miles; Office to Burger Heaven No. 1 = 20 miles; Burger Heaven No. 1 to Burger Heaven No. 2 = 25 miles; Burger Heaven No. 2 to home = 40 miles. Melissa's deductible mileage for the day is: A- 0 miles. B- 20 miles. C- 45 miles. D- 75 miles.

Solution: The correct answer is A. The first trip out to the post office and the last trip home are considered non-deductible commuting miles. However as an employee, Melissa may not deduct unreimbursed employee expenses.

One of the five tests which must be met to qualify as a dependent is: A- Whether the dependent files a joint return that year. B- That the dependent owes taxes from a previous year. C- Will the dependent owe taxes this year. D- The unearned income reported by the dependent.

Solution: The correct answer is A. The five dependency tests are: 1) Gross Income Test, 2) Support Test, 3) Member of Household or Family Member Test, 4) Citizenship Test (U.S., Canada or Mexico), and 5) Joint Filing Test.

One of the five tests which must be met to qualify as a dependent for a qualifying relative is: A- Gross income of the dependent. B- The ability of the dependent to provide for their own care. C- The age of the dependent. D- Whether the dependent owes income taxes from a previous year having filed single.

Solution: The correct answer is A. The five dependency tests are: 1) Gross Income Test, 2) Support Test, 3) Not a qualifying child, 4) Citizenship Test (U.S., Canada or Mexico), and 5) Joint Filing Test.

Jacqui, single and age 45, sold her personal home for $176,000 in October of last year after living there for ten years. The selling expenses were $10,000 and her adjusted basis in that home was $132,000. She rented an apartment until May of this year when she bought and moved into a new personal residence. She paid $155,000 for the new home. What is the maximum gain that Jacqui must report in connection with sale of her principal residence? A- $0 B- $11,000 C- $21,000 D- $34,000

Solution: The correct answer is A. The gain on the sale of the home was $176,000 less $10,000 in selling expenses for an adjusted selling price of $166,000. The basis of the home sold was $132,000 for a realized gain on sale of $34,000. Under Section 121, a taxpayer who 1) owns and 2) lives in their primary residence for two years will be able to exempt up to $250,000 in gain if single ($500,000 if married filing joint.) Here the gain is below the $250,000, so all of the gain will be exempt.

There are specific requirements that must be met to qualify for an individual "real estate investor exception" to the passive activity loss rules. Which of the following is not one of those requirements? A- The taxpayer must materially participate in the activity. B- The taxpayer must own at least 10% of the value of the real estate. C- The taxpayer must have an AGI of less than $150,000. D- The taxpayer must actively participate in the activity.

Solution: The correct answer is A. The taxpayer is not required to materially participate in the activity, but the taxpayer must actively participate in the activity. Material participation requires substantial, continuous involvement in the operation of the activity. Active participation means that the taxpayer participates in making management decisions concerning the property, but is not substantially and continuously involved in the operation of the activity. Summary of Rental Property Exceptions to Passive Categorization 1. Customer use less than or equal to 7 days 2. Customer use less than or equal to 30 days and significant personal services provided 3. Extraordinary personal services are provided 4. Rental activities incidental to non-rental activity 5. Rental activity available during business hours for nonexclusive use of customers 6. Rental property used in an activity conducted by partnership, etc. where the taxpayer is the owner and an active participant

Sara and Bill have rental property that was rented this year to a family whose primary bread winner lost his job. As a result they had uncollected rent for 2 months before they began the eviction process and 1 additional month before the family was finally evicted. If the rent was $800 per month, how much of a deduction may Sara and Bill claim on their income tax return for the uncollected rent? A- None, uncollected rent is not deductible. B- $800 of uncollected rent for the month it took to evict the renters. C- $1,600 of uncollected rent before action was taken to evict the renters. D- $2,400 of uncollected rent for the entire delinquency.

Solution: The correct answer is A. They cannot deduct from ordinary income an amount that was never included in taxable income. Therefore, Sara and Bill will not receive any deduction for the uncollected rents.

Elton and Elsie are husband and wife and file a joint return for this year. Both are under 65 years of age. They provide more than half of the support of their two daughters, Karen (age 17) and Kristie (age 25). Kristie is a full-time medical student. Kristie receives a $5,400 scholarship covering her room and board at college. They furnish all of the support of Hattie (Elton's grandmother), who is age 70 and lives in a nursing home. How many qualified dependent credits ($500) are Elton and Elsie potentially entitled to receive? A: Two B: Three C: Four D: Five

Solution: The correct answer is A. Two: One for Karen, she is a qualifying child but has aged out at 17, and one for Hattie. Kristie is not a qualifying child—although a full-time student, she is not under age 24 and she does not meet the qualifying relative category due to the gross income test—the type of scholarship aid she receives is taxable. Hattie is not a member of the household but satisfies the relationship test.

Which of the following is a 1231 asset subject to 1245 recapture? A: Machine, held 2 years, used in trade or business B: Personal Car, held 3 years C: Computer, held 6 months, used in trade or business D: Building, held 3 years, used in a trade or business

Solution: The correct answer is A. Type of Property1231 Asset Subject to 1245 Recapture1231 Asset Subject to 1250 RecaptureCapital Gain PropertyMachine, held 2 years, used in trade or businessX Computer, held 6 months, used in trade or business XPersonal Car, held 3 years XBuilding, held 3 years, used in trade or business X

Billy, single (he divorced in 2010) and age 42, has the following items of income and expense for the current tax year. Wages: $60,000 Interest: $1,200 Inheritance: $50,000 Alimony paid: $10,000 Child support paid: $8,000 Federal taxes paid: $5,000 State taxes paid: $2,000 Medical expenses: $7,500 Tickets from his employer for one basketball game: $100 What is his taxable income for 2023? A- $37,350 B- $46,150 C- $51,200 D- $87,350

Solution: The correct answer is A. Wages$60,000Interest$1,200Less Alimony*<$10,000>AGI$51,200Std Deduction**$13,850Taxable Income $37,350 *Divorce after 12/31/2018 will follow the post TCJA rules; no deductions, or inclusion in income. Divorces finalized by 12/31/2018 will remain under the old rules; Alimony will be deductible by the payor, and included in income for the payee. **The taxpayer gets either the standard deduction or the itemized deduction (below the line), not both. In this case, the itemized deductions would be $5,660 (state taxes of $2,000 plus the medical in excess of 7.5% of AGI: $51,200 × 7.5% = 4,500; medical of $7,500 − $3,840 floor = $3,660 deductible). The standard deduction is used since it is the greater of the two. A ticket to a sporting event is considered a de minimus employee benefit and not subject to tax. If the employee was given season's tickets, then the amount would be included in income. Inheritances are not considered income for federal tax purposes.

When reviewing a client's income tax return from the prior year you notice that they had adjusted gross income of $175,000 and paid federal income tax of $31,500. Assuming that the client's income for this year will closely approximate that of last year, what is the minimum amount to pay in estimates to meet safe harbor? A- $28,350 B- $31,500 C- $34,650 D- $37,800

Solution: The correct answer is A. Your primary choices are (A) = 90% of current year, (B) = 100% of prior year, and (C) = 110% of prior year. Since the client's AGI from last year is greater than $150,000 the safe harbor is 90% of current year tax or 110% of prior year tax. Since the client's income "closely approximates that of last year" we can utilize the 90% of current year amount. The question asked us for the minimum amount. If the client's income varies widely then we would use 110% of prior year.

John owns a rental home in Arizona. He decided that he would like to acquire a rental home in Washington. Ted who lives in Washington has a rental home. For health purposes, Ted must relocate to Arizona. John and Ted decide to exchange properties under section 1031 of the code. The other facts pertaining to the exchange are: Ted's Basis = $100,000 John's Basis = $75,000 Ted and John exchange the two properties, but Ted has to give John an additional $25,000 in cash. The fair market value of Ted's property is $100,000, and the fair market value of John's property is $125,000. What is John's recognized gain? A- $0 B- $25,000 C- $50,000 D- $75,000

Solution: The correct answer is B.

In September of this year, Rudolph refinanced his home. Prior to refinancing, his only outstanding debt was the balance due on his original mortgage of $110,000. Rudolph needed some additional money to pay for his child's college education and to take advantage of an investment opportunity, so upon refinancing, Rudolph took out a 30-year mortgage for $250,000. To reduce the interest rate on the mortgage, down to 5%, Rudolph paid $2,500 in points on refinance. Which of the following statements is correct? A: Rudolph can deduct all of the mortgage interest paid on the note. B: Rudolph can deduct the mortgage interest incurred on $110,000, plus a pro rata portion of the points paid on refinancing. The remaining interest is not deductible. C: Rudolph can only deduct the mortgage interest incurred on $210,000. D: Rudolph can only deduct the mortgage interest paid on $210,000 plus a pro rata portion of the points paid on refinancing.

Solution: The correct answer is B. $110,000 of the refinanced amount continues to be treated as acquisition indebtedness since that was the previous balance of Rudolph's mortgage. Post TCJA, if you refinance under the amount of original indebtedness, you can continue the pre-TCJA rules, if you refinance more (as in this case) you need to follow the new rules. You can deduct the original indebtedness (110k) up to $750,000. A home equity can only be included if it is used to better the property. So under these rules, they can deduct interest on $110,000, the home equity was not used for a qualified reason and will not be deductible. The pre TCJA would allow up to $100,000 in home equity for interest deductions, 40,000 would have not been deductible, leaving 210k that could have been deducted under the old rules Since not all of the new mortgage is considered acquisition indebtedness, only a portion of the points paid on the refinance will be deductible.

In September of this year, Rudolph refinanced his home. Prior to refinancing, his only outstanding debt was the balance due on his original mortgage of $110,000. Rudolph needed some additional money to pay for his child's college education and to take advantage of an investment opportunity, so upon refinancing, Rudolph took out a 30-year mortgage for $250,000. To reduce the interest rate on the mortgage, down to 5%, Rudolph paid $2,500 in points to refinance. Which of the following statements is correct? A- Rudolph can deduct all of the mortgage interest paid on the note. B- Rudolph can deduct the mortgage interest incurred on $110,000. The remaining interest is not deductible. C- Rudolph can only deduct the mortgage interest incurred on $210,000. D- Rudolph can only deduct the mortgage interest paid on $210,000 plus the points paid on refinancing (ratably over the life of the loan).

Solution: The correct answer is B. $110,000 of the refinanced amount continues to be treated as acquisition indebtedness since that was the previous balance of Rudolph's mortgage. An additional $140,000 will be considered home equity indebtedness but is not deductible based on its use. Anytime a pre-TCJA mortgage is refinanced for more than the existing balance, the mortgage will follow the current TCJA rules. C and D represents the amount under the pre-TCJA rules; up to 100,000 of home equity could be deducted in addition to the mortgage. Under TCJA only qualified residential interest (acquisition indebtedness plus home improvement portions of additional loans) may be deducted and that is capped at $1 million of debt ($750,000 for contracts after 12/15/17).

Iris, a widow, elected to receive the proceeds of a $50,000 face value life insurance policy on the life of her deceased husband in ten annual installments of $6,800 each beginning in 2019. In the current year, she received $6,800. Iris died in December of this year after collecting the installment payment. What is the amount subject to income tax on her final return? A- $0 B- $1,800 C- $5,000 D- $6,800

Solution: The correct answer is B. $50,000 ÷ 10 = $5,000 principal each year. $6,800 - $5,000 = $1,800 of interest. The $1,800 of interest must be included in Iris' gross income. Proceeds from a life insurance policy on the death of an insured are never taxable. Interest received even from installation payments of the proceeds of a life insurance policy are taxable as ordinary income. Because Iris received $1,800 in interest while alive, the interest will be included in the gross income on her final return. She receives no deduction for her remaining unrecovered basis because it is from life insurance. If it had been from an ordinary commercial annuity, she would have a deduction for any nonrecovered basis in the annuity.

Which of the following income(s) is/are NOT subject to Social Security tax? I- Rental real estate income. II- Small part-time repair shop income as a proprietor. III- Shareholder's share of S corporation's income in excess of salary. IV- Income of an individual working as an independent contractor. A- I, II and III only. B- I and III only. C- II and IV only. D- II, III and IV only.

Solution: The correct answer is B. (Options "I" and "III"). Neither income from rental real estate nor S corporation distributions are subject to self-employment taxes. Option "I" - Income from rental real estate is not subject to self-employment taxes. Option "II" - This implies there is no employee status, and therefore, if services are being provided then the individual's income from those services will be subject to self-employment taxes. Option "III" - A shareholder's distributive share of S corporation profits is recognized as ordinary income and not subject to self-employment taxes. Option "IV" - Income from work as an independent contractor is subject to self-employment tax. Self employment tax is the employee and employer portion of Social Security and Medicare tax for a total of 15.3%

Which of the following entities would be required to adopt a calendar year for federal income tax purposes? A- Estate. B- Irrevocable life insurance trust. C- Partnership. D- S Corporation.

Solution: The correct answer is B. A trust must adopt a calendar year for federal income tax purposes. A is incorrect. The executor of an estate can elect a fiscal year end for the estate. C is incorrect. A partnership must adopt the tax year end of its partners. If the partners have a fiscal year, the partnership can have a fiscal year. For example, if a partnership has two corporations as its partners, and the corporations have June 30 year ends, the partnership would have a June 30 year end. D is incorrect. An S corporation can have a fiscal year end for tax purposes.

Earl went from Portland, Oregon to New York City on business. After six days of business meetings promoting his new winery in hopes of expanding distribution, he took four days of vacation to go sightseeing. Earl's expenses for the trip are as follows: Airfare = $1,200 Lodging (10 days × $90) = $900 Meals (10 days × $100) = $1,000 Airport limo = $60 Earl's business deduction is: A- $2,400 B- $2,100 C- $1,896 D- $1,620

Solution: The correct answer is B. AFTER DECEMBER 31, 2022 The expenses associated with a trip that are for non-business purposes are not deductible from income. Remember that meals are only 50% deductible. Accordingly, the answer is calculated as follows: $1,200 (airfare) + $60 (limo) + $540 (hotel - 6 days at 90/day - the remaining 4 days are personal and not deductible) + $300 (meals - 6 days at 100/day × 50% meal allowance) = $2,100. The airfare (to and from) was transportation for business that would not change whether it was personal or not, so it is not pro-rated but rather fully deductible. $1,200 + $60 + $540 + $300 = $2,100. Airfare Deductibility Rules: Domestic: If primarily business then deduct all airfare. Prorata meals and lodging. Foreign: Prorata meals and lodging. Prorata airfare unless (then you can deduct all): < 7 days <25% on personal No control; vacation not a deciding factor

Which of the following authorized the first constitutional federal income tax? A: Revenue Act of 1861 B: 16th Amendment C: Revenue Act of 1916 D: None of the above

Solution: The correct answer is B. Answer A is incorrect because although the Revenue Act of 1861 did impose a federal income tax, it was later found to be unconstitutional because Congress did not have the power to levy an individual income tax at that time. Answer B is correct because the 16th Amendment gave Congress the power to impose an individual income tax, but did not itself impose that tax. Answer C is incorrect because the Revenue Act of 1916 raised the rates previously imposed under the Revenue Act of 1913.

Lisa owns the original copy of the Moaning Myrtle, one of the few paintings created by a renowned renaissance artist and inventor. The Moaning Myrtle has been in Lisa's family for years, but she is getting older and moving into a smaller house and none of her children want the painting. When Lisa inherited the Moaning Myrtle from her father's estate, the value was estimated to be $25,000, but an art expert recently appraised the painting at $750,000. Lisa wants the painting to be cared for, so she donates it to a local art museum, which has agreed to display the painting. Assuming that Lisa's AGI is $100,000 this year, which of the following statements is correct? A- Lisa can deduct $60,000 this year and carry forward $690,000 for the next five years. B- Lisa can deduct $30,000 this year and carry forward $720,000 for the next five years. C- Lisa's deduction is limited to her cost basis of $25,000. D- Lisa's charitable deduction carry-forward will be $730,000.

Solution: The correct answer is B. Because the Moaning Myrtle will be put to a use that is consistent with the museum's tax-exempt purpose, Lisa can use the fair market value of the painting to calculate her charitable deduction. However, her deduction this year is limited to 30% of her AGI. Therefore, Lisa can deduct $30,000 this year and carry forward the remaining $720,000 of her charitable deduction for up to five years.

Brady is starting a new business where he will be the only owner. He would like to have limited liability, but he would prefer flow-through taxation because he expects to have losses in the first few years. He is not concerned about incurring self-employment taxes. He does not want to have to file a separate tax return for the entity. Which of the following entities would best suit Brady's needs? A: Partnership B: Single-member LLC C: S corporation D: Proprietorship

Solution: The correct answer is B. Both the proprietorship and the single-member LLC would provide the flow-through taxation that Brady is looking for, but only the LLC would also provide limited liability. An S-corp would require filing 1120S, however, a single member LLC can be reported on Schedule C.

Cary invested $100,000 in an annuity contract. This year, Cary annuitized the contract. The insurance company agreed to pay Cary $520.83 per month for 20 years. Assuming that Cary receives eight payments this year, how much can Cary exclude from his gross income this year? A- $833.33. B- $3,333.31. C- $4,164.64. D- $6,249.96.

Solution: The correct answer is B. Cary's expected return is: $520.83 x 12 x 20 years = 124,999.20The original amount of 100,000 makes up 80% of the contract and is a return of principal.This year, Cary will receive 520.83 x 8 payments = 4,166.64 of which 80% is return of principle = 3,333.31 is tax free or excluded. 4,166.64 - 3,333.31 is excluded = 833.33 will be included in his income this year.

Disability income benefits are taxable to the recipient when: A: The taxpayer pays the premiums. B: Someone other than the taxpayer pays the premiums. C- The taxpayer is disabled but continues part-time work. D- The taxpayer is not employed during the period that the income is received.

Solution: The correct answer is B. Disability income is taxable to the taxpayer when someone other than the taxpayer pays the premiums or provides the benefit as in the care of a company who self insures employees for disability losses.

Albacore, Inc., an accrual method taxpayer, was incorporated on January 2 this year but did not begin business operations until July 1. Albacore adopted a calendar year tax year and incurred the following expenses during its first tax year: Incorporation fees paid to State: $150 Expenses in connection with issuing and selling stock: $1,800 Legal fees incident to incorporation: $1,650 If Albacore, Inc. makes an appropriate and timely election, the maximum organizational expenditures that it can properly deduct for the current year would be: A- None B- $1,800 C- $3,450 D- $3,600

Solution: The correct answer is B. Expenses incurred in connection with issuing and selling stock are not deductible. The rule is the lesser of expenditures or $5,000. Therefore, $1,650 + 150 = $1,800. Additional information: A corporation can deduct up to $5,000 of business startup costs under Sec. 195. The $5,000 deduction is reduced dollar for dollar (but not below zero) by the cumulative amount of startup costs exceeding $50,000. The remaining startup costs can be deducted ratably over a 15-year period (consistent with the amortization period for Sec. 197 intangibles), beginning with the month in which the active trade or business begins (Sec. 195(b)(1)). Active conduct of a trade or business generally occurs when the corporation has begun the conduct of operations for which it was organized (i.e., is in a position to begin generating revenue). Startup costs are costs paid or incurred in connection with investigating the creation or acquisition of an active trade or business or creating an active trade or business. Startup costs include amounts paid or incurred in connection with an existing activity engaged in for profit, and for the production of income in anticipation of the activity becoming an active trade or business. To be a startup cost, the expenditure must have otherwise been deductible as an ordinary and necessary business expense under Sec. 162. Expenditures that would have otherwise been capitalized, such as the costs associated with the construction of a capital asset, are not startup costs (Rev. Rul. 81-150).

Linwood files his tax return 65 days after the due date. Along with the return, Linwood remits a check for $6,000 which is the balance of the tax owed. Disregarding the interest element, Linwood's total failure to file penalty is: A: $90 B: $810 C: $900 D; $990

Solution: The correct answer is B. Following the procedure set forth in Chapter 2, the penalty is determined as follows: Failure to pay penalty [1/2% × $6,000 × 3 (three months violation)] $90 Plus: Failure to file penalty [5% × $6,000 × 3 (three months violation)] $900 Less: Failure to pay penalty (90) Total Failure to file Penalty $810 Total Penalties (failure to file plus failure to pay - if it had asked for it) $900 Watch these questions, this one ONLY asks for the FTF penalty! NOTE: 3 months is from April 15 - May 15 (30 days) May 16 - June 15 (30 days) June 15 - June 20 (5 days) Total 65 days (April 15 - June 20)

Billy, single (he was divorced in 2010) and age 42, has the following items of income and expense for the current tax year. Wages: $60,000 Interest: $1,200 Inheritance: $50,000 Alimony paid: $10,000 Child support paid: $8,000 Federal taxes paid: $5,000 State taxes paid: $2,000 Medical expenses: $7,500 Tickets from his employer for one basketball game: $100 What is his adjusted gross income? A- $49,200 B- $51,200 C- $51,300 D- $61,200

Solution: The correct answer is B. His income includes the wages and interest and the alimony paid is deductible for AGI. Therefore his AGI = $60,000 (wages) + $1,200 (interest) - $10,000 (deduction for alimony paid*) = $51,200. Note that the basketball tickets are "occasional" and thus a tax-free fringe benefit. *Alimony agreements made on or before 12/31/2018, and not materially altered, will follow the old alimony rules with deduction for the payor, and inclusion in income for the recipient. Alimony agreements made or materially altered after 12/31/2018 will follow the new rules (no deduction or inclusion in income).

Harvey works for Ice Cream Dream, a company that sells commercial ice cream makers. Ice Cream Dream normally has a gross profit percentage of 30%. Harvey's wife loves ice cream, so he decides to buy her a commercial ice cream maker for her birthday. Harvey paid $650 for a machine that would have normally retailed for $1,000. What, if any, amount must be included in Harvey's gross income? A: $0 B: $50 C: $350 D: $400

Solution: The correct answer is B. Ice Cream Dream's normal gross profit percentage is 30%. Therefore, Harvey must have paid at least $700 for the machine in order to avoid inclusion in his gross income. However, Harvey's discount was 35%. Therefore, Harvey must include 5% of the discount, or $50, in his gross income.

Your client is the sole shareholder of a closely-held corporation. In the current year, the IRS has deemed the operation to be a Personal Holding Company (PHC) because of involvement in a number of investments other than the stated business purpose. It has cited the business as having undistributed holding company income. What are the possible implications of this decision for your client? A- A penalty of 35% can be imposed on the value of the assets of the PHC because of potential fraud. B- A penalty tax of 20% can be imposed on the undistributed PHC income. C- A penalty tax of 35% can be imposed on the undistributed PHC income. D- Because the corporation is a PHC, there is no penalty tax, but the PHC will be required to pay tax at the 20% corporate level.

Solution: The correct answer is B. If the business is deemed to be a PHC by the IRS, then a penalty tax of 20% can be imposed on the undistributed personal holding company income.

Alberto (age 65) and Stephanie (age 58) are raising their grandson, Jackson, who is blind and qualifies as their dependent. What is Alberto and Stephanie's standard deduction in 2023 if they file a joint income tax return? A- $27,700 B- $29,200 C- $30,700 D- $32,200

Solution: The correct answer is B. In 2023 the standard deduction for a married couple is $27,700. Since Alberto is 65 years of age or older he may also take an additional standard deduction of $1,500. $27,700 + $1,500 = $29,200. Their grandson's blindness or being a dependent has no bearing on their standard deduction.

Donna Bella, whose AGI is $250,000, also has passive income of $125,000 and passive losses of $150,000. She uses $125,000 of her passive losses to offset the passive income and the rest is subject to suspension. She has come to you today to find out which of the following ideas is the best possibility for reducing her current tax liability. A- An investment in an oil by-product and natural gas limited partnership generating losses. B- An investment in an activity producing credits. C- An investment in rental real estate as an active participant designed to generate losses. D- An investment in limited partnership historic district rehab project producing both credits and passive losses.

Solution: The correct answer is B. In this case, more losses are not required at all. Rather, credits only are needed to improve the client's tax position.

Caterina inherited stock from her late uncle Billy. The stock was worth $5,500 at Billy's death, and was purchased by Billy one month before death for $5,000. If the current price is $6,000, and Caterina must pay $250 of commissions to sell the stock, what is her gain? A- $250 Short-term capital gain. B- $250 Long-term capital gain. C- $500 Short-term capital gain. D- $500 Long-term capital gain.

Solution: The correct answer is B. Inherited stock is long-term, regardless of the actual holding period. Her gain is $250 ($6,000 sales price less $250 commission less $5,500 basis).

Isaac is a middle school teacher with gross income this year of $35,000. Based on the following, what is Isaac's adjusted gross income? $4,000 qualified education interest expense $2,000 alimony received under a pre-2018 agreement $1,000 contribution to a traditional IRA A- $30,000 B- $31,500 C- $32,000 D- $33,500

Solution: The correct answer is B. Isaac's adjusted gross income is his total gross income of $35,000 (provided in the question) - $2,500 in qualified education interest expense (the deductible amount is limited to $2,500) - $1,000 contribution to a traditional IRA = $31,500. The alimony is RECEIVED, which is included in determining his gross income of $35,000 (provided in the question). If it was paid then there would be a deduction but that's not the case. NOTE: the question gave you his GROSS income, not just his salary. Understanding the terminology used in Income Tax and the Tax formula is critical for the CFP Board Exam.

Aurora had the following cash inflows during the current taxable year: Wages: $45,000 Loan Proceeds: $2,500 Child Support: $5,000 Stock Sale Proceeds: $3,000 U.S. Government Bond Interest: $1,000 What is her gross income for income tax purposes if her adjusted tax basis in the stock was $2,000? A- $45,000 B- $47,000 C- $49,000 D- $51,000

Solution: The correct answer is B. It is import to know the tax formula flow for this exam. All income, except for what is specifically excluded, is included in gross income. Items specifically excluded from gross income are items such as; Interest income from municipal bonds, child support payments received, alimony received from agreements after 2018, etc. Income that is taxed at the capital gain rate versus the ordinary income rate will be sorted out once taxable income is calculated. Wages plus stock sale proceeds less basis in the stock plus U.S. government bond interest = $45,000 + $3,000 - $2,000 (basis in the stock) + $1,000 = $47,000. Loan proceeds (what is paid out when you take out a loan) are not taxable income and neither is child support.

James received an academic scholarship to State University. He is a candidate for a degree. Under the scholarship agreement he received: Tuition ($1,500) Books ($400) Room and board ($5,000) What is James' gross income, if any, from the scholarship? A- None B- $5,000 C- $5,400 D- $6,900

Solution: The correct answer is B. James' gross income is $5,000. Tuition and books are not included in income, but living expenses such as room and board are included in taxable income.

Joel paid $25,000 for a 30 percent interest in a general partnership. The partnership loss for the year is $180,000. How much can Joel deduct? A- $0. B- $25,000. C- $54,000. D- $60,000.

Solution: The correct answer is B. Joel is a general partner which signifies he is a material participant. Joel's loss is limited to $25,000 due to the at-risk rules. He will also have a suspended loss of [($180,000 × 30%) - $25,000] = $29,000. The passive activity rules for income do not apply.

Kenny would like to make a deductible contribution to a Health Savings Account. Which of the following is/are a requirement in order for Kenny to be able to make such a contribution? I-Kenny must be eligible to establish a Health Savings Account. II- Kenny must have a high deductible health plan. III- Kenny must meet the deductible of his HDHP. A- I only. B- I and II only. C- II and III only. D- I and III only.

Solution: The correct answer is B. Kenny is not required to meet the deductible of his high deductible health plan in order to make a deductible contribution to his HSA. However, he is required to be eligible to establish an HSA and to have a high deductible health plan

Kenny would like to make a deductible contribution to a Health Savings Account. Which of the following is/are a requirement in order for Kenny to be able to make such a contribution? I: Kenny must be eligible to establish a Health Savings Account. II: Kenny must have a high deductible health plan. III: Kenny must meet the deductible of his HDHP. A: 1 only B: 1 and 2 C: 2 and 3 D: 1 and 3

Solution: The correct answer is B. Kenny is not required to meet the deductible of his high deductible health plan in order to make a deductible contribution to his HSA. However, he is required to be eligible to establish an HSA and to have a high deductible health plan.

Mona and Niles recently moved across the country so that Niles could take a new job. Mona and Niles incurred the following expenses during their move. $10,000 paid to the moving company. $400 in gas to drive across the country to the new home. $100 in meals during the trip across country. $600 in house-hunting expenses. Niles's employer reimbursed them the cost of the moving company for $10,000. What is the tax impact to Niles and Mona? A- They may deduct all moving expenses if they are offset by the employer's reimbursement. B- The employer will include $10,000 in Niles's W-2. C- They may deduct the additional $1,000 in qualified moving costs above the line. D- No impact.

Solution: The correct answer is B. Moving expenses are no longer deductible and when paid by the employer, will be includible in the employee's W-2.

Which of the following is a true statement regarding a current Net Operating Loss (NOL)? A- A NOL generally can be carried back two years prior to the loss year. B- A NOL can only offset up to 80% of the current year's income. C- A NOL deduction is available the year the loss occurs. D- An affirmative election must be made to forgo the carryback period.

Solution: The correct answer is B. NOL losses currently cannot be carried back but they can be carried forward, (except for select agricultural or insurance filers). However, the NOL can only offset 80% of the current year's income for years after 12/31/17. From irs.gov: Most taxpayers no longer have the option to carryback a net operating loss (NOL). For most taxpayers, NOLs arising in tax years ending after 2020 can only be carried forward. The 2-year carryback rule in effect before 2018, generally, does not apply to NOLs arising in tax years ending after December 31, 2017. The CARES Act provided for a special 5-year carryback for taxable years beginning in 2018, 2019 and 2020. Exceptions apply to certain farming losses and NOLs of insurance companies other than a life insurance company. Also, for losses arising in taxable years beginning after Dec. 31, 2020, the net operating loss deduction is limited to 80% of the excess (if any) of taxable income (determined without regard to the deduction, QBID, and Section 250 deduction over the total NOLD from NOLs arising in taxable years beginning before January 1, 2018.

Nancy and Oliver had been married for 25 years when Oliver died suddenly in February of the current year. Although Nancy was deeply depressed about Oliver's death, she knew that Oliver would want her to move on with her life and she began dating again. It wasn't long before Nancy was swept off of her feet by Paul. After a romantic weekend in the Catskills, Paul and Nancy got married in November of the current year. What filing status will be used for Nancy and Oliver for the current year? A- Nancy and Oliver must both use the single filing status. B- Nancy will use the married filing jointly status, and Oliver will use the married filing separately status. C- Nancy and Oliver will both use the married filing jointly status. D- Nancy will use the surviving spouse filing status, and Oliver will use the married filing jointly status.

Solution: The correct answer is B. Nancy cannot file MFJ on two returns. She is married to Paul currently and will file MFJ with him. Oliver's final tax return will be filed as MFS. Option "A" is incorrect because Nancy is eligible to use the married filing jointly status with Paul. Option "C" is incorrect; Oliver cannot use the married filing jointly status because Nancy was married to someone else at the end of the year. Option "D" is incorrect because Nancy is not eligible to use the surviving spouse filing status. Therefore, Option "B" is the best choice.

Olive's daughter Polly suffers from a rare illness and is currently undergoing a 6 hour treatment on a fairly regular basis. During the current year, Olive drove Polly to see a specialist in another state 15 times, while her husband remained home running their software company. Each trip was 300 miles each way and required an overnight stay in a hotel that costs $140 per night. They used the hotel the night before the treatment and drove home after the treatment the next day. The unreimbursed treatments totaled $10,000. Olive's joint AGI is $124,000. How much could they deduct in relation to Polly's medical expenses for 2023, assuming they itemize (mileage rates for medical are .22 per mile)? A- $9,300 B- $4,180 C- $700 D- $0

Solution: The correct answer is B. Olive may deduct $0.22 (2023) cents per mile for the travel associated with Polly's medical care and may deduct up to $50 per night per person for lodging. The trip mileage is stated as 300 miles each way, you will need to multiply by 2 and use round trip mileage.Therefore, the total medical expenses are $13,480 which is calculated as follows: [(300 miles x 2 x 15 trips x $0.22 = 1,980) + (15 x ($50 x 2) = 1,500) + $10,000]. However, Olive may only deduct the amount that exceeds 7.5% of their AGI (TCTDR Act of 2020). 7.5% of Olive's AGI is $9,300. Therefore, medical expenses deductions for Polly's treatment is $4,180. Note: $100 for lodging is allowed because the rule is $50 per person per night.

Olive's daughter Polly suffers from a rare illness. During the current year, Olive drove Polly to see a specialist in another state 15 times. Each trip was 300 miles each way and required an overnight stay in a hotel that costs $70 per night. Olive's AGI is $24,000. What is her medical expense deduction for the current year (assume the mileage rate is 22¢ per mile)? A: $0 B: $1,230 C: $1,800 D: $3,030

Solution: The correct answer is B. Olive may deduct 22¢ per mile (2022)** for the travel associated with Polly's medical care and may deduct up to $70 per night for lodging (limited to the lesser of $50 per eligible person or actual expense incurred). Therefore, the total medical expenses are $3,030 [(300 × 2 × 15 × $.22) + (15 × $70)]. However, Olive may only deduct the amount that exceeds 7.5% of her AGI in 2022. 7.5% of Olive's AGI is $1,800, making her deductible amount (3,030 - 1,800) $1,230. Taxpayer Certainty and Disaster Tax Relief Act of 2020 extends 7.5% of AGI to taxable years beginning after December 31, 2020. ** Effective 7/1/22 - 12/31/22, the medical mileage rate has been increased to $0.22 while 1/1/22 - 6/30/22 remains at $0.18. Choice A is incorrect. Medical mileage and hotel expenses can be deducted. Choice C is incorrect. $1,800 is Olive's hurdle for medical expenses. $24,000 x 7.5% = $1,800 Choice D is incorrect. Total medical expenses of $3,030 [(300 × 2 × 15 × $.22) + (15 × $70)] have to be reduced by 7.5% of AGI, resulting in the deductible medical expenses.

Sara's daughter Tara completed her senior year of college in the current year. Sara paid $5,000 in qualified education expenses for Tara in the current year. Sara is a MFJ taxpayer and has an AGI of $50,000 for the current year. What, if any, education credit will provide Sara the highest credit and how much is that credit? A- Sara is not eligible to claim an education credit. B- American Opportunity Tax Credit in the amount of $2,500. C- Lifetime Learning Credit in the amount of $1,000. D- Lifetime Learning Credit in the amount of $2,000.

Solution: The correct answer is B. Option "B" is correct because the American Opportunity Tax Credit will provide a credit of $2,500 = 100% of the first $2,000 of expenses and 25% of the next $2,000 of expenses. Option "C" is incorrect, because although the LLC may provide Sara a credit equal to 20% of her qualified education expenses, or $1,000, she would choose the American Opportunity tax credit - she cannot claim both. Option "D" is incorrect because Sara would need to have paid $10,000 in qualified education expenses in order to claim the maximum Lifetime Learning Credit of $2,000.

Which of the following is not an exception to the passive activity rules for rental activities? A- If the average period of customer use is seven days or less, the activity could be considered an active trade or business. B- If the average period of customer use is 30 days or less but the taxpayer does not provide significant personal services in concert with the rental activity, the activity may be classified as an active trade or business. C- The activity will be considered an active trade or business if the rental of property is incidental to the non-rental activity of the taxpayer. D- A rental activity that the taxpayer customarily makes available during business hours for nonexclusive use by customers will be classified as the active conduct of a trade or business, provided the taxpayer materially participates.

Solution: The correct answer is B. Option "B" is correct because the taxpayer must provide significant personal services in concert with the rental activity and must materially participate in the activity in order to classify the activity as an active trade or business.

Alexander Dumas has a salary of $80,000, dividends of $20,000 and limited partnership income of $15,000. This year, he also invested in an equipment-leasing partnership where he is not a material participant. His initial investment included $50,000 cash and a non-recourse note for $60,000. What is the maximum tax deduction Alexander could take if there were a loss on the equipment leasing investment this year? A- $0 B- $15,000 C- $35,000 D- $50,000

Solution: The correct answer is B. Read what the question is asking carefully. What is the MAXIMUM tax deduction Alexander could take IF there were a loss on the equipment leasing investment this year? This is a passive activity. Any deduction would be equal to his limited partnership income of $15,000 from other sources to the extent he is at risk. The maximum investment deduction may not exceed the cumulative investment income. The $60,000 non-recourse note is irrelevant to answering this question. This question also assumes that the investment occurred in the current tax year.

On the evening of its silent auction, Vokel Museum has a roomful of tables with various auction items. The items are numbered and the bid list is placed beside each item. A card beside each item on the tables states the estimated fair market value of the item as determined by the auction committee, generally based on representations made by the item donor. Rebecca was the successful bidder on a football autographed by a famous quarterback. The contributor of the item was a local sports memorabilia dealer who was an expert in valuing items of this nature. He represented to the Museum that the item had a fair market value of $500. Rebecca bid and paid $800 for the football. A similar football without the autograph can be purchased at the local sporting goods store for $100. Assuming she itemizes her deductions, and ignoring any adjusted gross income limits, Rebecca will be entitled to a charitable contribution deduction in the amount of: A- $100. B- $300. C- $500. D- $800.

Solution: The correct answer is B. Rebecca is entitled to a charitable contribution deduction in the amount of $300 because she paid more for the item than the value she received in return. The excess payment is a gift and a deduction is allowed but only to the extent of that excess.

Nathan sustained serious facial damage in a snow skiing accident. To restore his physical appearance, he had restorative surgery. At the same time, he also had surgery to remove wrinkles from under his eyes caused by age. Which of these procedures can Nathan claim as a deductibe medical expense? A- Both of the procedures can be claimed as deductible medical expenses because he had them done at the same time. B- Only the restorative surgery can be claimed as medical expenses. C- Only the wrinkle removal surgery can be claimed as medical expenses. D- None of these can be claimed as medical expenses because they are both cosmetic.

Solution: The correct answer is B. Reconstructive surgery to correct disfigurement is a deductible medical expense subject to 7.5% AGI*. Elective cosmetic surgery is not deductible. *Taxpayer Certainty and Disaster Tax Relief Act of 2020 set the medical expense deduction floor at 7.5% permanently.

Which of the following statements about casualty losses are correct? A- Casualty losses can be deducted if they are personal. B- Insurance reimbursements of casualty losses can reduce the loss amount to the taxpayer. C- Casualty losses can be reimbursed but are subject to inclusion in the taxpayers AGI. D- Casualty losses are reduced by $500 per event if the asset loss was held for personal use.

Solution: The correct answer is B. Reimbursement of any loss the taxpayer sustained would limit their overall loss the taxpayer must cover with out of pocket funds. Personal casualty losses are not deductible, unless the loss was as a result of a federally declared disaster. Casualty loss reimbursements are not included in AGI but rather reduce the loss. Casualty losses that could be deducted (federally declared disasters) would be subject to a $100 floor on the taxpayer's income tax return

During the current year, JoAnne's business made net income before any Section 179 deduction of $15,000. She added an allowable Section 179 asset to her business valued at $150,000 as of the first of the current year. What can she deduct under Section 179 in the current year? A- $25,000 B- $15,000 C- $135,000 D- $150,000

Solution: The correct answer is B. Section 179 deduction cannot exceed net business income in a given year. The rest of the Section 179 deduction can be carried over, but a 179 deduction cannot be used to create a business loss. Therefore, they would only be allowed to deduct $15,000 for the current year and carry forward $135,000.

Reilly owns and operates an accounting practice as a sole proprietorship. For tax reporting, Reilly uses the accrual method of accounting. Last year, Reilly prepared a tax return for a client and billed the client $600. The client did not pay and has recently disappeared. The reminder notices that Reilly's secretary had sent to the client have been returned with no forwarding address. How should this bad debt be treated for income tax purposes? A- No bad debt deduction is permitted. B- Reilly may deduct $600 from his business income. C- Reilly may deduct $600 as a short-term capital loss. D- Reilly may deduct $600 as a long-term capital loss.

Solution: The correct answer is B. Since Reilly uses the accrual method of accounting, he reported and paid tax on the $600 of income last year when he performed all of the services necessary to collect the income. It is clear that Reilly will not be able to collect the debt, so he can use the specific charge off method and deduct $600 from his business income this year.

Scary Berries, Inc. is a C corporation that specializes in carving berries into frightening images. The company's primary profit generation is the sale of berries and they generate $4 million in annual revenue on average. What accounting method may Scary Berries, Inc. utilize for tax purposes? A- Cash basis B- Either cash or accrual C- Accrual basis D- Units of production

Solution: The correct answer is B. Since Scary Berries is a qualifying small business taxpayer with average gross receipts below the IRS limit for small businesses (typically around $27 million), they can choose to use the cash basis or the accrual basis accounting method. For companies with gross receipts above the the IRS small business amount (that carry inventory) they will need to use the accrual method.

On December 31 of last year, Uli purchased 100 shares of Runway, Inc. (a publicly held company) for $5,000. On March 1 of this year, Runway, Inc. declared that it was bankrupt, that it will wrap up operations, and that all of its assets will be used to satisfy secured creditor claims so there will be no residual equity left for the stockholders. Which of the following statements describes the tax treatment of this transaction this year? A- Uli may deduct the $5,000 as an ordinary loss. B- Uli may deduct the $3,000 investment against ordinary income. C- Uli may deduct the $5,000 investment as a long-term capital loss. D- No loss deduction is permitted.

Solution: The correct answer is B. Since the company became worthless during the year, a constructive sale of the stock occurs on December 31 of this year. Therefore, Uli has a short-term holding period because she is deemed to have held the stock for exactly one year (long-term holding period requires at least one year and one day. Therefore, Uli will be able to deduct $3,000 against ordinary income in the year the company becomes worthless and the remaining $2,000 investment will carry forward as a short-term capital loss .

All of the following statements concerning the AMT as it applies to individual taxpayers are correct EXCEPT: A- Some itemized deductions taken for regular tax purposes must be added back to regular income to determine income under AMT. B- Taxpayers are permitted to take the standard deduction for both regular and AMT tax purposes. C- All adjustments made to itemized deductions when calculating AMT result in a permanent increase in tax. D- Charitable deductions are claimed in the same manner for regular tax and for AMT tax purposes.

Solution: The correct answer is B. Taxpayers who do not itemize deductions take the standard deduction for regular tax purposes, but this is added back to alternative minimum taxable income for AMT purposes. All of the other statements are correct.

Matt owned stock in Whitline Corporation that he donated to a university (a qualified charitable organization) on May 1 of this year. What is the amount of Matt's charitable income tax deduction assuming that he had purchased the stock for $3,500 on March 3 of this same year, and the stock had a value of $4,600 when he made the donation? His AGI was $8,100. A- $1,100 B- $3,500 C- $4,050 D- $4,600

Solution: The correct answer is B. Taxpayers who donate short term gain property are required to use the lesser of fair market value or adjusted basis for the determination of the charitable income tax deduction. In this case, Matt only owned the stock for 2 months, therefore it will be short term. Mat will be limited to 50% of AGI: $8,100 x 50% = $4,050 The basis ($3,500) is less than the FMV ($4,600).

To which of the following entities does the term "pass-through" entity apply? I- LLC. II- C corporation. III- Not for Profit Corporation. IV- S corporation. V- Partnership. A- I, III and IV only. B- I, IV and V only. C- I, III, IV and V only. D- II and III only.

Solution: The correct answer is B. The S corporation, LLC and Partnership are considered "pass through" entities. "Pass through" means that the entity is not taxed separately from its owners, but passes its profits and losses through to the owners in their pro rata share of ownership.

George failed to pay $5,000 of income tax due with his return, which was timely filed on April 15th. He waits for 2 months after April 15th to pay the tax. How much will his penalties be? A- $25 B- $50 C- $55 D- $75

Solution: The correct answer is B. The amount for failure to pay is point five percent (0.5%) per month = $5,000 × .005 × 2 = $50 Clock starts April 15. April 15 - May 15 May 15 - June 15 Any time after June 15th and before July 15th would be a full month.

Kevin's 12 year old daughter, Angel, has a brokerage account that generates $13,000 of interest income and $2,000 of qualified dividends for the current year. Angel also has earned income of $14,000 from modeling that she is saving for college. How much will be taxed at Angel's tax rate? A- $1,250 B- $2,650 C- $12,500 D- $13,850

Solution: The correct answer is B. The calculation is as follows: Start with figuring out the standard deduction. She can use her earned income plus $400 not to exceed 13,850 in 2023. Calculate Unearned Income first: 13,000 + 2,000 = 15,000 15,000 - 1,250 (standard deduction) = 13,750 remaining 13,750 - 1,250 (at the child's rate) = 12,500 remaining The remaining 12,500 will be taxed at the parent's rate. Then Calculate Earned Income: 14,000 - 12,600 (remaining standard deduction: 13,850 - 1,250 from the standard deduction used for unearned income) = $1,400 at the child's rate. Summary: Standard deduction is 1,250 + 12,600 = 13,850 At the parent's rate 12,500. At the child's rate (1,250 + 1,400) = 2,650.

For purposes of the Social Security earnings test, which of the following types of income would potentially cause a reduction in Social Security benefits? A- Dividends received from a global mutual fund. B- Royalties from a published novel. C- Distribution from a defined benefit plan. D- Required minimum distribution received from a traditional IRA.

Solution: The correct answer is B. The earnings test only applies to earned income received by the Social Security recipient. Royalties in general are taxed at the ordinary income tax rate, but royalties that are in connection with any publication of the person's work are earned income. A is incorrect. Dividends received from investments are considered unearned income. C is incorrect. Distributions from retirement plans are considered unearned income. D is incorrect. Distributions from retirement plans are considered unearned income.

Kent earned a salary of $175,000 during 2023. Assuming the Social Security wage base was $160,200, what is the amount of FICA taxes paid by Kent (rounded to the nearest dollar)? A- $10,924. B- $12,470 C- $13,388. D- $26,775.

Solution: The correct answer is B. The first $160,200 (for 2023) of Kent's salary will be taxed at the partial employee FICA rate of 6.2%. All earnings will be taxed at the remaining FICA Medicare rate of 1.45%. The total FICA due is: $160,200 x 6.2% = $9,932.40 $175,000 x 1.45% = $2,537.50 Total FICA = $12,469.90

One of the five tests which must be met to qualify as a dependent is: A- The age of the dependent. B- The dependent is either a member of the taxpayers household or meets the criteria for family relationship. C- The taxpayer is a U.S. citizen. D- All of the above.

Solution: The correct answer is B. The five dependency tests are: 1) Gross Income Test, 2) Support Test, 3) Member of Household or Family Member Test, 4) Citizenship Test (U.S., Canada or Mexico), and 5) Joint Filing Test.

One of the five tests which must be met to qualify as a dependent is: A- The dependent has no income. B- The taxpayer contributed over 50% of the dependent's support. C- The dependent must be under 23 years old. D- The dependent must not be required to file an income tax return.

Solution: The correct answer is B. The five dependency tests are: 1) Gross Income Test, 2) Support Test, 3) Member of Household or Family Member Test, 4) Citizenship Test (U.S., Canada or Mexico), and 5) Joint Filing Test.

What classifications of property are subject to cost recovery? I- Personalty. II- Personal use property. III- Natural resources. IV- Intangible property. V- Real estate including land. A- II and IV only. B- I, III and IV only. C- V only. D- All of the above.

Solution: The correct answer is B. The following refers to what is not covered (and why), the answer reflects what is covered. Personal use property is not subject to cost recovery since it is not used for income generating business purposes. Natural resources are subject to depletion and intangibles are subject to amortization. Personalty (tax term for personal property) assets are used in business and are subject to depreciation. Real estate, as in permanent structures on land, are subject to cost recovery, but the land is not.

Giselle became an AMT taxpayer last year. She had to add several items to her regular taxable income in arriving at alternative minimum taxable income. Which of the following items will result in an AMT credit that can be used to offset future regular tax liability? A- $5,000 in property taxes paid on her principal residence if paid in advance. B- A $75,000 difference between the fair market value of stock and the strike price in the incentive stock option used to purchase the stock. C- $4,000 in interest on private activity municipal bonds. D- $4,000 in additional medical expenses.

Solution: The correct answer is B. The inclusion of the difference between the fair market value and exercise price of the stock options will result in a credit that Giselle can use against future regular income tax liability. The other items are adjustments made to her itemized deductions, which result in permanent differences in tax liability as a result of the imposition of the AMT.

Bill and Renee are married and in community property but living apart and filing separate federal income tax returns. Each earned a salary of $25,000 and Renee received $5,000 in interest on money she inherited from her deceased mother after her marriage to Bill. Which of the following is correct? A- In some states, Bill's gross income is $55,000. B- In some states, Renee's gross income is $27,500. C- In all community property states, Bill's gross income is $25,000. D- In all community property states, Renee's gross income is $30,000.

Solution: The correct answer is B. The inherited property may be considered separate property, and therefore, in some community property states the income earned from it is separate. In others, it is community. Some community property states are quasi community property. That means the $5,000 income from the separate property would be joint income, making Renee's income her salary and half the property income of $2,500. If asked a general question about community property states, it is best to stick to some states. The exam typically states which state the client lives in. They most often use California and Texas, which are true community property states. Arizona is one of the quasi community property states. Meaning transferring in individual assets would mean keeping them individual instead of making them automatically community property.

Last year, Horace paid the following interest: Interest on his home mortgage = $6,200; Interest on a loan to purchase household furniture for his personal residence = $700; Interest on a loan to purchase State of Nebraska general purpose bonds = $1,400. If Horace itemizes his deductions for last year, the amount of deductible interest expense is: A- $6,900 B- $6,200 C- $7,600 D- $8,300

Solution: The correct answer is B. The interest on the loan to purchase household furniture is non-deductible consumer interest. The interest on the loan to purchase State of Nebraska bonds is not deductible under Section 265 because he used the loan to purchase tax exempt municipal bonds.

Ron sells two personal use assets during the taxable year. A gain of $3,000 is realized on the sale of one asset and a loss of $9,000 is realized on the sale of another asset. What is his gain or loss for the year? A- ($3,000) loss B- $3,000 gain C- ($6,000) loss of which only $3,000 is deductible this year. D- ($9,000) loss of which only $3,000 is deductible, the $6,000 is carried over indefinately.

Solution: The correct answer is B. The loss on sale of a personal asset (one not used in business) is not deductible. The sale of a personal asset at a gain is included in income. Therefore, the $9,000 loss does not offset the $3,000 gain.

During the current year, Harrison sustained a serious injury in the course of his employment. As a result of the injury sustained, he received the following payments during the year: Unemployment compensation $5,000 Worker's compensation $6,500 Damages for physical personal injuries $5,000 Reimbursement from his employer's accident and health plan for medical expenses paid by Harrison $2,000 What is the amount to be included in Harrison's gross income for the current year? A: $2,000 B: $5,000 C: $6,500 D: $11,500

Solution: The correct answer is B. The only item included in Harrison's gross income is the unemployment compensation. All other payments received are specifically excluded from gross income.

Frank is considering selling a parcel of raw land located in South Dakota that he owns. If Frank sells the land, he would like to invest the proceeds in another piece of real property and would like to qualify for like-kind exchange treatment. Which of the following assets would not qualify as like-kind property for the sale of raw land? A- Raw land located in Florida. B- Raw land located in Canada. C- New land located in South Dakota. D- An industrial warehouse located in California.

Solution: The correct answer is B. The question is looking for which one would NOT qualify as like kind exchange. U.S. real estate and foreign real estate are not like-kind assets for income tax purposes. Therefore, if Frank exchanges his raw land in South Dakota for raw land in Canada, he will not qualify for like-kind exchange treatment.

Veronica borrowed $300,000 to acquire a parcel of land to be held for investment purposes. During the year, she paid interest of $30,000 on the loan. She had AGI of $70,000 for the year. Other items related to Veronica's investments include the following: Investment income = $15,200 Long-term gain on the sale of stock = $6,000 Investment counsel fees = $900 Veronica is unmarried and elected to itemize deductions. She had no miscellaneous deductions other than the investment counsel fees. Determine Veronica's maximum investment interest deduction. A- $30,000 B- $21,200 C- $16,100 D- $15,200

Solution: The correct answer is B. The taxpayer's investment interest deduction is limited to the investment income. The investment income is $15,200 plus she can add the capital gains of $6,000 and deduct $21,200. The excess investment interest ($30,000 - $21,200) can be carried over to next year. Note: In order to treat the LTCG as ordinary income to allow for a greater deduction, it needs to state the special election was made or they would like to maximize the amount they can deduct.

Chris is a CPA and recently donated a rare coin valued at $10,000 to his church. He acquired the coin at a flea market several years ago for $20. In addition, he performed six hours of bookkeeping work for the church at no charge. He normally bills $150 per hour. Ignoring AGI limits, what is the amount that Chris may deduct as a charitable contribution? A- $0. B- $20. C- $920. D- $10,000

Solution: The correct answer is B. The value of services donated to charity is nondeductible. He will only be entitled to a $20 deduction for the coin, because the deduction is limited to basis for a use-unrelated asset.

Cecilia and Landon DeFee purchased their primary residence in North Carolina this year, but still maintain a vacation property in the Smokey Mountains. Their home in N. Carolina was purchased for $450,000, and they paid $22,349 in interest the first year. The vacation property was purchased 5 years ago for $300,000 and they paid $14,795 in interest this year. The couple has no other itemized deductions. What is the mortgage interest deduction for Cecilia and Landon? A: $10,000 B: $37,144 C: $25,100 D: $22,349

Solution: The correct answer is B. Their itemized deductions exceed the standard deduction of $25,900 for 2022. Their housing acquisition costs are under the $750,000 cap from TCJA of 2017, and no additional equity loans or credit has been taken. They can deduct their full mortgage interest. SALT (State and Local Taxes) is limited to $10,000

Which of the following are the types of credits available to individual taxpayers? I- Refundable. II- Deductible. III- Non-refundable. IV- Partially refundable. V- Exempted. A- I and II only. B- I and III only. C- II, IV and V only. D- I, III and IV only.

Solution: The correct answer is B. There are many non-refundable credits including, but not limited to, child care. EIC is the only fully refundable credit. The IRS recognizes two categories of credits; refundable and non-refundable. The term refundable does not necessarily imply that it is fully refundable, it may be fully or partially refundable. AOTC and child tax credit are partially refundable, but the IRS will consider them in the refundable category.

Depreciation is allowed for which of the following business assets: I-Land II-Tangible personalty III-Stock in trade IV- Improvements to land A- IV only. B- II and IV only. C- I, II and III only. D- I, II, III and IV.

Solution: The correct answer is B. There is no depreciation allowed for inventories, stock in trade or land apart from its improvements. Tangible personalty (tax term for personal property) used in a trade or business is section 1245 property and is depreciable.

Homer and Marge have been unable to have a baby. They decided last year that adoption would be the best choice for them. They adopted, Maggie, a 4 year old child this year. They paid $15,000 in qualifying adoption expense for the current year. Their MAGI is $170,000 and their tax due before the application of the qualified adoption credit is $11,000. What is Homer and Marge's available adoption credit for the current year? A- $9,000. B- $11,000. C- $13,570. D- $15,000.

Solution: The correct answer is B. They are limited to the least of a) qualifying adoption expenses (15,000), b) adoption credit of $15,950 for 2023, or c) amount of tax due ($11,000). The adoption credit is not refundable.

Keith, age 12, has $10,000 in unearned income and $20,000 in earned income in 2022. How much will be taxed at the parent's rate? A: $0 B: $7,700 C: $9,350 D: $17,050

Solution: The correct answer is B. Unearned $10,000 Earned Income $20,000 Total $30,000 Less SD (2022) ($12,950) $17,050 At parent Rate $7,700 $10,000 unearned income - $2,300* At kid's rate $9,350 SECURE Act 2019 reverted the TCJA 2017 back to the pre-TJCA calculation. Amounts above the unearned income standard deduction will be taxed at the parent's tax rate (no longer at the trust and estate rate). * 2022 standard deduction as applies to unearned income of $1,150 and the amount of $1,150 taxed at the child's rate

Rob was given a residence in the current year. At the time of the gift, the residence had a fair market value of $200,000, and its adjusted basis to the donor was $140,000. The donor paid a tax of $10,000 on the taxable gift of $185,000. What is Rob's basis for gain? A: $140,000 B: $143,000 C: $150,000 D: $200,000

Solution: The correct answer is B. The formula for gift tax attributable to appreciated property is: FMV - Donor's basis ------------------------ × gift tax paid value of taxable gift 200,000 - 140,000 200,0000 × 10,000 = $3,000 (140,000 [donor's basis]+3,000=143,000)

Hannah owns an event planning company that specializes in very high-end events. Several years ago, Hannah purchased a magnificent chocolate fountain for $3,000 and has since taken $1,200 in depreciation deductions on the fountain. Hannah is now ready to replace the fountain with tools for creating ice sculptures, but she is not sure what the tax consequences of selling the fountain will be. Which of the following statements is true regarding the tax consequences of selling the fountain? A- If Hannah sells the chocolate fountain for $1,800, she will have a $1,200 ordinary loss. B- If Hannah sells the chocolate fountain for $1,700, she will have a $100 capital loss. C- If Hannah sells the chocolate fountain for $2,000, she will have an ordinary gain of $200 and no capital gain. D- If Hannah sells the chocolate fountain for $3,300, she will have a $1,500 capital gain.

Solution: The correct answer is C.

Mel made the following contributions to charity during the past year: Used clothing of the taxpayer and family - Basis = $900 and FMV = $300 Stock in GMC held as an investment for 13 months - Basis = $8,000 and FMV = $7,000 Stock in United Corp. held as an investment for 9 months - Basis = $9,000 and FMV = $10,000 Real estate held as an investment for six years - Basis = $10,000 and FMV = $25,000 The used clothing was donated to the Salvation Army; the other items of property were donated to a Methodist seminary. Disregarding any percentage limitations, Mel's charitable contribution deduction is: A- $43,300 B- $42,900 C- $41,300 D- $26,300

Solution: The correct answer is C. A donation of short-term capital assets is recognized as a charitable expense at the lower of the FMV or basis. Donations of appreciated long-term assets are recognized as a charitable expense at the fair market value unless basis is chosen. In this question, the United Corp stock is a short-term investment that was held for less than 12 months, therefore, it will be expended as a charitable contribution at the donor's basis of $9,000. The remainder of the donated assets are expensed for charitable purposes at their fair market value. In summary: Used clothing of the taxpayer and family - Basis = $900 and FMV = $300. A Taxpayer cannot take basis on loss property. Stock in GMC held as an investment for 13 months - Basis = $8,000 and FMV = $7,000. A Taxpayer cannot take basis on loss property (default valuation is FMV unless Basis is elected for non-loss property) Stock in United Corp. held as an investment for 9 months - Basis = $9,000 and FMV = $10,000. Short term property is lower of basis or FMV. Real estate held as an investment for six years - Basis = $10,000 and FMV = $25,000. Real Property uses FMV.

Abigail was an original investor in, and owns a 25% interest in, Decorate Your Dream LLC, a home decorating company. Abigail paid $50,000 for her interest. This year, Decorate Your Dream did very well and had a profit of $100,000. However, no distributions were made. What is Abigail's basis in her interest in Decorate Your Dream at the end of this year? A- $25,000. B- $50,000. C- $75,000. D- $100,000.

Solution: The correct answer is C. Abigail's original cost basis is increased by her share of the profits. Therefore, her original cost basis of $50,000 is increased by $25,000. If there had been a distribution, this would have reduced Abigail's adjusted basis in her interest in the company.

Craig and Mallory got married and bought a house 15 months ago. Mallory's job recently transferred her to an office in a different state, so Craig and Mallory sold their house. What is the maximum amount of gain from the sale of the personal residence that Craig and Mallory can exclude from income taxation? A- $0. B- $250,000. C- $312,500. D- $500,000.

Solution: The correct answer is C. Although they did not live in their house for a full two years, Craig and Mallory are eligible for a prorated exclusion because of Mallory's change in employment. Therefore, they are eligible for a maximum exclusion of [(15 / 24) × $500,000]=$312,500.

Ralph is not married and does not have any children. However, Ralph is a very good son and provides more than half of the cost of maintaining a very nice apartment and the necessary expenses for his mother. Which of the following filing statuses should Ralph use and why? A- Single, because Ralph is not married. B- Single, because Ralph does not have any qualifying children. C- Head of Household, because Ralph's mother qualifies as a dependent. D- Head of Household, because Ralph's mother is a qualifying child.

Solution: The correct answer is C. Because Ralph provides more than half of the cost of maintaining his mother's apartment and necessary expenses, he is eligible to use the Head of Household filing status if he qualifies his mother as a dependent. Options "A" and "B" are incorrect; since Ralph is eligible to use the Head of Household filing status, he should use that filing status rather than the less advantageous single filing status. Option "D" is incorrect because Ralph's mother is a qualifying relative, not a qualifying child. Note: Don't confuse qualifying child with qualifying relative. Qualifying relative test is relationship, gross income, support and not a qualifying child test. Living in the household is not a requirement. Many adult children pay the living expenses of their elderly parents; could be a retirement home, assisted living facility or nursing home.

Vince, a single individual, is one of the founders and original shareholders of Security Consulting, Inc., a corporate security consulting firm. The company was initially capitalized with $200,000, and Vince was a 50 percent owner. The company was structured as a C corporation and filing requirements and permissible tax elections that could benefit the owners were made at the time the company was created. After several years of successful operations, Security Consulting lost market share to large national firms, and eventually closed down operations. Since it had no assets other than the goodwill of the business, there was nothing left to distribute to the shareholders. Assuming that there were no changes to Vince's ownership interest over the period of his ownership, and that Vince has no capital transactions in the current year, by how much can Vince reduce his adjusted gross income this year due to the company becoming worthless? A- $3,000. B- $50,000. C- $53,000. D- $100,000.

Solution: The correct answer is C. Because it was capitalized with less than $1 million and Vince was an original shareholder, the stock is Section 1244 stock in Vince's hands. Vince can deduct up to $50,000 of losses as an ordinary loss in any one tax year and the remaining loss is treated as a capital loss. Therefore, Vince will be able to deduct $50,000 of his loss as a Section 1244 loss against ordinary income and will qualify for an additional $3,000 long-term capital loss deduction. The remaining capital loss of $47,000 will be carried forward to future tax years.

Taxable income is the amount: A- Remaining after adjustments to income are subtracted. B- From which allowable itemized deductions are subtracted. C- Used to determine tax liability. D- To which tax credits are applied.

Solution: The correct answer is C. By definition the liability is computed by using the tax tables or tax rate schedules on the amount of taxable income. Statement A is false. This describes AGI. Statement B is false. This is also AGI. Statement D is false. This is the tax liability. Remember the tax formula: Gross income - Above the line deductions = AGI - Below the line deductions (standard deduction or itemized) - Personal and dependency exemptions (after TCJA expires) = Taxable Income Calculate tax based on filing status (tax liability) - Credits + Other taxes - Prepayments = refund or additional tax due

Charlie purchased a 25% interest in a general partnership for $40,000. He is a material participant in the partnership's activities. In the current year, the partnership borrowed $20,000 from a local bank. The loan is considered a recourse loan. Assuming the partnership incurred a total loss for the current year of $200,000, how much of the loss will Charlie be permitted to deduct on his federal income tax return? A- $0. B- $40,000. C- $45,000. D- $50,000.

Solution: The correct answer is C. Charlie's share of the loss is $50,000 ($200,000 total loss x 25% ownership interest). However, his deductible loss is limited to the amount Charlie has "at risk" because he is a general partner (material participant). Charlie's "at risk" amount is equal to the amount he contributed to the partnership, plus his share of recourse loans incurred by the partnership. Recourse loans add to basis since the taxpayer is responsible for the loan, which is the opposite of a non-recourse loan. Charlie's share of recourse loans is $5,000 ($20,000 total loan x 25%). Therefore, the total amount at risk for Charlie is $45,000 ($40,000 contributed + $5,000 recourse loan). He can take the loss without having to offset income because this is a general partnership, not a limited partnership.

Fiona is a highly compensated employee of GreatWorks, Inc. Which of the following fringe benefits would be taxable to Fiona? A: Health insurance provided by GreatWorks to all employees. B: Group term life insurance in the amount of $40,000 paid for by GreatWorks. C: Dependent care assistance for the highly compensated employees of GreatWorks. D: On-premises athletic facilities that may only be used by the managers and vice-presidents of GreatWorks.

Solution: The correct answer is C. Dependent care assistance can only be excluded from a highly compensated employee's gross income if it is provided on a nondiscriminatory basis. Answer d is not correct because access to athletic facilities can be provided on a discriminatory basis without causing inclusion in the employee's gross income.

Which of the following creates potential Section 1245 recapture? A- Amortization of goodwill. B- Immediate expense deduction under Section 179. C- The sale of tangible personalty used in a trade or business at a gain. D- There is Section 1231 loss from the property's disposition.

Solution: The correct answer is C. Depreciable property includes equipment, patents, copyrights and other intangibles. Option "A" is incorrect because goodwill is amortized. Option "B" is incorrect because Section 1245 is not applied to Section 179 expensing. Option "D" is incorrect because a Section 1231 is applied to gains recognized after recapture. Note: The term personalty refers to personal (non-real estate) property. Instructor note: section 1245 is tangible personal property and thus section 1245 recapture is on depreciation of personal property and not on amortization of goodwill.1250 is real property

Julian purchased 100 shares of Home Depot, a domestic corporation, common stock on July 7th this year. The ex-dividend date for their quarterly dividend is July 12th. Julian sells the Home Depot stock on August 15th of this year. If Home Depot paid a dividend of $10 on Julian's 100 shares, what are the tax consequences to Julian if Julian is in the 24% tax bracket? A- No tax liability since the qualified dividend rate is 0% for individuals in the 24% bracket. B- $1.50 of tax liability since the qualified dividend rate is 15% for individuals in the 24% bracket. C- $2.40 of tax liability since the ordinary dividend rate is the taxpayer's marginal rate. D- Not enough information to answer the question.

Solution: The correct answer is C. Home Depot dividends will qualify for qualified dividend treatment if the individual meets the requisite holding period, which is more than 60 days in the 121 days surrounding the ex-dividend date. Since he only held the stock for 39 days, he does not meet the holding period. Therefore, the tax rate applied against the dividend is his ordinary income tax rate of 24%.

Tony Scarponi has come to you asking about the basis of property that his brother Calvin gave to him. The property had a market value of $75,000 and Calvin's adjusted basis in the property was $18,000 at the time of the gift. Calvin paid gift tax of $3,500 on the gift. Tony wants to know what his adjusted basis in the property is. Assume Calvin had utilized his annual gift tax exclusion for gifts previously given to Tony that year. What will you tell him? A- Tony's new basis is $18,000, the same as Calvin's basis was at the time the gift was made. B- Tony's new basis is the fair market value of the gift at the time of the gift. C- The adjusted basis for Tony is $20,660. D- The adjusted basis for Tony is $21,500.

Solution: The correct answer is C. Increase in Donee's Basis = (Appreciation of the Property/ Taxable Gift) × Gift Tax Paid [($57,000 ÷ $75,000 ) × $3,500] = $2,660 You then add the increase in basis to the adjusted basis: $2,660 + $18,000 = $20,660

Trenton owns a country home and sometimes rents it out to other people. Which of the following statements regarding Trenton's rental activity is correct? A- If Trenton rents out the country home for less than half of the year, it will be considered a nontaxable activity. B- If Trenton rents out the home for exactly half the year and uses the home personally for exactly half the year, the activity will be considered primarily a rental activity. C- If Trenton rents out the home for 180 days per year and uses the home personally for 20 days of the year, the activity will be considered a mixed-use activity. D- If Trenton rents out the home for more than half of the year, it will be considered primarily a rental activity regardless of his personal use.

Solution: The correct answer is C. Option "A" is not correct because the activity will only be considered a nontaxable activity if Trenton rents out the country home for less than 15 days per year. Option "B" is not correct because this option meets the requirements of a mixed-use activity. That is, the property is rented out for 15 days or more, but the owner personally uses it for the greater of 14 days or 10% of the rental days. Option "D" is not correct because even if Trenton rents out the home for more than half of the year, the activity may be a mixed-use activity if he personally uses the property for the greater of 14 days or 10% of the rental days.

Eloise is an unmarried elderly woman who lives alone in a small apartment. Eloise is only able to provide 6% of her own support. The remainder of her support is provided by the following people: 10% of her support is provided by her oldest son Frank. 22% of her support is provided by her daughter Gertrude. 30% of her support is provided by her son Henry. 32% of her support is provided by her friend Irene. Which of these individuals is eligible to claim Eloise as a dependent? A- Irene can claim Eloise as a dependent because she provides more support than anyone else. B- Frank can claim Eloise as a dependent because he is the oldest son. C- Henry can claim Eloise as a dependent, but only if Gertrude signs an appropriate statement. D- None of these individuals may claim Eloise as a dependent.

Solution: The correct answer is C. Option "A" is not correct; Irene may not claim Eloise as a dependent because she does not meet the relationship test as a qualifying relative. Option "B" is not correct; Frank has not provided more than 10% of Eloise's support, so he is not a qualifying person. Either Gertrude or Henry may claim Eloise as a dependent because they each provided more than 10% of Eloise's support and together they provided more than 50% of her support. In order for one of them to claim Eloise as a dependent, however, the other must sign a statement agreeing not to claim an exemption for Eloise for this year.

Ursula's divorce from her husband Boris became final on December 30 of the current year. Ursula's children lived with her for the first four months of the current year, but moved in with their father after Ursula was declared legally blind. Ursula did not contribute anything to the cost of maintaining the household when the children were living with her husband. Ursula is 40 years old. What filing status can Ursula use during the 2023 tax year and what is her standard deduction? A- Married Filing Jointly; $29,200. B- Head of Household; $22,650. C- Single; $15,700. D- Single; $13,850.

Solution: The correct answer is C. Option "C" is correct; Ursula must use the Single filing status. In addition, she is entitled to one additional standard deduction because of her blindness. Therefore, her standard deduction for the current year is $15,700 ($13,850 + $1,850 for 2023). While she does need to file for blindness, the question asked what she can take. Option "A" is incorrect because even though Ursula was married during the current year, she was not married as of the end of the year. Option "B" is incorrect because Ursula does not qualify for the Head of Household filing status. Ursula did not maintain a household for a qualifying child for more than half of the year. Her children only lived with her for four months of the year and she did not pay for the cost of maintaining a household for them for the remainder of the year. Option "D" is incorrect; Ursula must use the Single filing status but, she is entitled to file for one additional standard deduction because of her blindness.

Marsha has the following income and losses for the current year: I. ($1,000) loss from a 30% interest in Laminate Partnership in which she does NOT materially participate. II. ($1,500) loss from a 2% limited partnership interest in Venture, a limited partnership. III. ($3,000) loss from a 12% interest in an S corporation in which she manages one of the departments. IV. $40,000 salary as manager with an S corporation. V. $1,200 of dividend income from Higher Mutual Funds Assuming Marsha has sufficient at risk basis in each of the entities, what is Marsha's adjusted gross income? A- $35,700 B- $41,200 C- $38,200 D- $36,700

Solution: The correct answer is C. Option "I" - A loss from a limited partnership in which there is no material participation is governed under the passive activity loss rules. Since there is no other passive activity income to offset the loss, the loss is not currently deductible. Option "II" - The same passive activity loss rules apply, and therefore, the loss is not currently deductible. Option "III" - Because she is a material participant in managing the S corporation, the losses are deductible. Option "IV" - Wages are always included in AGI. Option "V" - Dividend income unless excluded is included in AGI. $40,000 (wages) minus $3,000 (S corp loss) plus $1,200 (dividends) = $38,200.

Claude and Daphne are trying to calculate their gross income for this year. Which of the following items should they include in their gross income? I: Child support payments in the amount of $15,000 received by Daphne from her ex-husband for the support of Daphne's minor child Emile. II: $1,200 of dividends received by Claude and Daphne from Mudbugs, Inc., a corporation in which they own 200 shares of stock. III: Unemployment benefits in the amount of $800 received by Claude from the state of Louisiana. IV: $3,000 that Daphne earned selling her homemade andouille sausage. A: 4 only B: 1 and 2 C: 2, 3, and 4 D: 1, 2, 3, and 4

Solution: The correct answer is C. Option 1 is not correct because child support is not includible in gross income. All of the other options are included in gross income (dividend income, unemployment compensation benefits, and gross income from self-employment).

Ursula and her husband Boris were legally separated in January 2022, and their divorce became final on December 30, 2022. Ursula's children lived with her for the first four months of 2022, but moved in with their father after Ursula was declared legally blind in April. Ursula did not contribute anything to the cost of maintaining the household when the children were living with her husband. Ursula is 40 years old. What filing status can Ursula use for her 2022 tax filing and what is her standard deduction? A; Married Filing Jointly; $27,300 B: Head of Household; $21,150 C: Single; $14,700 D: Head of Household; $21,500

Solution: The correct answer is C. Option C is correct; Ursula must use the Single filing status. In addition, she is entitled to one additional standard deduction because of her blindness. Therefore, her standard deduction for 2022 is $14,700 ($12,950 + $1,750). Option A is incorrect because even though Ursula was married during 2022, she was not married as of the end of the year. Options B and D are incorrect because Ursula does not qualify for the Head of Household filing status. Ursula did not maintain a household for a qualifying child for more than half of the year. Her children only lived with her for four months of the year and she did not pay for the cost of maintaining a household for them for the remainder of the year.

Sabina Herz, a school teacher who is in a low marginal income tax bracket and is interested in investing in an educational business with some of her colleagues. Sabina says that she wants to keep the money in the business and doesn't want to pay taxes on income she never sees. Furthermore, Sabina wants to be assured the business would NOT be disrupted if one of her partners lost interest or encountered personal financial problems. What legal form of business makes the most sense, given Sabina's desires? A- A Limited Partnership. B- A General Partnership. C- A C Corporation. D- A Professional Corporation.

Solution: The correct answer is C. Options "A" and "B" are incorrect in that they would dissolve in the event of a 50% turnover in ownership in a 12-month period. Option "C" is correct because a C corporation generally retains income that is not passed to its owners through dividends. Option "D" is incorrect because it is structured as a C corporation for professional services. Not an educational business.

Which of the following statements correctly describes the cash receipts and disbursements method of accounting? A- Income is reported as it is earned and expenses are reported as they are incurred. B- The cash receipts method is the most difficult accounting method to understand. C- A taxpayer who uses the cash method for reporting most items may use a different method for reporting self-employment income. D- Reporting of income and expenses is subject to the all events test.

Solution: The correct answer is C. Options "A" and "D" describe the accrual method of accounting, not the cash receipts and disbursements method of accounting. Option "B" is incorrect; the cash receipts method is generally the easiest method of accounting to understand and the simplest to use.

Which of the following are characteristics of a C corporation? I- The number of shareholders is unlimited. II- The bankruptcy of a major shareholder has NO effect on the business form. III- Shareholder liability is limited. IV- The capital structure is dependent upon the resources of the shareholders. A- I and II only. B- III and IV only. C- I, II, and III only. D- I, II, III and IV.

Solution: The correct answer is C. Options "I", "II" and "III" are accurate. The number of shareholders is limited in an S corporation, but not in a C corporation. Capital may be raised by the sale of stock and by borrowing in capital markets.

Saul was divorced in 1996 and is now single, age 63. He has gross income of $50,000. His bona fide deductible expenses are as follows: Alimony = $8,000; Charitable contributions = $2,000; Contribution to an IRA = $2,000; Net expenses paid on rental property = $5,000; Interest on personal residence = $7,400; Taxes on personal residence = 4,000 State income tax = $2,200. What is Saul's AGI? A- $28,000 B- $33,000 C- $35,000 D- $40,000

Solution: The correct answer is C. Saul's AGI is calculated as follows: Gross income of $50,000 minus deductions for AGI of $15,000 (Alimony* paid of $8,000, IRA of $2,000, and expenses on rental of $5,000 - question facts stated this are bonafide deductible expenses) = $35,000. The others are deductions from AGI or below-the-line deductions. The itemized deductions are greater than the standard deduction for a single tax filer. *Alimony due to a divorce finalized by 12/31/18 will remain under the old rules, deductible to payor and included in income for the recipient. Divorces finalized after 12/31/18 will follow the new rules; not deductible and not includable in income.

Sean owns a personal fitness facility that specializes in training competitive athletes. Several years ago, Sean purchased a new weight machine for $2,000 and has since taken $600 in depreciation deductions. Sean is now ready to replace the weight machine with a more current model, but he is not sure what the tax consequences of selling the old weight machine will be. Which of the following statements is true regarding the tax consequences of selling the old weight machine? A: If Sean sells the old weight machine for $1,400, he will have a $600 capital loss. B: If Sean sells the old weight machine for $1,200, he will have a $600 ordinary gain. C: If Sean sells the old weight machine for $1,600, he will have an ordinary gain of $200. D: If Sean sells the old weight machine for $2,200, he will have a $800 capital gain.

Solution: The correct answer is C. Sean's amount realized of $1,600 less his adjusted basis of $1,400 equals $200. Sean's adjusted basis is $1,400 because he paid $2,000 for the machine and took $600 in depreciation deductions. Sean's gain is less than the depreciation taken, so the entire gain is taxed as ordinary gain.

With regard to Sections 1245 and 1250, Section 1231 will be applied only when: A- Any depreciable tangible personal property is sold at a profit. B- Any depreciable tangible personal property is sold at a profit above its adjusted (depreciated) basis. C- Any depreciable property is sold at a profit above its original cost. D- Any depreciable property subject to MACRS rules is sold at a profit.

Solution: The correct answer is C. Section 1231 gain is capital gain. Section 1231 gain occurs when the sale price exceeds the original purchase price.

Jody, an unmarried taxpayer using the single filing status, received $17,000 of Social Security retirement benefits this year. He also received $5,000 of interest income and $50,000 of income from his retirement plan during the year. How much of Jody's Social Security benefits must be included in his gross income this year? A: $0 B: $8,500 C: $14,450 D: $17,000

Solution: The correct answer is C. Since his MAGI ($55,000) plus one-half of his Social Security benefits (0.5 × $17,000 = $8,500) exceeds his adjusted base amount ($34,000), he must calculate his includible Social Security benefits using the formula 3 or 4 below. 0.85 × $17,000 = $14,450 0.85 × [$55,000 + (0.50 × $17,000) - $34,000] = $25,075 plus the lesser of the amount calculated using 1 and 2 below: 0.50 × $17,000 = $8,500 0.50 × [$55,000 + (0.50 × $17,000) - $25,000] = $19,250 The lesser amount is $8,500 The formula 4 total is $33,575 ($25,075 + $8,500) The lesser of the formula 3 or 4 amounts is $14,450. Therefore, $14,450 of the Social Security benefits must be included in Jody's gross income.

Which of the following completes this statement accurately? Alimony is... A- not deductible for income tax purposes by the payor spouse for divorces prior to 2019. B- subject to gift tax if paid to a third party on behalf of the intended recipient. C- not deductible by the payor if the original agreement was from 2017 and restructured in 2020. D- subject to recapture rules in the third year if the settlement was a front loaded property settlement in the current year.

Solution: The correct answer is C. Statement C is true. Any divorces materially altered after 12/31/2018 will be subject to the new rules, and no longer be deductible. Statement A is not a true statement. Alimony is deductible by the payor spouse and taxable to the payee spouse if the agreement is dated prior to or on 12/31/18. Statement B is not a true statement. There are no gift tax consequences related to qualifying alimony paid to a third party. Statement D is not true. Recapture rules are no longer applicable for divorces finalized after 12/31/2018. Those finalized in 2018 would have been subject to recapture rules up until 2021.

Tim and Janet were divorced. Their only marital property was a personal residence with a value of $100,000 and cost of $40,000. Under the terms of the divorce agreement, Janet would receive the house and Janet would pay Tim $10,000 each year for 5 years, or until Tim's death, whichever should occur first. Tim and Janet lived apart when the payments were made to Tim. The divorce agreement, dated December 15, 2017 did not contain the word "alimony." Which of the following is true? A: Tim must recognize a $30,000 [$50,000 - 1/2($40,000)] gain on the sale of his interest in the house. B: Tim does not recognize any income from the above transactions. C: Janet is allowed to deduct $10,000 each year for alimony paid. D: Janet is not allowed any alimony deductions.

Solution: The correct answer is C. The $10,000 cash meets all of the requirements for alimony treatment. Although the circumstances suggest that Janet is paying Tim for his share of the marital property, the agreement must specify that the payments are not alimony to avoid alimony treatment.

Which of the following statements concerning the Alternative Minimum Tax (AMT) system is correct? A- Deferral items for AMT purposes result in a permanent increase in tax. B- Exclusion items for AMT purposes may be reversed in future tax years. C- There is an unlimited carryforward for AMT credit generated in a year the taxpayer becomes an AMT taxpayer. D- The AMT credit can be carried back for up to two years, provided that use of the credit does not force the taxpayer to become an AMT taxpayer in those years.

Solution: The correct answer is C. The AMT credit that is generated from deferral items in the AMT calculation may be carried forward indefinitely. The AMT credit may not be carried back. Deferral items may be reversed in future years through the use of the AMT credit, while exclusion items result in a permanent increase in tax.

Lucy and Lou are married and normally file a joint return. Under which of the following circumstances are they required to file a tax return for 2023? A- If Lucy is 64 and Lou is 66 and their gross income is $28,200. B- If Lucy and Lou are both 35 and have one dependent and their gross income is $26,000. C- If Lucy is 64 and Lou is 64, Lou is blind, and their gross income is $28,200. D- None of the above.

Solution: The correct answer is C. The MFJ standard deduction is $27,700 and the additional standard deduction for 65 and over is $1,500 for MFJ. In option "A", their total standard deduction is ($27,700 + $1,500) = $29,200. In option "B", their standard deduction is $27,700. In option "C", their standard deduction is $27,700. Lou may benefit from an additional $1,500 additional standard deduction for blindness upon filing. A return must be filed to claim the ASD for blindness. The IRS will know your age, but not that you are blind.

A client sold an apartment building last year for $100,000, paying a sales commission of $5,000 plus $2,500 closing costs. The building originally cost $80,000 20 years ago. Total straight line depreciation of $40,000 had been taken. The building had a mortgage of $60,000 which was assumed by the buyer. What is the seller's adjusted cost basis prior to the sale? A- $32,500 B- $37,500 C- $40,000 D- $42,500

Solution: The correct answer is C. The adjusted cost basis is determined by subtracting from the acquisition costs any depreciation or unrecovered casualty losses. The adjusted cost basis was as follows: $80,000 (acquisition cost) minus $40,000 (depreciation) = $40,000 adjusted cost basis. NOTE: Only acquisition costs add to basis. The seller is covering the cost of disposition here, it does not add to their original basis.

Carl Borden is a contractor who has just purchased a tractor for use in his business. Carl paid $25,000 plus $1,250 in sales tax for the tractor. The local municipality also imposes an annual personal property tax of $500. The tractor has an expected useful life of 5 years. What is Carl's basis in the tractor for depreciation purposes? A- $25,000 B- $25,500 C- $26,250 D- $26,750

Solution: The correct answer is C. The basis of depreciable property begins with the acquisition costs plus any additional expense necessary to acquire or making such property ready for use. Here, sales tax is a required expense by law and is therefore added to the acquisition cost resulting in the depreciable basis. $25,000 (cost) + $1,250 (sales tax) = $26,250. The CFP® Exam deals with taxation at the federal level, state if specifically addressed. The municipal property tax (local tax, not state level tax), is not taken into account.

Arthur purchased a home for $175,000. Over the years, Arthur has made the following repairs and improvements: Room addition = $20,000 Pool = $6,000 Roof repair = $3,500 Painting = $2,000 Two years ago, a storm caused damage to the roof. Arthur decided to repair rather than replace the roof. Approximately six months prior to selling the home this year, Arthur had the house painted. What was the basis of the home when Arthur sold it in this past year? A- $206,500 B- $204,500 C- $201,000 D- $195,000

Solution: The correct answer is C. The basis of the home is increased by the addition of the room and pool. No increase is realized nor recognized at the time of sale for repairs made to the roof or for the painting prior to the sale. Roof replacement would add to basis (was not the case here), painting is cosmetic and does not add to basis.

Earl entertains one of his clients on January 15, 2023. Expenses paid by Earl are as follows: Cab fare = $22; Dinner at Chez Marco = $190; Tips to waiter = $38; Cover charge at night club = $40; Presuming proper substantiation, Earl's deductions for the night will be... A- $0 B- $114 C- $136 D- $250

Solution: The correct answer is C. The cab fare is deductible as a transportation expense at the full cost. The business meal and tip are subject to a deduction equal to 50% of the expense*. The entertainment expenses are not allowed deductions. = $22 + [50% x ($190 + $38)] = $22 + $114 = $136 *NOTE Expired Rule: Taxpayer Certainty and Disaster Tax Relief Act of 2020 allows meals provided by restaurants to be fully deductible when paid or incurred after December 31, 2020 and before January 1, 2023.

A client sold an apartment building last year for $100,000, paying a sales commission of $5,000 plus $2,500 closing costs. The building originally cost $80,000 20 years ago. Total straight line depreciation of $40,000 had been taken. The building had a mortgage of $60,000 which was assumed by the buyer. What is the purchaser's cost basis? A- $70,000 B- $92,500 C- $100,000 D- $107,500 E- $160,000

Solution: The correct answer is C. The cost basis to a purchaser is the acquisition cost plus any other costs associated with purchasing the property or making it useful for service. The buyer paid $100,000, and no acquisition costs. The question indicates that it was the seller, and not the buyer, who paid the sales commission and the closing costs. Therefore, the buyer's basis is only the acquisition costs of $100,000. The sale of a property is stated as the cost without regard to how the purchaser structured the deal. The structure was: cash $40,000 plus mortgage $60,000 which equal the purchase price of $100,000.

Which of the following statements regarding family limited partnership is not correct? A- The primary purpose of a FLP is to transfer assets to younger generations of a family using annual exclusion gifts and valuation discounts. B- Upon creation of a FLP, there are neither income nor gift tax consequences because the entity created is owned by the same person, or persons, who owned it before the transfer. C- Publicy traded securities make an ideal asset to transfer to a FLP. D- The use of a FLP can help protect family assets.

Solution: The correct answer is C. The creation of family limited partnerships and the use of discounts to transfer value at a lower gift tax cost has been regulary contested by the IRS. The risk of the IRS contesting any discounts for publicly traded securities is a significant risk associated with the FLP.

Chloe and Liam are married and own a self-liquidating investment that was originally purchased for $20,000 six years ago. YearDividend Distributions NondividendDistributions 1 $2,000 $0 2 $2,150 $4,000 3 $2,200 $2,000 4 $2,250 $3,000 5 $2,400 $9,000 6 $2,500 $4,000 In year six, how much will be included in Chloe and Liam's gross income for income tax purposes? A- $2,500 B- $4,000 C- $4,500 D- $6,500

Solution: The correct answer is C. The dividend distributions are taxable each year, so in year 6 the taxable dividends are $2,500. Nondividend distributions are taxable to the extent they exceed basis. The total nondividend distributions at the end of year 6 are $22,000, basis was $20,000, so only $2,000 is taxable gain. Dividends of $2,500 plus gain of $2,000 = $4,500.

Ford's federal income tax return was due on April 15 of the current year, but Ford did not file his return or pay his taxes until June 30 of the current year. Ford's unpaid tax balance during this period was $400. What is the total penalty that will be imposed on Ford for his failure to file and failure to pay? A- $6. B- $394 C- $400. D- $450.

Solution: The correct answer is C. The failure to file penalty is 5% of the unpaid tax balance for each month or part thereof that the tax return is late (up to 25% of the unpaid tax balance). Therefore, Ford's failure to file penalty is $60 (3 months × $400 × 5%). However, if a tax return is filed more than 60 days late (as it is in Ford's case), the minimum failure to file penalty is the lower of $450 or 100% of the tax due. The penalty will not exceed 100% of the tax due. Therefore, Ford's failure to file penalty is actually $400 (100% of taxes due) Ford is also subject to a failure to pay penalty of 0.5% per month or part thereof. Therefore, Ford's failure to pay penalty is $6 (3 months × $400 × 0.5%). Note that the failure to file penalty is reduced by the failure to pay penalty. Therefore, Ford's total penalty is $400 ($400 failure to file penalty - $6 failure to pay= $394, plus $6 failure to pay penalty). NOTE: Taxes filed for the tax year 2022 (filed in 2023 calendar year) will have a $450 minimum penalty. Taxes filed for the tax year 2023 (filed in 2024 calendar year) will have a $485 minimum penalty.

Brenda purchased 50 shares of Walsh Co. stock three years ago for $1,000. Brenda recently gifted the stock to her brother, Brandon. On the date of the gift, the stock had a fair market value of $750. Six months after receiving the stock from Brenda, Brandon decides to sell the stock. Which of the following statements is correct? A- If Brandon sells the stock for $700, he will have a long-term capital loss. B- If Brandon sells the stock for $1,100, he will have a short-term capital gain. C- If Brandon sells the stock for $675, he will have a short-term capital loss. D- If Brandon sells the stock for $800, he will have a long-term capital gain.

Solution: The correct answer is C. The following are the corrected statements, with basis noted. This is a gift of loss property. When double basis is in effect, the gains are that of the holding period of the donor. For loss in this case below $750, it will use the donee's basis, in this case short term loss. If Brandon sells the stock for $700 he will have a short-term capital loss of $50 (basis is $750). If Brandon sells the stock for $1,100 he will have a long-term capital gain of $100 (basis is $1,000). If Brandon sells the stock for $675 he will have a short-term capital loss of $75 (basis is $750). If Brandon sells the stock for $800 he will have no gain or loss.

Last year, Paris bought a home in Los Angeles. Paris is now considering selling her home and buying a new home, but she is not sure whether she can qualify for a prorated exclusion of the gain on the sale of her Los Angeles home under Section 121 of the IRC. Under which of the following circumstances would Paris not qualify for a prorated exclusion? A- Paris has decided to sell her house because she has accepted a new job in New York City. Her last job was in Los Angeles. B- Paris has decided to sell her house because her personal physician recommended that she move to the desert in Arizona because the smog in Los Angeles was causing her asthma to get worse. C- Paris has decided to sell her house because her dog Tinkerbell has arthritis and can't walk down the stairs in Paris' home. D- Paris has decided to sell her house because she is bothered by excessive noise from a nearby airport.

Solution: The correct answer is C. The health of a pet is not considered a change in health that justifies a partial exclusion under Section 121. All of the other options are reasons that would justify a partial exclusion under Section 121.

Two years ago, Bill purchased stock in Pinkley Corporation (the stock is not small business stock) for $1,000. In the current year, the stock became worthless. During the current year, Bill also had an $8,000 loss on small business stock (Section 1244) purchased two years ago, a $9,000 loss on a non-business bad debt, and a $5,000 long-term capital gain. What should Bill report this year? A- $4,000 long-term capital loss and $9,000 short-term capital loss. B- $3,000 long-term capital loss and $10,000 long term loss carry forward. C- $8,000 ordinary loss; $3,000 short-term capital loss and a $2,000 short-term capital loss carryover. D- $8,000 ordinary loss and $5,000 short-term capital loss.

Solution: The correct answer is C. The non-business bad debt is treated as a short-term capital loss. The loss on worthless stock held for more than one year is a long-term capital loss. The loss on small business stock Section 1244 is recognized as an ordinary loss not subject to the capital loss rules. Worthless stock has a fictitious sale on December 31st in the year of loss, which allows the stockholder to take the full loss. Note the following calculation: $5,000 (long-term capital gain) - $1,000 (long-term capital loss worthless stock) = $4,000. From this amount, subtract $9,000 (non-business bad debt expense - short-term capital loss) to obtain a net short-term loss of ($5,000.) However, the maximum annual capital loss deduction is net $3,000.

Leon, age 32, is an active participant in his employer's defined benefit plan, but he would also like to make a deductible contribution to a traditional IRA this year. Leon is married, files a joint return with his wife, and they have an AGI of $120,000 in the current year. What is the maximum deductible contribution that they each can make to a traditional IRA, assuming his wife is also an active participant? A- $0. B- $1,300. C- $5,200. D- $6,500.

Solution: The correct answer is C. The phase-out range for taxpayers who are active participants and use the married filing jointly filing status is $116,000 - $136,000 for 2023. Since Leon's AGI is within this range, he may not make a full $6,500 deductible contribution to a traditional IRA, but may make a reduced deductible contribution, as calculated by the following formula: Reduction = Contribution Limit × AGI-Lower Limit ÷ $20,000 Therefore, Leon's deductible contribution is reduced by $1,300 which is calculated as follows: [$6,500 × (120,000 - 116,000) / 20,000)]. Both spouses are active participants. The maximum deductible contribution that each can make to a traditional IRA is (6,500 - 1,300) = $5,200.

Leon, age 71, is an active participant in his employer's defined benefit plan, but he would also like to make a deductible contribution to a traditional IRA this year. Leon is married, files a joint return with his wife, and has an AGI of $111,000 in 2022. What is the maximum deductible contribution that Leon can make to his traditional IRA? A: $700 B: $5,300 C: $6,300 D: $7,000

Solution: The correct answer is C. The phaseout range for taxpayers who are active participants and use the married filing jointly filing status is $109,000 - $129,000 for 2022. Since Leon's AGI is within this range, he may not make a full deductible contribution to a traditional IRA, but may make a reduced deductible contribution. Leon can contribute $6,000 plus $1,000 catch up. Therefore, Leon's deductible contribution is reduced by $700 [$7,000 × (($111,000 - $109,000)/$20,000]. The maximum deductible contribution that Leon can make to a traditional IRA is $6,300. SECURE Act 2019 removed the age maximum on contributions to Traditional IRAs. Leon can continue to contribute if he has sufficient earned income.

Ryan is self-employed and owns a sole proprietorship involved in party planning. He is working a New Year's event. Under what circumstances will the income for this party be deferred for tax purposes until next year? A- Ryan's business is an accrual-basis taxpayer; the check is mailed on January 2 of next year, and received on January 5th. B- Ryan's business is an accrual-basis taxpayer; the check is mailed on December 31 of this year, and received on January 2nd. C- Ryan's business is a cash-basis taxpayer; the check is mailed on December 28th of this year, and received on January 1st. D- Ryan's business is a cash-basis taxpayer; the check is mailed on December 28th of this year, and received on December 31st.

Solution: The correct answer is C. The question is asking, which will have income deferred to the following year? Accrual-basis taxpayers must recognize the income in the year the services or goods are provided. Choice A will be recognized on December 31st. Choice B will be recognized on December 31st. Cash-basis taxpayers will recognize the income in the year that the payment is actually or constructively received. The only situation where the income is deferred until next year is when the taxpayer is cash basis and the check is received in the next year. Choice C will be recognized on January 1st. Choice D will be recognized on December 31st.

Arrange the following statutes of limitation from shortest to longest: I- Collection of deficiency by the IRS. II- Fraud. III- General Statue of Limitations under Section 6501. IV- Substantial Understatement of Income greater than 25%. A- I, II, III, IV. B- II, III, IV, I. C- III, IV, I, II. D- I, IV, III, II.

Solution: The correct answer is C. The statute of limitations for the collection of a deficiency by the IRS is 10 years. There is no statute of limitations for fraud. The general statute of limitations under Section 6501 is 3 years. The statute of limitations for a substantial understatement of income greater than 25% is 6 years.

Which statement is false with respect to the U.S. Tax Court? A: Appeals from the Tax Court are brought to the U.S. Court of Appeals. B: The Court is located in Washington, D.C., but the judges hear cases around the country. C: A taxpayer must pay the deficiency before litigating here. D: The Court hears only tax cases.

Solution: The correct answer is C. The taxpayer does not have to pay the tax before litigating in the U.S. Tax Court. The other options are true.

Donna sells stock in Martin Corporation to her brother David for $1,800. Donna purchased the stock four years ago for $3,000 and the current fair market value of the stock is $1,800. David paid Donna $1,800 for the stock. Which of the following statements is correct regarding the tax consequences of this transaction? A- If David subsequently sells the stock to an unrelated party for $3,500, he will realize a gain of $1,700. B- Donna has a recognized loss of $1,200. C- If David subsequently sells the stock to an unrelated party for $2,200, he will have no gain or loss. D- If David subsequently sells the stock to an unrelated party for $3,500, he will have no gain or loss.

Solution: The correct answer is C. This is a related party sale of loss property. Dual basis rules will apply. David will have a double basis in the stock, determined as follows:

If a 42-year old individual wants to begin taking substantially equal periodic payments from their IRA to avoid an underpayment penalty, the payments must continue for: A- 5 years. B- 13 years. C- 18 years. D- 29 years.

Solution: The correct answer is C. To avoid an underpayment penalty, substantially equal periodic payments must continue for the greater of: 1) 5 years, or 2) until the individual attains age 59 ½. Therefore, a 42-year old would need to continue payments for 18 years.

John owns a rental home in Arizona. He decided that he would like to acquire a rental home in Washington. Ted who lives in Washington has a rental home. For health purposes, Ted must relocate to Arizona. John and Ted decide to exchange properties under section 1031 of the code. The other facts pertaining to the exchange are: Ted's Basis = $100,000 John's Basis = $75,000 Ted and John exchange the two properties, but Ted has to give John an additional $25,000 in cash. The fair market value of Ted's property is $100,000, and the fair market value of John's property is $125,000. What is Ted's new basis? A- $25,000 B- $50,000 C- $75,000 D- $125,000

Solution: The correct answer is D.

To what extent may the rental losses of an active participant be deducted against active and passive income? I- $25,000 of losses from rental property income may be deducted against ordinary income. II- The taxpayer must be considered "active" in that they participate in the general management and decision making of the property. III- The $25,000 is reduced $1 for every $2 over an AGI limit of $100,000. IV- When the AGI reaches $150,000, the deduction is lost and must be treated as regular passive income. A- I and II only. B- II and III only. C- II, III and IV only. D- All of the above.

Solution: The correct answer is D. $25,000 of losses from rental property income may be deducted against ordinary income. The taxpayer must be considered "active" in that they participate in the general management and decision making of the property. Also, the $25,000 is reduced $1 for every $2 over an AGI limit of $100,000. When the AGI reaches $150,000, the deduction is lost and must be treated as regular passive income.

Tikia, Inc. is filing their corporate income tax return for last year with a tax-year end of September 29. What is their tax accounting period? A- Fiscal B- Calendar C- Part-year D- 52-53 week

Solution: The correct answer is D. A fiscal year ends on the last day of a month other than December. A calendar year ends on the last day of December. A partial-year is for a time span less than 1 year. The IRS does recognize 52-53 week if the year ends on a specified day of the week (such as Friday) that occurs in the last week of the last month of the tax year.

Kelly and Terry are separated and in the process of divorce so they are going to file their income tax return for last year separately. Kelly made a large number of charitable contributions from her own separate checking account at the end of last year so she wants to file using itemized deductions. What are Terry's options assuming Kelly files using itemized deductions? A- Terry may utilize the standard deduction or itemized deduction, whichever is less. B- Terry may utilize the standard deduction or itemized deduction, whichever is greater. C- Terry must utilize the standard deduction. D- Terry must utilize itemized deductions.

Solution: The correct answer is D. A married individual who files a separate return (married filing separately filing status) cannot use a standard deduction if that person's spouse itemizes deductions.

Aaron and Doris have been married for 53 years. They have two children and five grandchildren. During the current year, Aaron wrote a check out of his personal checking account to the University of Montana. The $80,000 check was for the payment of his oldest grandchild's tuition. Which of the following statements is correct regarding gift tax compliance related to the check? A- Aaron will need to file a gift tax return for the current year, but no gift tax will be due. B- Aaron will need to file a gift tax return for the current year, and generation-skipping transfer tax will be due. C- Both Aaron and Doris will need to file gift tax returns because a gift-splitting election must be made. D- There are no gift tax return filing requirements.

Solution: The correct answer is D. A payment directly to a university is considered a qualified transfer and is therefore not considered a gift for gift tax and generation-skipping tax purposes. Therefore, there are no filing requirements for the payment.

For purposes of determining taxable income, which of the following is true? A taxpayer who finds a suitcase of money and spends the money is not required to recognize income. The taxpayer who collects income from a customer during the year, but the customer has sued for a refund, can defer recognition of the income until the suit has been resolved. Embezzlement proceeds are not included in the embezzler's gross income because the embezzler has an obligation to repay the owner. A person who rents out their personal residence 10 days during the year is not required to include any income received.

Solution: The correct answer is D. A person who rents their home for less than 15 days is not required to include the income as it is considered personal property, not rental or mixed use. However, no deductions related to the expense of renting out the home are allowed other than taxes and interest associated with the property that would normally be deductible as an itemized deduction.

Under which of the following circumstances will a taxpayer be subject to an accuracy-related penalty? I- If the taxpayer files an incorrect return and has failed to make a good faith effort to comply with the tax law. II- If the taxpayer understates his tax liability by more than 5 percent of the correct tax liability. III- If the taxpayer makes a substantial understatement associated with an estate or gift tax valuation. A- I only. B- I and II only. C- II and III only. D- I and III only.

Solution: The correct answer is D. A taxpayer will be subject to an accuracy-related penalty if he makes a substantial understatement of his tax liability, generally more than 10 percent of the correct tax liability and at least a $5,000 tax deficiency. The penalty imposed is generally 20% of underpayment amount.

Janice, who is single, had gross income of $38,000, and incurred the following expenses: Charitable contributions (cash) = $2,500 Taxes and interest on home = $9,000 Legal fees incurred in a tax dispute = $1,000 Medical expenses = $4,000 Penalty on early withdrawal of savings = $200 Her AGI is: A- $38,000 B- $21,300 C- $29,000 D- $37,800

Solution: The correct answer is D. All but penalty for early withdrawal of savings are itemized deductions. The question asks to determine the AGI. Therefore, the AGI is $38,000 less $200 in penalties on savings (such as CDs) which equals $37,800. Schedule 1, part 2 addresses above the line deductions. Those flow through to the 1040 line 10. Line 11 is AGI or "the line".

Chuck purchases land for $250,000. He incurs legal fees of $1,000 associated with the purchase. He subsequently incurs additional legal fees of $15,000 having the land rezoned from agricultural to residential. He subdivides the land and installs streets and sewers at a cost of $500,000. What is Chuck's basis for the land and the improvements? A- $250,000 B- $750,000 C- $765,000 D- $766,000

Solution: The correct answer is D. All costs of making property ready for use are capitalized. Here, the acquisition cost, plus legal fees to consummate the acquisition, rezoning costs and property improvement for streets and sewers are capitalized and become a part of the property's basis. The total basis is $766,000.

Johnny and June are married and had the following income and expenses for the this year. Johnny's salary of $300,000 Johnny had royalties on the music he wrote of $40,000 June had salary of $50,000 and unemployment compensation of $10,000 June won $2,000 on a game show What is Johnny and June's joint gross income? A- $352,000 B- $362,000 C- $392,000 D- $402,000

Solution: The correct answer is D. All items will be included in income. Johnny's salary of $300,000 Johnny had royalties on the music he wrote of $40,000 June had salary of $50,000 and unemployment compensation of $10,000 June won $2,000 on a game show Total income: $402,000

Nina has been very ill this year and has a significant amount of medical expenses, as listed below. $1,200 in prescription drug costs. $2,000 in doctor's bills not reimbursed by Nina's insurance. $800 in travel expenses to see a specialist in another state (these expense fall within the IRS guidelines). $2,000 out of pocket for health insurance premiums. Nina's AGI is $20,000. What is the amount of her deduction for medical expenses? A- $1,500 B- $2,000 C- $4,000 D- $4,500

Solution: The correct answer is D. All of Nina's medical expenses are deductible medical expenses. Her total deductible medical expenses are $6,000. However, her actual deduction is limited to the amount of deductible medical expenses that exceed 7.5% of her AGI or $1,500. Therefore, her deduction for medical expenses is $4,500 ($6,000 - $1,500).

Which income is subject to self-employment tax: A- Self-employment income. B- Income from an LLC when acting as a member-manager. C- Distributive share of a general partner's income regardless of the nature of the partnership. D- All of the above.

Solution: The correct answer is D. All of the above answers will subject a taxpayer to self-employment tax.

Brody has set up a dividend reinvestment plan at Coca-Cola Enterprises, Inc. where all dividends are automatically reinvested into more Coca-Cola Enterprises stock. Brody never receives a check for the dividends. His initial stock ownership was 100 shares at a cost of $2,000. The total dividends for the year equaled $60. The administrative fee to convert into new shares was $5. How much, if any, of the dividends are included in Brody's gross income? A- None B- $9 C- $55 D- $60

Solution: The correct answer is D. All of the dividends are included in Brody's gross income for the year. These dividends, while not received by Brody in the form of cash, could be taken as cash upon his election to end the dividend reinvestment plan. He has "constructively" received the dividends and decided to reinvest them in stock of Coca-Cola Enterprises, Inc. The administrative fee may be deducted (after 2025 - currently suspended by TCJA) but all the dividends must be reported as income.

Which of the following is a risk faced by an owner of a business entity? A: Death or disability of another owner B: Retirement of another owner C: Bankruptcy D: All of the above

Solution: The correct answer is D. All of the options are potential risks faced by the owner of a business entity.

Which of the following is a risk faced by the owner of a multi-owner business entity? A- Death or disability of another owner. B- Retirement of another owner. C- Bankruptcy. D- All of the above.

Solution: The correct answer is D. All of the options are potential risks faced by the owner of a business entity.

Which of the following statements is correct? A- Gains on the sale of personal use assets are taxable. B- Losses on the sale of personal use assets are disallowed. C- If a personal use asset is sold at a gain and another personal use asset is sold at a loss, the gains and losses are netted. D- A and B only.

Solution: The correct answer is D. All statements except "C" are correct. Losses from personal use assets are not deductible.

Alberto Sanchez purchased a piece of equipment last year for his computer business. Which of the following depreciation methods would provide Alberto with the least depreciation during the period in question? A- MACRS. B- ACRS. C- Units of production. D- Straight line.

Solution: The correct answer is D. Although available as a tax depreciation method, straight line will provide the least depreciation expense for a given period. Option "A" provides the most depreciation, as well as one which will yield the greatest expense in the early portion of the asset's life. Option "B" is incorrect because ACRS application was discontinued in 1987. Option "C" is incorrect because it is not applicable to the type of equipment in question.

Taxpayer receives stock as a gift from his uncle. The adjusted basis of the stock is $10,000 and the fair market value is $17,000. Taxpayer trades the stock for bonds with a fair market value of $15,000 and $2,000 cash. What are his recognized gain and the basis for the bonds? A: $0; $8,000 B: $0; $15,000 C: $2,000; $10,000 D: $7,000; $15,000

Solution: The correct answer is D. Amount realized ($15,000 + $2,000) $17,000 Adjusted basis (10,000) Realized gain $ 7,000 Recognized gain $ 7,000 The property received by gift has a carryover basis of $10,000. Stocks and bonds do not qualify as like-kind property. Therefore, the realized gain of $7,000 is recognized. The bonds have a basis of $15,000, the fair market value.

Taxpayer exchanges a business use machine with an adjusted basis of $22,000 and a fair market value of $30,000 for another business use machine with a fair market value of $28,000 and $2,000 cash. What is the taxpayer's recognized gain? A: $0 B: $2,000 C: $6,000 D: $8,000

Solution: The correct answer is D. Amount realized ($28,000 + $2,000) $30,000 Adjusted basis (22,000) Realized gain $ 8,000 Recognized gain $ 8,000 Tax deferred treatment offered in like kind exchanges is no longer available to tangible personal property, like business machines. Only real property may use the like-kind exchange rules since TCJA 2017. The gain is taxable.

Edward told his nephew that if the nephew would care for Edward in his old age, the nephew could have all of Edward's securities when he died. At the time of the promise, the securities had a fair market value of $50,000. The nephew took good care of Edward, whose will left the securities to the nephew. The fair market value of the securities at the time of Edward's death was $80,000. Edward could have gone to a nursing home and obtained the same services as provided by the nephew for $40,000. The nephew's gross income from the above is: A- $0; this is an inheritance. B- $40,000; this is earned income at the fair market value. C- $50,000; this is earned income as of the time of the promise. D- $80,000; this

Solution: The correct answer is D. Because the agreement was to compensate Edward's nephew for his services, even though the transfer occurred following death, it is not a gift or bequest. It is compensation for services performed. Compensation of property has a value equal to its fair market value on the date of transfer. The fact that Edward died and a step up in basis would ordinarily occur is immaterial.

Bill and Nancy are married and together they have AGI of $80,000. They have no dependents and they file a joint income return. Each pays $1,200 for hospitalization insurance. During the year, they paid the following amounts for medical care: $7,200 in doctor and dentist bills and hospital expenses and $800 for prescribed medicine and drugs. In December, they received an insurance reimbursement of $1,500 for hospitalization. Determine the deduction allowable for medical expenses paid during the year. A- $8,900 B- $2,400 C- $0 D- $2,900

Solution: The correct answer is D. Bill and Nancy can claim a medical expense deduction for the current year of $2,900 determined as follows: Insurance Premiums $2,400; Doctor $7,200; Medicine $800; Total $10,400. Minus Reimbursement $ 1,500 = $8,900. 7.5% of 80,000 AGI = $6,000. Deduction will be 8,900 - 6,000 = $2,900

Abner owned bonds that paid $750 of interest on the first day of January each year. Exactly one-third of the way through the current year, Abner gave the bonds to his brother, Brody. When Brody receives the $750 of interest on the first day of January next year, what amount will be included in Brody's gross income next year? A- $750. B- $250. C- $0. D- $500.

Solution: The correct answer is D. Brody must include $750 on his Schedule B, but will subtract the $250 going to Abner, leaving $500 to carry over to the 1040. January 1, 202x the full $750 was paid to Abner based on holding it in the prior year. At some point in April 202x he transferred to Brody. The next payment is made on January 1, of the following year for the current year. Taxed in the year received, paid in arrears. Brody owes 1/3 of the payment to Abner for owning it January - April of the current year. Interest is not paid forward.

What effect does claiming the Section 179 deduction have on an asset's adjusted taxable basis? A- Immediate expense of the total cost and a resulting basis of zero not subject to recapture. B- Immediate expense of the total cost and a resulting basis of zero subject to recapture. C- Partial expense with a reduced basis and not subject to recapture. D- Any amount of expense not to exceed the total cost, subject to income and Section 179 expense limits, reduces the adjusted taxable basis.

Solution: The correct answer is D. Claiming Section 179 expense immediately reduces the basis of the property by whatever the amount claimed (not to exceed original cost). The amount of deduction is limited in total and further subject to income limitation before the deduction is taken.

Which of the following statements is correct? A- If an activity is classified as a trade or business, any net loss may be deductible against other income. B- If an activity is classified as a hobby, expenses can only be deducted against the income for AGI (not net losses). C- If an activity is classified as a hobby, the expenses are not deductible. D- Only A and C are correct.

Solution: The correct answer is D. Classification as a trade or business means it is an active activity. If it were passive it would use investment instead of trade or business. Business losses are deductible against other income presuming the activity is active and the person is a material participant. If the activity is deemed a hobby, the income is taxable, but no deduction is allowed for the related expenses.

For tax purposes, a deduction is allowed for the consumption of the cost of an intangible asset through: A- Depletion. B- Depreciation/Cost Recovery. C- Goodwill. D- None of the above.

Solution: The correct answer is D. Cost recovery of an intangible asset is allowed through amortization. Cost recovery and depreciation (one in the same) are applied to tangible assets. The costs of natural resources are recovered through depletion.

Which of the following must be capitalized by a business? A- A major tune-up of a truck used in the business. B- The replacement of a windshield of a business truck, which was broken in an accident. C- The repair of a roof damaged by a hurricane. D- The cost of a concrete pad upon which to place a machine.

Solution: The correct answer is D. Costs to improve, better, or extend the life of an asset are capitalized. Tune-ups are a maintenance activity. Maintenance and repairs (roof and windshield) are period costs deductible in the period in which they were incurred.

Which of the following is incorrect? A- All business deductions are classified as deductions FOR AGI. B- Some personal deductions are classified as deductions FROM AGI. C- Some business and some personal deductions are classified as deductions FOR AGI. D- Some business and some personal deductions are classified as deductions FROM AGI.

Solution: The correct answer is D. Deductions occur above the line (for) AGI and below the line (from) AGI. All business deductions are for AGI (above the line). All business expenses (taken by their owners, i.e. sole proprietor or partner) are taken above the line or FOR AGI. "Business deductions" are deductible directly against the business income, and will be reported on Schedule C or Schedule E (flow through from a K-1 is reported on Schedule E, and rental income and expenses are reported on Schedule E as well). On those forms, the business income and business deductions are netted, and the net amount is reported on the Form 1040 above-the-line. QBI is not a business expense against revenue. It is a deduction based on pass through income of the owners. The QBI deduction is at the individual level and is taken below-the-line. It is not a direct deduction against business income, it is a deduction against the individual owner's share of income. Due to the phaseouts being based on each owner's adjusted taxable income, it could be available for one owner, but not for another owner of the same business. "Personal deductions" are not tied to a business. For example, mortgage interest is a personal deduction below the line (FROM AGI). An HSA contribution is a personal deduction above the line (FOR AGI). "Job-related employee expenses" used to be deductible as a miscellaneous itemized deductions subject to the 2% floor, a below the line deduction (FROM AGI). Those are no longer available for tax years 2018-2025.

Ellen lost a $40,000 deposit in Tan financial institution when Tan became insolvent. If Ellen has a salary of $50,000, short-term capital gains of $38,000, and no other deductions, determine the maximum deduction Ellen may take with respect to the loss in computing this year's taxable income. A- $31,000 B- $34,900 C- $38,000 D- $40,000

Solution: The correct answer is D. Ellen would treat the loss as a short-term capital loss rather than as a personal casualty loss. She could use $38,000 of the Short Term Capital Loss (STCL) to offset the $38,000 capital gains. She deposited $40,000, so the remaining $2,000 of the STCL can then be deducted against ordinary income since it is within the $3,000 statutory limit.

Which of the following is a deductible miscellaneous itemized deductions? A- Union dues. B- Appraisal fees for establishing a casualty loss. C- Job hunting expenses. D- Gambling losses to the extent of gambling gains.

Solution: The correct answer is D. Gambling losses and all deductions incurred in carrying out wagering transactions are limited to gambling winnings for 2018 - 2025. Miscellaneous itemized deductions not subject to the 2% floor: 1. Gambling losses (to the extent of gambling income) 2. Credit for estate taxes imposed on IRD (income in respect of a decedent's assets) 3. Loss on the disposition of an annuity contract 4. Repayments of income (such as repayments of Social Security income when the taxpayer fails the earnings test) TCJA of 20177 suspended itemized deductions that are subject to the 2% floor have been suspended after Dec 31, 2017 - before Jan 1 2026.

On January 1st of this year, Linda sold a piece of land she had had for years to George. Linda's basis in the land was $75,000 and she sold it for $100,000. It was agreed that George would pay Linda $10,000 as a down payment and would make installment payments of $10,000 for the next 9 years plus 10% interest. His second payment was due and payable December 31 of this year. What is Linda's tax consequence of this transaction this year? A- $20,000 of ordinary income B- $20,000 of long term capital gain C- $2,500 of long term capital gain and $9,000 of ordinary income D- $5,000 of long term capital gain and $9,000 of ordinary income

Solution: The correct answer is D. George is paying her $100,000. Her amount invested is $75,000. Therefore, over 10 years, her total profit will be $25,000 or $2,500 per year except for the down payment. There are two payments at the end of the year. An interest payment of 10% × $90,000*=$9,000 (ordinary income). The second payment, $10,000, consists of $2,500 capital gain and $7,500 of return of basis. *Recall the amount paid was $100,000 less a down payment of $10,000, so $90,000 was outstanding. The question is asking for the current year, the full schedule is for informational purposes. January 1st was the sale date, December 31 is the first payment made after the downpayment was paid.

Harry inherited a residence from his mother when she died. The mother had a tax basis of $566,000 for the residence when she died and the residence was worth $433,000 at the date of her death. Which of the statements below is/are correct? A: Harry's holding period for the residence includes his mother's holding period for the residence. B: Harry's holding period for the residence does not include his mother's holding period for the residence. C: Harry's holding period for the residence is long term. D: b and c.

Solution: The correct answer is D. Harry has an automatic long-term holding period (regardless of his mother's holding period), because the residence is inherited property. If Harry's mother purchased the residence two weeks before her death, the holding period would still be long term for Harry. It is important to understand only gifts keep the donor's basis and holding period, not an inheritance.

Gene has an adjusted gross income of $550,000 and considers himself a savvy investor. He purchased an investment in a limited partnership four years ago, and it has reported losses for each of those four years. Gene is interested in purchasing a 3% interest in a master limited partnership (MLP) in the current tax year. Which of the following statements is/are correct regarding the tax consequences of the purchase of this investment? I- Losses from the MLP will be considered passive losses. II- The losses from the limited partnership can be used to offset income from the MLP if both partnerships are oil and gas investments. III- Interest income earned by the partnership will be included in the net investment income tax calculation on Gene's individual income tax return. IV- If the MLP is profitable, none of the losses from the limited partnership can be used to offset the MLP income. A- III only. B- II and IV. C- I, II, and III. D- I, III, and IV.

Solution: The correct answer is D. II is incorrect. Although special rules apply to oil and gas interests, losses from a limited partnership cannot be used to offset income from a master limited partnership.

Which of the following statements are correct regarding the home office deduction for federal income tax purposes, prior to TCJA 2017? I- Employees may only take the home office deduction if they maintain the home office for the convenience of their employer. II- If the home is later sold, the entire gain on the sale can potentially be excluded from income, even if the homeowner claimed a depreciation deduction for the home office. III- The deduction is limited to income from home office activities, but any allowable excess deduction may be carried forward to future tax years. IV- The home office must be used exclusively and regularly as the principal place of business or to meet and deal with customers in the normal course of business. A- I and II only. B- I, II, and III. C- II and III only. D- I, III, and IV

Solution: The correct answer is D. II is incorrect. In general, any depreciation allowed or allowable on the home office portion of the home is subject to recapture (included in income) when the home is sold or otherwise disposed of. Taxpayers must adjust the basis of the home for any depreciation that was allowable for business use.

Which of the following statements regarding the deduction of costs associated with investigating the purchase of a new line of business is not correct? A: Assuming the taxpayer is not currently engaged in business, if the new line of business is not purchased, no deduction is permissible. B: If the new line of business is purchased and it is in the same line of business as the current trade or business operation, the cost of investigating the new business is fully deductible. C: The ability to deduct the cost of investigating a new line of business is often overlooked by taxpayers. D: If the new line of business is purchased and it is in a different line of business as the current trade or business operation, there is no way to recoup the costs of investigation.

Solution: The correct answer is D. If the new line of business is purchased and it is in a different line of business as the current trade or business operation, the costs of investigation are recouped by capitalizing the expenses and amortizing it ratably over a 60-month period.

In calculating a net operating loss for an individual, which of the following items would not be added back to negative taxable income? A- Net operating loss deduction B- Long-term capital losses in excess of short-term capital gains. C- Section 1202 exclusions D- None of the above would be added back.

Solution: The correct answer is D. In general, the following items are not allowed when figuring an NOL. · Any deduction for personal exemptions. · Capital losses in excess of capital gains. · The section 1202 exclusion. · Nonbusiness deductions in excess of nonbusiness income. · Net operating loss deduction. · The domestic production activities deduction.

Anbu has a $1,000,000 whole life insurance policy on his life, with his cousin currently named as the beneficiary. The policy has a cash value of $67,000 and Anbu pays an annual premium of $18,000. He is considering changing the primary beneficiary designation from his cousin to a qualified public charity. If he does so, he will: A- Receive an immediate gift tax charitable deduction, but not receive an income tax charitable deduction. B- Receive an immediate income tax charitable deduction, but not receive a gift tax charitable deduction. C- Receive an immediate income tax charitable deduction and receive an immediate gift tax charitable deduction. D- Not receive a gift tax charitable deduction and not receive an income tax charitable deduction.

Solution: The correct answer is D. In order for Anbu to receive any current tax benefits, he must assign the life insurance policy to charity. A mere beneficiary change will not result in income or gift tax charitable deductions.

James and Marilyn Herbert are married and own a vacation home on the beach in Florida. Each summer they are able to rent the property for $1,000 per week. This year they rented the property to six different parties and the total rental period was 133 days. James and his family vacationed there in the fall for three weeks. Expenses for the entire year totaled: Mortgage Interest: $13,500 Mortgage Principal: $4,200 Real Estate Taxes: $6,400 Utilities: $2,300 Trash: $300 Management Fees: $3,800 Depreciation: $10,000 What is the Herbert's profit or loss, and its nature, associated with this property for the current year? A- $21,500 loss, not deductible. B- $8,000 loss, deductible. C- $12,350 loss, not deductible. D- $2,414.87 loss, not deductible.

Solution: The correct answer is D. Interest and taxes are accrued daily. $13,500 Mortgage + $6,400 Real estate taxes = $19,900 × (133÷365) = $7,251.23 Other costs are deducted according to use time (133 + 21 = 154) $16,400 (utilities, trash, management fees, & Depreciation) × (133 ÷ 154) = $14,163.64 Total Costs $21,414.87 - Total Revenue $19,000 (133 ÷ 7 = 19 × $1,000) = Loss $2,414.87 not deductible due to mixed use.

Eric's daughter Emilie is attending a local university. Listed below are the items that Eric has paid for this year related to Emilie's education. What are Eric's total qualified tuition and related expenses for the purpose of claiming the American Opportunity credit? $12,000 tuition paid directly to the university. $400 student activity fee which is required by the university for enrollment. $600 in textbooks purchased from an off-campus bookstore. $100 lab fee for a required science course. A- $12,000. B- $12,500. C- $12,700. D- $13,100.

Solution: The correct answer is D. Item 1 is a qualified tuition and related expense. Item 2 is a qualified tuition and related expense because it is required by the university for enrollment. Item 3 is a qualified tuition and related expense because textbooks can be purchased directly from the school or another off campus source and still qualify as a qualified education expense. Item 4 is a qualified tuition and related expense because the class is required as part of Emilie's degree program.

Ivan invests in land and Grace invests in taxable bonds. The land appreciates by $5,000 each year, and the bonds earn interest of $5,000 each year. After holding the land and bonds for five years, Ivan and Grace sell them. There is a $25,000 realized gain on the sale of the land and no realized gain or loss on the sale of the bonds. Which of the following best describes the tax consequences to Ivan and Grace for the five years? I- Ivan will recognize a capital gain of $25,000 since appreciation (or the gain) on sale is not taxed until the asset is sold. II- Grace will have recognized the interest received over the years. III- Ivan will not have recognized gain over appreciation of unsold property. IV- Grace will not recognize a gain on sale since the bonds haven't appreciated. A- I only. B- II only. C- I, II and III only. D- All of the above.

Solution: The correct answer is D. Ivan will recognize a capital gain of $25,000 since appreciation, or the gain on the sale, is not taxed until the asset is sold. Grace will not recognize a gain on sale since the bonds haven't appreciated. Grace will however have recognized the interest received over the years while Ivan will not have recognized gain over appreciation of unsold property. Note: The question asks the tax consequences over the 5 years. While Ivan was holding the property (remains unsold) there is no gain recognition. This is an accurate statement. The bonds would have recognition of income each year, even before they sell them.

Jackie and Julie have been in a long-term relationship and were married on January 1 of the current year. They intend to file a joint return for all years in which they are eligible to do so. Neither Jackie nor Julie have ever been married before and neither has any children or other dependents. What filing status can Julie use on her tax return for last year? A- Married Filing Jointly. B- Surviving Spouse. C- Head of Household. D- Single.

Solution: The correct answer is D. Jackie and Julie are not eligible to file as Married Filing Jointly for the prior year because they were not married as of the end of the tax year (December 31). Since Julie has never been married before, she cannot be eligible to file as a Surviving Spouse. Furthermore, Julie is not eligible to file as Head of Household since she does not have a qualifying child or a qualifying relative who is her dependent. Therefore, Julie must use the Single filing status for last year.

Terry and Jim are both involved in operating separate illegal businesses. Terry operates a gambling business and Jim operates a drug running business. Both businesses have gross revenues of $500,000. The businesses incur the following expenses: Terry's employee salaries = $200,000 Terry's bribes to police = $25,000 Terry's rent and utilities = $50,000 Terry's cost of goods sold = $-0- Jim's employee salaries = $200,000 Jim's bribes to police = $25,000 Jim's rent and utilities = $50,000 Jim's cost of goods sold = $125,000 Which of the following statements is correct? A- Terry should report a profit of $225,000. B- Jim should report a profit from his business of $100,000. C- Jim should report a profit from his business of $500,000. D- Jim should report a profit from his business of $375,000.

Solution: The correct answer is D. Jim's profit would be as follows: $500,000 (income) - $125,000 (cost of goods sold) = $375,000 of profit. Terry's profit would be as follows: $500,000 (income) - $200,000 (wages) - $50,000 (rental expenses) = $250,000 of profit. Please note that illegal expenses such as bribes to police are neither allowable nor deductible. If the illegal business is drug running then you are limited to only deducting the cost of goods sold. The costs associated in doing business (even illegal) can be deducted. Unless you are running drugs, then you can only deduct cost of goods sold, nothing else. It is an IRS rule (strange as it is). A good example of this marijuana sales and dispensaries. Many states have legalized it, but the federal government has not. The added federal taxes that will be paid due to the higher profit the company must report for federal tax purposes certainly has an impact of the investment analysis of the business versus another business that is permitted to reduce their taxable income by all expenses.

Which of the following taxpayers can use the standard deduction? A- Zeke, who files a separate return from his wife Yasmine. Yasmine itemizes deductions on her return. B- Xavier, who is a nonresident alien. C- William, who files a tax return for less than 12 months because he changed his annual accounting period. D- Violet, who is a non-citizen spouse but files MFJ

Solution: The correct answer is D. Only answer D describes a taxpayer who is permitted to use the standard deduction. All of the other taxpayers are required to itemize their deductions. In three situations, a taxpayer is not allowed to use the standard deduction and must itemize deductions: 1. A married individual who files a separate return (married filing separately filing status) cannot use a standard deduction if that person's spouse itemizes deductions. 2. A nonresident alien and a dual-status alien is not allowed to use a standard deduction. 3. An individual who files a tax return for less than 12 months because of a change in annual accounting period is not allowed to use a standard deduction (not common for individual taxpayers).

Section 1245 recapture does not apply to business equipment held for 17 months or longer if: A- The property was destroyed by fire and the insurance recovery exceeds the property's adjusted basis. B- The property was sold for a gain but was depreciated using straight-line depreciation rather than MACRS. C- The property was acquired, depreciated, and exchanged for a less valuable asset where the buyer of the asset paid additional cash in the exchange. D- The property was abandoned as worthless.

Solution: The correct answer is D. Option "A" is incorrect because if the insurance proceeds exceed the property's adjusted basis, the excess is considered a sale and any portion of gain attributable to depreciation will be subject to Section 1245 recapture. Option "B" is incorrect because Section 1245 recovery occurs any time a gain results from the reduction of basis due to depreciation. Option "C" is incorrect because Section 1245 applies to gain resulting from a reduced basis due to depreciation. Option "D" is correct. Property sold or abandoned below the basis adjusted by depreciation is not subject to Section 1245 recapture because either not all depreciation was taken or there was more likely a loss rather than a gain. For 1245 recapture to occur there must be a gain over the basis.

Which of the following is correct? A- Realized gains are always recognized and realized losses are never recognized. B- Realized gains and realized losses on the sale of personal use assets are not recognized. C- Realized gains and realized losses on the sale of personal use assets are recognized. D- In an involuntary conversion, the date of realization, not the payment date, determines the date of recognition.

Solution: The correct answer is D. Option "A" is incorrect because realized gains may, in some instances, be deferred or excluded such as the gain on sale of a personal residence. Option "B" is incorrect because realized gains on some personal use assets are recognized unless excluded. Option "C" is incorrect because realized losses on personal use assets are not recognized.

Ethan is starting a new business with his best friend Max. They are concerned about liability. They would prefer flow-through taxation because they expect to have losses in the first few years. They are not concerned about incurring self-employment taxes. Which of the following entities would best suit Ethan and Max's needs? A- A General Partnership. B- A Proprietorship. C- A C Corporation. D- A LLC.

Solution: The correct answer is D. Option "A" is not correct because a partnership would not provide the limited liability that Ethan and Max are looking for. Option "B" is not correct because there are multiple owners. Option "C" is not correct because Ethan and Max want flow-through taxation. Only the LLC meets all of Ethan and Max's requirements.

Carlos recently won a $20,000 prize for outstanding achievement in his field. Carlos has requested that the prize money be paid directly to his favorite charity. Which of the following requirements must be met in order to exclude the prize money from Carlos' gross income? A- The prize must have been awarded in recognition of religious, charitable, scientific, educational, artistic, literary, or civic achievement. B- Carlos must not have applied for the prize. C- Carlos must not be required to render substantial future services in order to receive the prize. D- All of the above.

Solution: The correct answer is D. Options "A," "B" and "C" are all requirements that must be met in order to exclude the prize from Carlos' gross income when donating to a charity. The award will not be included in his gross income based on exclusion covered in 26 US Code § 74(b): Exception for certain prizes and awards transferred to charities Gross income does not include amounts received as prizes and awards made primarily in recognition of religious, charitable, scientific, educational, artistic, literary, or civic achievement, but only if— (1) the recipient was selected without any action on his part to enter the contest or proceeding; (2) the recipient is not required to render substantial future services as a condition to receiving the prize or award; and (3) the prize or award is transferred by the payor to a governmental unit or organization described in paragraph (1) or (2) of section 170(c) pursuant to a designation made by the recipient.

Larry, Mike and Clarence wish to start a manufacturing business. The business is expected to generate a large income which it will reinvest for many years. Larry has substantial assets which he plans to contribute to the business. Larry is also concerned about showing too much business income on his personal return. Which of the following structures would serve as an appropriate form of organization to satisfy Larry, Mike and Clarence's objective? I- A limited partnership with Larry as the limited partner. II- A business trust with all three as equal interests. III- An S corporation with all three as equal shareholders. IV- A C corporation with all three as shareholders. A- I, II and III only. B- I and IV only. C- II and IV only. D- IV only.

Solution: The correct answer is D. Options "I," "II" and "III" are all pass through entities which will require the owner to report on their personal income tax return their pro rata share of income. Therefore, Options "I," "II" and "III" are all incorrect. The C corporation will not expose the shareholders to their share of undistributed corporate income.

Sarah is a 10 percent owner in Canine Connection, LLC, a day-care center for dogs. She is also a 15 percent owner in Little Laughter, LLC, a successful children's clothing store. She does not materially participate in either business. Her at-risk and loss/income for the current year is as follows: Canine Connection-At-risk = $175,000; Loss of $275,000 Little Laughter-At-risk = $25,000; Income of $125,000 She also has wage income of $80,000 and capital gain income of $30,000. Which of the following statements is true? A- The loss suspended because of the at-risk rules is $75,000 and the loss suspended because of the passive activity loss rules is $75,000. B- The loss suspended because of the at-risk rules is $75,000 and the loss suspended because of the passive activity loss rules is $0. C- The loss suspended because of the at-risk rules is $50,000 and the loss suspended because of the passive activity loss rules is $100,000. D- The loss suspended because of the at-risk rules is $100,000 and the loss suspended because of the passive activity loss rules is $50,000.

Solution: The correct answer is D. Passive Activity rules are for taking deductions based on loss in a passive activity. Think of passive activity loss in terms of filters. Filter one you look at the At Risk Amount. Filter two, you look at the Passive Income Amount. She has 175k invested (at risk) in Canine Connection, she can take up to that in losses for the first filter (at risk) Of the 275k loss, only 175k of the loss can go through to the second filter (passive income offset), and the remaining 100k of loss is suspended for this year under the at-risk rules. Next we look for passive income to offset loss. Little Laughter has 125k of income we can offset with the 175k of loss that made it through the first filter. We cannot take more passive loss than passive income, so the175k loss - 125k of passive income leaves 50k of loss we cannot take and will be suspended under passive activity rules.

Kate owns a downtown office building. Kate originally purchased the building for $900,000 and took depreciation deductions of $400,000. Kate is in the 37% tax bracket. Straight-line depreciation would have been $400,000. What are the tax consequences if Kate sells the building for $2,100,000? A- Kate will have ordinary income of $0. B- Kate will have $400,000 of gain taxed at 25%. C- Kate will have 1231 gains of $1,200,000 taxed at 20%. D- All of the above.

Solution: The correct answer is D. Sale Price $2,100,000 Less Adjusted Basis $500,000 ($900,000 - $400,000) Equals Gain $1,600,000 Breakdown of Gain $400,000 Straightline Depreciation Recapture (25%) $0 Excess Depreciation (Ordinary Income) $1,200,000 Gain (Capital Gain) The capital rate in this case will be 20% because she is in the 37% tax bracket.

Sara was the victim of sexual harassment and collected the following monies from her employer: $15,000 for lost wages and $50,000 in punitive damages. How much of the damage award must Sara include in her gross income? A- None B- $15,000 C- $50,000 D- $65,000

Solution: The correct answer is D. Sara must include all $65,000. Lost wages are nothing more than recovery of wages which would have been taxable. Punitive damages are always included as taxable income. Other than damages awarded for personal injury, other damages, including punitive damages and recovery of lost wages, are taxable. Punitive damages are always included in income. They may be excludible if they are the only recovery in a wrongful death case (The Alabama Rule).

Which of the following distributions of IRC Section 1245 recapture property may result in the immediate recapture of some or all of previous depreciation deductions? A- A distribution by a partnership to one of its partners. B- A non-simultaneous like-kind exchange. C- A disposition at death. D- A sale for an interest-bearing note.

Solution: The correct answer is D. Section 1245 recapture is applied to the sale of depreciated assets. Option "A" is incorrect because the distribution is a property distribution and not a sale. Option "B" is incorrect because there is no "sale" as part of a like-kind exchange. Option "C" is incorrect because the property transferred at death is not classified as a sale. Option "D" is correct because it is a sale, regardless for cash, notes, either or both.

Assuming an asset is sold for a gain, when would Section 1250 ordinary income occur? A- Depreciable property is sold at a gain. B- Depreciable property is sold regardless of whether there is a gain or loss. C- Straight line depreciation is used on real property subject to ACRS. D- Real property subject to ACRS and accelerated depreciation was used.

Solution: The correct answer is D. Section 1250 gain applies to the realized gain on real property where the accelerated method was used. The gain is the excess of accelerated over straight line (ACRS). Section 1250 gain is taxed as ordinary income. Under current law (MACRS), only straight line depreciation of real property is used.

Emily, whose husband died in December of this year, maintains a household in which her dependent daughter lives. Which of the following is most likely to be her filing status for this year? A: Single B: Surviving spouse C: Head of household D: Married, filing jointly

Solution: The correct answer is D. Since she is deemed married in the year of her husband's death, she cannot file as single (choice A) or head of household (choice C). She does not qualify for surviving spouse status until the next year. Therefore, she is most likely to use the married filing jointly filing status.

Which of the following is true of the substitute basis of a qualifying asset in a like-kind exchange? A- The substitute basis is the asset's fair market value increased by the gain realized but not recognized. B- The substitute basis is the asset's basis reduced by the gain realized but not recognized. C- The substitute basis is the asset's basis increased by the gain realized but not recognized. D- The substitute basis is the asset's fair market value reduced by the gain realized but not recognized

Solution: The correct answer is D. Substitute basis is the fair market value of an asset, reduced by gain realized, but not recognized. Instructor note: If like kind exchange is a struggle, please ensure you watch the pre-study lecture on like kind exchange.

Nell sold a passive activity with an adjusted basis of $50,000 for $90,000. Suspended losses attributable to this property were $30,000. Her taxable gain is: A- $70,000 taxable gain. B- $40,000 taxable gain. C- $60,000 taxable gain. D- $10,000 taxable gain.

Solution: The correct answer is D. Suspended losses for a passive activity (passive losses carried forward) are expensed when the passive activity either has a passive income for that year or the activity is disposed. If passive income is available for a year in which there are suspended losses, then loss to the extent (but not more than) the passive income, are used to offset income and thus reduce the total suspended loss. When an activity is disposed, the total suspended losses are applied to the disposition resulting in a gain or loss on disposition. $90,000 (sale price) - $50,000 (adjusted basis without application of suspended loss) = $40,000 of gain on disposition before application of suspended loss. $40,000 - $30,000 (suspended losses from prior years) = $10,000 (adjusted gain on disposition).

Marge made a $60,000 interest-free loan to her son, Steve, who used the money to buy an automobile. Steve's only sources of income were $25,000 from wages and $250 of interest on his checking account. The relevant Federal interest rate was 5%. Based on the above information: A: Marge must recognize $250 of imputed interest income on the below market loan. B: Marge must recognize $1,000 of imputed interest income on the below market loan. C: Marge must recognize $3,000 of imputed interest income on the below market loan. D: Marge is not required to impute any interest.

Solution: The correct answer is D. The $100,000 exception would apply, so interest will need to be imputed to the lender if the Borrower's net investment income exceeds $1,000. The lender will only impute the lesser of the Borrower's net investment income or the AFR. Marge is not required to recognize imputed interest income because Steve's investment income is less than $1,000.

Which of the following tests must be satisfied by a qualifying child? I- Relationship Test. II- Gross Income Test. III- Abode Test. IV- Citizenship Test. A- I and II only. B- III and IV only. C- I, II and III only. D- I, III, and IV only.

Solution: The correct answer is D. The Gross Income Test is a requirement for a qualifying relative, not a qualifying child. All of the other tests must be satisfied by a qualifying child. In addition to either the qualifying child or relative test, a dependent must be a citizen and must not be required to file a joint tax return (unless to claim a refund).

In Year 1, Ted purchased an annuity for $60,000. The annuity is to pay him $1,500 per month for the rest of his life. His life expectancy is 120 months. Which of the following is true? A: Ted is not required to recognize any income until he has collected 40 payments (40 × $1,500 = $60,000). B: If Ted collects 24 payments and then dies in Year 3, Ted's estate should amend his tax returns for Year 1 and Year 2 and eliminate all of the reported income from the annuity for those years. C: If Ted lives and collects on the annuity for 130 months, the amounts received in the last 10 months are excludible from his gross income. D: For each $1,500 payment received in the first year, Ted must include $1,000 in gross income.

Solution: The correct answer is D. The annuity exclusion formula is investment/expected return = $60,000/($1,500 × 120) = $60,000/$180,000 = .333. Therefore, when Ted collects $1,500, he must recognize $1,500 × (1 - .333) = $1,000 gross income. Answer b. is incorrect. Instead of amending prior returns, Ted will be allowed to deduct a loss on his final return for the cost of the annuity less the amount previously excluded as a return of capital.

If personal use property is converted to business use: A- Gain is recognized on the date of conversion to the extent of the excess of the fair market value over the adjusted basis. B- Loss is recognized on the date of conversion to the extent of the excess of the adjusted basis over the fair market value. C- There is no gain or loss from the later sale of the property if the sales price is below the original adjusted basis and over the fair market value at the date of conversion. D- The adjusted basis is the lower of the taxpayer's adjusted basis or the fair market value on the date of conversion.

Solution: The correct answer is D. The basis of personal property converted to business use will be the taxpayer's adjusted basis on that property as of the date of conversion or the FMV if lower. Option "A" is incorrect because no gain on conversion is recognized. Option "B" is incorrect because no loss on conversion is recognized. Option "C" is incorrect because the statement pertains to the basis of gifted property.

Harry and Wilma are married and file a joint income tax return. On their return, they report $60,000 of adjusted gross income ($38,000 salary earned by Harry and $22,000 salary earned by Wilma). During the year, they paid the following amounts to care for their 4-year old son and 6-year old daughter while they worked: Check paid to ABC Day Care Center = $3,000 Check paid to the nanny who also cleans the house and does laundry = $2,000 Cash paid to Mrs. Smith (Harry's mother) = $1,500 Harry and Wilma may claim a credit for child and dependent care expenses of: A- $2,100 B- $1,000 C- $3,000 D- $1,200

Solution: The correct answer is D. The child care credit applies to child care expenses not to exceed $3,000 for one child and $6,000 for two or more children. Check paid to ABC Day Care Center = $3,000 Check paid to the nanny who also cleans the house and does laundry = $2,000 Cash paid to Mrs. Smith (Harry's mother) = $1,500 Total paid for care: $6,500 Based on the AGI, the maximum allowable credit is 20% of the total care costs (to a $6,000 maximum.) $6,000 care cost for two children × 20% = $1,200. It is irrelevant that the nanny also cleans. The money paid to the grandmother is also included in the childcare costs.

Which of the following is correct about property transferred between spouses, or former spouses, incident to a divorce? A- Property transfers within 18 months of the divorce will have a basis equal to its fair market value at the date of transfer. B- Property transfers are NOT treated as if it were acquired by gift and gain or loss will be recognized to the transferor. C- Property transfers are treated as if acquired by gift and the basis carried over to the transferee is the lower of the fair market value or the transferor's basis. D- Property transfers are treated as acquired by gift and the transferor's basis in the property is carried over to the transferee.

Solution: The correct answer is D. The correct answer may also read "property transfer will result in the transferor's basis in the property being carried over to the transferee as if the property is acquired by gift." Transfers between divorcing spouses ALWAYS transfer their basis and their holding period, regardless of FMV at date of transfer.

One of the five tests that must be met to qualify as a dependent is: A- Whether the dependent is employed. B- Whether the dependent is living in the household. C- The dependent is incapable of self care. D- The dependent must be a citizen or resident of the U.S., Canada or Mexico.

Solution: The correct answer is D. The five dependency tests are: 1) Gross Income Test, 2) Support Test, 3) Member of Household or Family Member Test, 4) Citizenship Test (U.S., Canada or Mexico), and 5) Joint Filing Test. For the Citizenship Test: The claimed dependent must be a US citizen, national or resident of the United States or a resident of Canada or Mexico at some time during the calendar year in which the tax year of the taxpayer begins. In addition to these five tests, an individual must be claimed as a dependent on another person's return, like a parent.

Sam's Turbo Repair, Inc. (STR) purchased a new machine for cleaning and retooling the turbo blades on semi-trucks. The machine cost was $30,000, there was 10% sales tax, and $1,000 delivery and setup fee. What is STR's basis in the new machine? A- $30,000 B- $31,000 C- $33,000 D- $34,000

Solution: The correct answer is D. The machine cost of $30,000 plus the sales tax of $3,000 plus the $1,000 freight is the basis for depreciation.

Which of the following is/are more likely to result in the Internal Revenue Service challenging the reasonableness of compensation paid to a shareholder-employee? I- Compensation which is computed according to a contingent compensation agreement. II- Compensation paid by a company that has a history of no dividends. III- Compensation which is paid under a cost-of-living clause in an employment agreement. IV- Compensation paid by a company in a personal service business. V- Compensation paid by a company in a capital intensive business. A- IV only. B- I and III only. C- II and IV only. D- II and V only.

Solution: The correct answer is D. The question is directed to the issue of excessive compensation or compensation intended to forgo taxable income. Option "II" is suspect because dividends are not a deductible expense, and therefore, have no impact on lowering company income taxes. Option "V" is also suspect. The fact that the company is capital intensive indicates that it has a higher amount of revenue to expenses and a potentially high income tax burden. Both examples would be suspect in that the desire to reduce income tax requires the need to increase expenses, and therefore, gives rise to potential excessive or unreasonable compensation.

Ned, a college professor, owns a separate business in which he participates during the current year. He has one employee who works part-time in the business. Which of the following statements is correct? A- If Ned participates for 120 hours and the employee participates for 120 hours during the year, Ned does not qualify as a material participant. B- If Ned participates for 95 hours and the employee participates for 5 hours during the year, Ned probably does not qualify as a material participant. C- If Ned participates for 500 hours and the part-time employee participates for 520 hours during the year, Ned still qualifies as a material participant. D- If Ned participates for 600 hours and the part-time employee participates for 1,000 hours during the year, Ned nevertheless qualifies as a material participant.

Solution: The correct answer is D. The rules for material participation are: 1. More than 500 hours of participation 2. Taxpayer is the only one who substantially participates 3. Taxpayer spends greater than 100 hours in the tax year and no one else spends more 4. Taxpayer has materially participated in any 5 of the previous 10 years 5. The activity is a personal services activity and the individual has materially participated in any 3 prior years 6. Taxpayer participates 100 or more hours in this activity and total participation in all such activities exceeds 500 hours A is incorrect because he would be a material participant. The rule is > 100 hours and no one spends more. They can spend the same, but not more. (see rule #3) B is incorrect because he is the only one who substantially participates (see rule #2) C is incorrect because he needs to spend more than 500 hours or at least the same as the highest working person to be a material participant. (see rules #1, #3) D is correct because he spent more than 500 hours (see rule #1)

Freda purchased a stereo system for her son Wes, age 16. The stereo was placed in Wes' room and is used exclusively by him. Freda also purchased a new sports car in her own name, that was used 90% of the time by Wes. Which of the cost of these items may be considered as support in determining whether Freda may claim Wes as a dependent? A- Both the stereo and the car qualify as support because of the use test. B- Neither the stereo nor the car qualify as support because the car is Freda's and the stereo is diminimus. C- The stereo does not qualify for support but the car does because he uses it 90% of the time. D- The stereo qualifies for support, but the car does not even though it is de minimus.

Solution: The correct answer is D. The stereo system purchased and GIVEN to Wes qualifies as support. Because the car was not GIVEN to Wes (although he is allowed to use it) it will not be considered support. However, maintenance costs, such as gas and insurance that the taxpayer provides for his use of the car will qualify as support.

Pat invests $150,000 for a 10% interest in a limited partnership. He receives a K-1 with his loss at $80,000. How much of his loss is suspended? A- $0. B- $8,000. C- $15,000. D- $80,000.

Solution: The correct answer is D. To determine whether any of the losses are suspended you must first apply the at-risk rules, then the passive loss rules. The amount at risk is the basis of $150,000. Since the loss is less than the amount at risk, none of the loss is suspended due to the at-risk rules. In applying the passive loss rules, the passive loss is limited to the amount of passive income for the year. Since there is no passive income for the year, none of the loss may be recognized and the $80,000 loss is suspended. Note for the exam: If it does not state something in the question fact pattern, it does not apply. By definition, a limited partner is NOT an active participant. By definition, as general partner IS an active participant.

What difference does it make if a taxpayer's expenses are classified as un-reimbursed employee expenses rather than expenses from self-employment? I- Un-reimbursed employee expenses are subject to a hurdle of 2% of the taxpayer's AGI. They also require the taxpayer to itemize before the deduction can be taken. II- Expenses from self-employment are deducted above the line and have no AGI floor. III- Un-reimbursed employee expenses are not deductible. IV- Expenses from self-employment are subject to a floor of 2% of the taxpayer's AGI. They also require the taxpayer to itemize before the deduction can be taken. A- I & IV only. B- I only. C- I and II only. D- II and III only.

Solution: The correct answer is D. Un-reimbursed employee expenses are not deductible after 1/1/18 per the TCJA. Expenses from self-employment are deducted above the line and have no AGI floor.

As a direct result of the rules under TCJA 2017, qualifying dividends will be treated in which manner: A- Qualifying dividends are taxed in the same way as capital gains at an 18% rate for those in the 28% and higher marginal tax bracket. B- Qualifying dividends are taxed at a newly instituted 5% tax rate. C- The taxation on dividends have not been impacted under TCJA. D- Qualifying dividends are taxed at set dollar breakpoints.

Solution: The correct answer is D. Under prior law, the capital gain breakpoints were related to the tax breakpoints. Under TCJA the capital gain breakpoints are at set dollar amounts not corresponding to the current tax brackets. Qualified dividends are taxed at the capital gains rate tables. See provided tax tables posted in the Tax inflation Numbers section of online learning system.

Which of the following is a deductible loss? A- Losses on the sale of personal use assets. B- Losses on the subsequent sale of property gifted or sold to a related party when its fair market value is less than the original owner's adjusted basis. C- A wash sales transaction. D- Capital losses in excess of $3,000.

Solution: The correct answer is D. Up to $3,000 of capital losses may be recognized against ordinary income in any one tax year. However, if a taxpayer has capital losses in excess of $3,000, these losses are not disallowed, but are carried over indefinitely to future tax years. No loss can be taken on a personal use asset. Personal use asset might be your furniture, your car, etc. If a related party gives you property that is valued lower than what they paid for it, it is called loss property. If you then try to sell it, you can not take a loss. Wash sale is a sale of stock for a loss when an identical purchase was made within 30 days of the sale. The IRS will not allow a sale, for the purpose of taking a loss, to be repurchased again with in 30 days.

Five years ago, Jordan was granted a nonqualified stock option giving him the right to purchase 1,000 shares of employer stock at $8 per share. He exercised the option three years ago, when the value of the stock was $10 per share. He sold the shares today at a price of $15 per share. Which of the following statements is correct regarding this series of events? A- Jordan reported $8,000 of ordinary income upon exercise. B- Jordan will have a capital gain of $7,000 as a result of selling the stock today. C- Jordan reported an alternative minimum tax adjustment of $2,000 when he exercised the options. D- Jordan's employer received an income tax deduction of $2,000 when Jordan exercised the option.

Solution: The correct answer is D. When Jordan exercised the option, the bargain element was $2,000 [($10 Fair Market Value less $8 cost) x 1,000 shares]. The $2,000 will be reported as ordinary income by Jordan, and his employer will receive a $2,000 deduction. A is incorrect. Jordan was only required to report $2,000 of income. B is incorrect. Jordan's basis in the stock is $10,000 ($10 Fair Market Value x 1,000 shares). Therefore, his capital gain would be $5,000 ($15,000 sales price less $10,000 basis). C is incorrect. Alternative minimum tax does not apply to nonqualified stock options.

Mortimer is an avid collector of antiques associated with the funeral industry. The local hospital is running a campaign to redecorate and expand their lobby, and as a show of support, Mortimer donates a 19th century horse-drawn hearse in mint condition to the hospital. He purchased several of these hearses 30 years ago for $300 each, but the current estimated market value of the hearse today is in the range of $30,000. The hospital decides to sell the hearse and dedicate the proceeds to the renovation effort. Mortimer's AGI is $50,000. Which of the following statements concerning Mortimer's charitable deduction is correct? A- Mortimer's deduction for the current tax year will be limited to $25,000. B- Mortimer's deduction for the current tax year will be limited to $15,000. C- At least $5,000 of the deductible amount will have to be carried over to future tax years. D- Mortimer's income tax deduction is $300.

Solution: The correct answer is D. When tangible personal property donated to a charity will not be used by the charity to carry out its tax-exempt purpose, the deduction available to the donor is limited to the donor's cost basis and will be subject to the 50 percent limitation. Redecorating the lobby is not part of the hospital's tax-exempt purpose. In this case, Mortimer's cost basis is $300 and since 50% of his AGI is $25,000, Mortimer may take his entire charitable deduction this year. NOTE: While Mortimer purchased more than one, he only donated one.

Which of the following is the best reason to select a C corporation over an S corporation? A- Losses are expected during the first few years of the business's operations. B- The organization has significant earnings in excess of business needs. C- The owners of the business need income from the business for personal purposes. D- Various fringe benefits will be offered by the company to owner-employees.

Solution: The correct answer is D. While restrictions apply to various fringe benefits offered to more-than-2% shareholders of S corporations, a C corporation can offer fringe benefits without restriction. A is incorrect. A C corporation's losses do not flow through to the shareholders and are essentially trapped inside the C corporation. A S corporation's losses flow through to the individual shareholders and can be deducted by the shareholders within limits. B is incorrect. A C corporation that has earnings in excess of reasonable business needs will be subject to the accumulated earnings tax on the excess earnings if they are accumulated by the company. S corporations are not subject to the accumulated earnings tax. C is incorrect. A C corporation has double taxation of dividends. Therefore, if a business owner withdraws funds from a C corporation in the form of a dividend, the dividend will be double taxed. S corporations do not have double taxation of dividends.

Wanda is employed as a retail store manager. For the last calendar year, she had a $100,000 AGI and paid $7,600 in medical insurance premiums. During the year, she paid the following other medical expenses: Doctor and hospital bills for Bob and Sara (Wanda's parents) = $12,000 Doctor and dentist bills for Wanda = $6,400 Prescribed medicines for Wanda = $1,600 Non-prescribed insulin for Wanda = $700 Although Wanda paid more than 50% of their support, Bob and Sara did not qualify as Wanda's dependents because they have income that requires they file a joint return. Wanda's medical insurance policy does not cover them. Wanda filed a claim for $4,200 for her own expenses with her insurance company in December of last year. She received the $4,200 reimbursement this January. What is Wanda's maximum allowable medical expense deduction for last year? (Assume the prior year medical floor was 7.5% of AGI) A- $7,600 B- $28,300 C- $18,300 D- $20,800

Solution: The correct answer is D. You can include medical expenses you paid for an individual that would have been your dependent except that: He or she received gross income of $4,700 or more in 2023; He or she filed a joint return for 2023; or. You, or your spouse if filing jointly, could be claimed as a dependent on someone else's 2022 return. IRS Pub 502 (SEE BELOW). Medical expenses are an itemized deduction subject to a floor of 7.5% above AGI . The question here is whether non-prescribed insulin is a deductible medical expense. The answer is yes, non-prescribed insulin is deductible as a medical expense. The second issue is that the reimbursement was received this year and the question concerns last year's expenses. The total medical expenses are $28,300 less $7,500 ($100,000 AGI × 7.5%) = $20,800. The $28,300 in medical expenses comes from: Wanda is employed as a retail store manager. For the last calendar year, she had a $100,000 AGI and paid $7,600 in medical insurance premiums. During the year, she paid the following other medical expenses: Doctor and hospital bills for Bob and Sara (Wanda's parents) = $12,000 Doctor and dentist bills for Wanda = $6,400 Prescribed medicines for Wanda = $1,600 Non-prescribed insulin for Wanda = $700 FROM IRS PUBLICATION: You can include medical expenses you paid for your dependent. For you to include these expenses, the person must have been your dependent either at the time the medical services were provided or at the time you paid the expenses. A person generally qualifies as your dependent for purposes of the medical expense deduction if both of the following requirements are met. 1. The person was a qualifying child (defined later) or a qualifying relative (defined later). 2. The person was a U.S. citizen or national or a resident of the United States, Canada, or Mexico. If your qualifying child was adopted, see Exception for adopted child, later. You can include medical expenses you paid for an individual that would have been your dependent except that: 1. He or she received gross income of $4,700 or more in 2023; 2. He or she filed a joint return for 2023; or 3. You, or your spouse if filing jointly, could be claimed as a dependent on someone else's 2023 return.

Which of the following statements correctly reflects the automatic mileage method (as used to arrive at automobile expenses)? A- The rate includes parking fees and tolls. B- The method does not preclude the later use of MACRS on the same vehicle. C- The election of the automatic mileage method precludes a later change to the actual operating cost method. D- The method can be used on a car that is either purchased or leased by the taxpayer.

Solution: The correct answer is D. You can still deduct parking fees and tolls, MACRS is not permitted--nor is bonus depreciation such as 179--and you may switch between the standard mileage method and the actual operating cost method. The method does allow use on leased autos but the only exception is "If you want to use the standard mileage rate for a car you lease, you must use it for the entire lease period."

Stan and Susan, a married couple, recently sold their home in Massachusetts after living in the home for 19 years. They purchased the home for $225,000 and sold it for $850,000. They paid a 5% commission on the sale and used the proceeds to purchase a Winnebago for $600,000. Assuming they have a marginal tax rate of 24%, what is the amount of tax due related to the sale of the residence? A- $0. B- $7,000. C- $12,375. D- $19,800.

The amount of the commission paid was $42,500 ($850,000 x 5%). Therefore, the net proceeds received on the sale of the home by Stan and Susan was $807,500 ($850,000 - $42,500). Their realized and recognized gain is calculated as follows: Amount realized - $807,500 Less: adjusted basis - $225,000 Equals: realized gain - $582,500 Less: Exclusion of gain on sale - $500,000 Equals: taxable gain - $82,500 Since the home is considered a capital asset, the remaining taxable gain will be taxed at a long-term capital gain rate of 15%*. Therefore, the total tax due is $12,375 ($82,500 x 15%). *To determine capital gain rate, view the current tax table for the income level at the 24% rate and compare that to the capital gain table. Stan and Susan will be at the 15% cap gain rate.

Which of the following is not an administrative source of tax law? A: Revenue Ruling. B: Treasury Regulations. C: Decisions by the U.S. Tax Court. D: Technical Advice Memoranda.

C

Which of the following statements regarding cafeteria plans is not correct? A: A cafeteria plan must offer at least three nontaxable benefits. B: A cafeteria plan is a written plan under which the employee may choose to receive either cash or taxable benefits as compensation or qualified fringe benefits that are excludable from wages. C: Cafeteria plans are authorized by Section 125 of the Internal Revenue Code. D: A cafeteria plan is appropriate when employee benefit needs vary within the employee group.

Solution: The correct answer is A. A cafeteria plan must offer at least one taxable benefit, usually cash, and one qualified nontaxable benefit. All of the other statements regarding cafeteria plans are correct.

Two years ago, Green Corporation, a cash basis taxpayer, sold services to Albert for $25,000. During the prior year, Green collected $10,000 from Albert. In the current year, Green collected $5,000 from Albert in final settlement of the debt. The proper treatment for the bad debt deduction is: A- $0 for the prior year and $0 for the current year. B- $0 for the prior year and $10,000 for the current year. C- $15,000 for the prior year and $0 for the current year. D- $15,000 for the prior year and $5,000 income for the current year.

Solution: The correct answer is A. A cash basis taxpayer does not recognize income not received. Since the bad debt was never recognized as income, it cannot be recognized as a loss or a bad debt expense.

The Tax Reform Act of 1986 was roughly revenue neutral because: I: It was supported by both Republicans and Democrats. II: It was not intended to raise or lower tax revenues. III: It divided the tax burden evenly between individuals and businesses. IV: It made the tax rates equal across all tax brackets. A: 2 only B: 1 and 3 C; 2 and 3 D: 1, 2, and 4

Solution: The correct answer is A. A piece of tax legislation is considered revenue neutral when it is expected to neither raise nor lower the total amount of taxes to be collected.

Linus (age 64) and Karen (age 63) are raising their granddaughter, Marie, who qualifies as their dependent. Marie is blind. Linus and Karen file a joint return in the current year. What is their standard deduction? A- $27,700 B- $29,200 C- $29,550 D- $30,700

Solution: The correct answer is A. Linus and Karen's standard deduction is $27,700. They are not age 65 or older so they don't receive an additional standard deduction. They do not receive an additional standard deduction for Marie's blindness because additional standard deductions for age and blindness are allowed only for the taxpayer and spouse, and not for their dependents.

Jacob is divorced and has full custody of his two children although they spend every other weekend with their mother. How many personal exemptions is Jacob permitted on his Form 1040? A- 0 B- 1 C- 2 D- 3

Solution: The correct answer is A. Per the TCJA, personal exemptions are suspended from 1/1/18 through 12/31/2025.

What is the marginal income tax rate? A- The tax rate applied to the last dollar of taxable income earned. B- The lowest tax rate a taxpayer can pay. C- The tax rate on business profit (margin). D- The average tax rate for an individual.

Solution: The correct answer is A. The marginal tax rate is the tax rate applied to the last dollar earned

Under MACRS, if the mid-quarter convention is applicable, all property is treated as being placed in service during the fourth quarter. A: True B: False

Solution: The correct answer is B The first-year depreciation deduction will be calculated as if each asset purchased during the year was placed in service at the mid-point of the quarter in which the property is actually placed in service (versus when it was actually purchased).

Which of the following is a deduction from AGI (itemized deduction)? A; Contribution to a traditional IRA B: Roof repairs to a rental home C: Mortgage Interest D: Alimony payment made under a contract dated 12/1/13

Solution: The correct answer is C. A, B, and D are deductions FOR AGI.

Mr. Rangel files a fraudulent income tax return (again) and has a $10,000 income tax deficiency because of it. What is the amount of his penalty? A- $10,000 B- $8,000 C- $7,500 D- $5,000

Solution: The correct answer is C. The penalty for filing a fraudulent income tax return is 75% of the deficiency.

Which of the following is a deduction from AGI? A- A contribution to a deductible IRA. B- Major roof repairs to a rental home. C- Real estate taxes of $6,000 on your residence D- Alimony payments made to a former spouse.

Solution: The correct answer is C. The question is looking for the below the line deductions (FROM AGI). State and local income taxes or sales tax and real estate taxes up to $10,000 are part of itemized deductions, and therefore, deducted FROM AGI (below the line). IRA contributions (when deductible), and alimony paid (for divorces prior to 12/31/18 based on TCJA) are deductions FOR AGI (above the line). Major roof repairs on rental property are deducted FOR AGI on Schedule E.

In year 1 Justin earns $700 from delivering papers for a newspaper company and he is treated as self-employed. In year 2 the newspaper company hires him as an employee and pays him $700 as W-2 income with no federal or state income tax withholding. Does Justin have to file a tax return in either year? A- Year 1: Yes Year 2: Yes B- Year 1: No Year 2: Yes C- Year 1: Yes Year 2: No D- Year 1: No Year 2: No

Solution: The correct answer is C. The rule is that a taxpayer must file if he has greater than or equal to $400 of net earnings from self-employment. If the taxpayer does not have self-employment income there is no requirements to filing unless your income exceeds the standard deduction and personal exemption ($0 for 2018 - 2025) for that year.

Drew recently purchased a new machine for his business. The price of the machine was $12,000. Drew also paid $100 in sales tax, $300 in freight, and $1,000 in installation and testing costs. What is Drew's basis in the new machine? A- $12,100. B- $12,400. C- $13,000. D- $13,400.

Solution: The correct answer is D. All costs to get the asset into operation are included in the cost basis of the asset. Therefore, Drew's cost basis includes the price of the machine, sales tax, freight, and installation and testing costs.

Last year Dan Walker had a rough year in his taxable brokerage account. He sold when he should have bought, he held when he should have sold and he bought just before prices plunged in most instances. Still he managed to eke out $27,000 in short term capital gains, with only $25,000 of short term capital losses. He also had another $2,000 of investment expenses (research reports). All other considerations aside, he had an AGI of $50,000, so what are the implications of Dan's activities? A- Dan may realize a $2,000 short term gain and deduct $2,000 as a miscellaneous itemized deduction. B- Dan may deduct $25,000 as short term losses and deduct $2,000 as expenses. C- Dan may realize a $25,000 passive activity loss and deduct $2,000 as itemized deduction. D- Dan must recognize

Solution: The correct answer is D. He gets to net his portfolio losses against his portfolio gains. He will not be able to deduct his expenses.

John took out a loan of $40,000 from his home equity line of credit account, and used the proceeds to pay for his daughter's wedding. Which of the following statements is correct regarding the regular income tax and alternative minimum tax (AMT) consequences of the interest expense paid by John on the HELOC this year? A- The interest expense is deductible for regular tax purposes and deductible for AMT purposes. B- The interest expense is not deductible for regular tax purposes but is deductible for AMT purposes. C- The interest expense is deductible for regular tax purposes but not deductible for AMT purposes. D- The interest expense is not deductible for regular tax purposes and not deductible for AMT purposes.

Solution: The correct answer is D. Interest from a home equity loan or HELOC is not deductible for regular tax purposes, unless the proceeds are used to purchase or improve a residence.

Which of the following types of exchanges of insurance contracts qualify for nonrecognition treatment under § 1035? A: Exchange of life insurance contracts B: Exchange of a life insurance contract for an endowment or annuity contract C: Exchange of an endowment contract for an annuity contract D: A, B and C

Solution: The correct answer is D. Life insurance can transfer tax-free to another life insurance contract, Endowments or Annuities.

Which of the following income(s) is/are NOT taxed under Social Security self-employment tax? I- Income from personal property leased with real estate. II- Income from debentures and investment notes. III- Gain from the sale of property not held for sale in a trade or business. IV- Income of a person working as a private consultant who is over normal age retirement. A- All of the above. B- I and III only. C- II and IV only. D- I, II, and III only.

Solution: The correct answer is D. Option "I" - Income from personal property leased with real estate is not subject to self-employment taxes. Option "II" - This implies there is debt involved. As such this income is not subject to self-employment taxes. Option "III" - This is not earned income and not subject to self-employment taxes. Option "IV" - Income from work as a private consultant is subject to self employment tax regardless of age.

Under the accrual method of accounting, the taxpayer (seller) recognizes income when: A- The bill is received by the buyer. B- The goods are accepted by the buyer. C- The goods are loaded on the truck at the seller's facility. D- The seller writes and sends the invoice after sending the goods.

Solution: The correct answer is D. The accrual accounting method recognizes income when the taxpayer has a right to collect. This occurs usually after the completion of a job and in no case later than when the invoice is prepared and sent.

Maria and Orlando are married taxpayers in their 30s, who file their income tax return jointly. They have asked you to explain to them how the current law will affect their 2023 tax filing. You tell them: I- Their standard deduction is $27,700. II- Their standard deduction is 200% of a single tax filer. III- The maximum taxable income in the 22% bracket for MFJ filers will be twice the amount as for single filers. A- I only. B- I and II only. C- II and III only. D- I, II and III.

Solution: The correct answer is D. The deduction for joint filers is 200% of a single tax filer. Single standard deduction = $13,850 x 200% = $27,00 which is the MFJ standard deduction. Single 22% tax bracket is $44,725 - $95,375 and if you multiply by 200% = the MFJ 22% bracket of $89,450 - $190,750.

Avery owns a condo on the beach in California that he rents out each year. The rental income for the current year is $178,000 from the 206 days it was rented. Avery spent 30 days at the property during the year cleaning and performing maintenance activities. If rental expenses are $60,000, what is the net income reported on Avery's 1040 for the rental activity? A- $178,000 B- $144,137 C- $125,627 D- $118,000

Solution: The correct answer is D. The time Avery spent at the condo does not count as personal days since he was there to "clean and perform maintenance." If he was there to relax then they would be personal days. The full $60,000 is allowed as a deduction from the income of $178,000 to equal $118,000 of income on the 1040.

Which is the only court that allows a jury trial? A: Appropriate U.S. Circuit Court of Appeals B: U.S. District Court C; U.S. Tax Court D: U.S. Court of Federal Claims

B

The election to itemize is appropriate when total itemized deductions are less than the standard deduction based on the taxpayer's filing status. A: True B: False

Solution: The correct answer is B. The election to itemize is appropriate when total itemized deductions exceed the standard deduction based on the taxpayer's filing status.

Which of the following can be claimed as a deduction for AGI? A: Personal casualty losses B: Investment interest expenses C: Self-employment tax D: Property taxes on personal use real estate

Solution: The correct answer is C. One half of the self-employment is an adjustment to gross income.

The adoption expense credit for 2023 is limited to: A- $5,800 B- $10,810 C- $15,950 D- $20,040

Solution: The correct answer is C. The adoption expense credit limit for 2023 is $15,950.

Which of the following is not one of the five types of filing status? A- Single. B- Head of household. C- Qualified dependent child. D- Married filing jointly.

Solution: The correct answer is C. The five types of filing status are: Single, Married filing jointly, Married filing separately, Qualifying widow/widower with dependent child, and Head of household.

Refundable tax credits include the: A- Foreign tax credit. B- Tax credit for rehabilitation expenses. C- Disabled access credit. D- Earned income credit.

Solution: The correct answer is D. Earned income tax credit is refundable. The remainder of the choices are not refundable credits.

During the current year, Susan Smith accepted and received a $10,000 award for outstanding scientific achievement. Susan was selected without any action on her part and no future services are expected of her as a condition of receiving the award. What amount if any will Susan have to include in her gross income in connection with the receipt of this award? A- $0 B- $2,500 C- $5,000 D- $10,000

Solution: The correct answer is D. Prizes and awards are fully taxable when received.

The short-term and long-term holding periods, respectively, are: A: Eighteen months or less and more than eighteen months. B: Twenty-four months or less and more than twenty-four months. C: Six months or less and more than six months. D: Twelve months or less and more than twelve months.

Solution: The correct answer is D. The short-term holding period is one year or less and the long-term holding period is more than one year.

In the current year, Tom and Tammy Williams, a married couple, file a joint federal income tax return. During the current tax year, the Williams made deductible IRA contributions of $3,000. They are in a 24% marginal income tax bracket. What amount of tax savings is generated by this deduction? A- $720 B- $2,190 C- $3,000 D- $3,846

Solution: The correct answer is A. $3,000 multiplied by the 24% marginal income tax bracket equals $720.

Kenny has been a night watchman at Burgundy Company for 10 years. During the current year, he received the following benefits from Burgundy Company: Salary = $15,000; Hospitalization premiums = $3,600; Required lodging on Burgundy's premises as a condition of Kenny's employment = $2,400; Reward for preventing a break-in = $1,000. What amount is includible in Kenny's adjusted gross income for the current year? A- $16,000 B- $17,400 C- $18,400 D- $22,000

Solution: The correct answer is A. $16,000 ($15,000 salary + $1,000 reward). Hospitalization premiums paid by the employer and any other benefit required by the employer for the employer's benefit are not included in the employee's AGI income.

Alice purchased a business asset (three-year property) this year, at a cost of $100,000. This is the only asset she purchased during the year. Alice did not elect to expense any of the asset under Section 179, nor did she elect straight-line cost recovery. Determine the cost recovery deduction for her tax filing this year of purchase. A- $33,333 B- $25,000 C- $50,000 D- $66,667

Solution: The correct answer is A. 1/2 year convention applies in this case. This is 3 year property, so 33.3% is depreciated per year. Using double declining: $100,000 × .3333 = $33,330 Year 1: 200% declining balance 1/2 year convention with 33.33% × 200% = 66.66% × 1/2 year convention = 33.33% × 100,000 = $33,333 Year 2: $100,000 - $33,333 = $66,667 33.33% × 200% = 66.66% × $66,667 = $44,440

Paul (age 35) and his wife Stacey (age 33) are married with three young children. They both work outside the home. Paul is a corporate executive with Wellstar and Stacey is an executive assistant with a small local company. Paul fully participates in his company's qualified retirement plan by contributing $22,500 of his salary, which is matched 100% up to 3% of compensation. Stacey's employer does not offer a retirement plan. In addition, during the year they had the following items of income and expense: Paul's gross salary: $150,000 Stacey's gross salary: $32,000 Stacey's cash gift to her mother: $5,000 Interest from a joint savings account: $100 Federal income taxes withheld from paychecks: $30,000 State income taxes withheld from paychecks: $12,000 Charitable contributions made: $3,400 State and Local Taxes: $24,000 Mortgage interest paid: $24,800 Contribution to Paul's traditional IRA: $6,500 Contribution to Stacey's traditional IRA: $6,500 What is Paul and Stacey's adjusted gross income? A- $153,100 B- $159,600 C- $176,100 D- $182,000

Solution: The correct answer is A. 150,000 Paul's gross salary +32,000 Stacey's gross salary 182,000 + 100 Interest 182,100 -22,500 His 401K 159,600 -6,500 Her IRAs 153,100 AGI

AMT adjustments can be positive or negative, whereas AMT preferences are always positive. A: True B: False

Solution: The correct answer is A. A tax preference increases AMTI, but an adjustment can increase or decrease AMTI.

Abner and Bette have been married for 20 years and have always filed a joint return. They never itemize their deductions. Abner and Bette have gross income of $80,000 and deductions for adjusted gross income in the amount of $5,000, and they do not have any children. Neither Abner nor Bette are over the age of 65 and neither is blind. What is Abner and Bette's taxable income in 2023? A- $47,300 B- $52,300 C- $55,800 D- $75,000

Solution: The correct answer is A. Abner and Bette's taxable income is equal to their gross income less deductions for adjusted gross income less the greater of the standard deduction or itemized deductions, and less personal and dependency exemptions (currently suspended through 2025 by TCJA). Therefore, their taxable income can be calculated as follows: Gross Income $80,000 Less Deductions for AGI ($5,000) Adjusted Gross Income $75,000 Less Standard Deduction ($27,700) Taxable Income $47,300

Petra Walenski purchased a personal residence for $166,000, and insured it for full replacement value. It had a fair market value of $180,000 when it was damaged by fire. The fair market value after the fire was $140,000, and Petra received insurance proceeds of $15,000. What is the net amount of casualty loss that Petra can deduct if her adjusted gross income is $78,000? A- $0 B- $1,000 C- $17,100 D- $24,900

Solution: The correct answer is A. After 12/31/17, TCJA eliminated deductions for personal casualty losses except where Federal disaster's were declared, so the deduction here is $0. Had it been a federal disaster, the calculation would be as follows: The loss is the $40,000 decline in value, reduced by the $15,000 of insurance proceeds, $100 non-included amount (IRS $100 floor) for a total reduction of $24,900. TCDTR Act of 2020 changed the casualty loss formula. It still only applies to federally declared disasters.

Which statement is true with respect to private letter rulings? A: They cover facts applicable to a particular taxpayer. B: They deal with completed transactions. C: They are not binding on the IRS. D: They are issued at the request of the IRS.

Solution: The correct answer is A. All of the statements except A are false regarding letter rulings. Private letter rulings cover facts applicable to a particular taxpayer (option A), are issued at the request of the taxpayer (option D), deal with proposed transactions (option B), and binding on the IRS with respect to the requesting taxpayer and the particular transaction (option C).

Income to U.S. taxpayers is taxed in the year it is derived in which of the following situations? I- Interest earned but reinvested in a savings account in an FDIC savings bank. II- Unrealized long-term capital gains on stocks. III- Income earned on most municipal bonds. IV- Short-term gains realized within a qualified plan. V- Increased value of personal residence. A- I only. B- I and III only. C- II, III and IV only. D- II, III, IV and V.

Solution: The correct answer is A. All others are forms of legal tax deferral or tax exemption. Option I. Interest earned but reinvested in a savings account in an FDIC savings bank would be taxable in the year it was derived and paid. Option II. Unrealized gains are not taxed in the year they are derived, they are taxed when the position (or shares of the position) are sold. III. Since bonds pay in arrears and that may split tax years, this statement does not match what the question is looking for. Bonds that pay in April include interest from the prior 6 months which includes income derived from the prior year. IV. Gains in a QP are taxed as distribution. V. Increased value of personal residence would be taxed in the year the residence was sold if the gain exceeded the section 121 exclusion.

Hansel and Gretel, a married couple, manage apartments and they are required to live in the managers' apartment as a condition of their employment. Instead of providing the apartment to Hansel and Gretel rent-free, the owner of the apartment building gives Hansel and Gretel a housing allowance of $600, which they use to pay rent on the managers' apartment. Hansel and Gretel pay $600 per month in rent. If they did not live in the managers' apartment, Hansel and Gretel could live in another apartment building where they would only pay $500 in rent. What amount, if any, must be included in Hansel and Gretel's gross income? A- $0. B- $100. C- $500. D- $600.

Solution: The correct answer is A. An employee is allowed to exclude from gross income the value of lodging furnished by an employer to the employee if the lodging is furnished (1) on the employer's business premises, (2) for the convenience of the employer, and (3) the employee is required to accept the lodging as a condition of employment. It does not matter that Hansel and Gretel were paid a housing allowance, which they were then required to pay back to the employer for rent. Hansel and Gretel can exclude the entire value of their housing from their gross income.

Robin, a senior at State University, receives free room and board as full compensation for working as a Resident Advisor, and is responsible for a floor of residents at the university dormitory at all hours of the day and night. The regular housing contract is $1,400 a year total, $800 for lodging and $600 for meals in the dormitory. What is Robin's gross income from this employment? A: $0, the entire value of the contract is excluded, assuming the meals are provided for the convenience of the employer. B: $600, the meal contract must be included in gross income. C: $800, the lodging contract must be included in gross income. D: $1,400, the entire value of the contract is compensation.

Solution: The correct answer is A. As an R.A., you are required to live in the dorm, since the meals and lodging are provided for the convenience of the employer. There is no taxable income.

During the year, Rick had the following insured personal casualty losses (arising from a tornado, a Federally declared disaster). Rick also had $18,000 AGI for the year. AssetAdjusted BasisFMV BeforeFMV AfterInsurance RecoveryA$500$700$300$100B$3,000$2,000$0$500C$700$900$0$200 Rick's casualty loss deduction is: A: $400 B: $600 C: $1,000 D: $1,400

Solution: The correct answer is A. Asset A: Change in FMV is 700 - 300 = 400. The lesser of ATB or decline in FMV is $400 - insurance = $300 Asset B: Change in FMV is 2,000 - 0 = 2,000. The lesser of ATB or decline in FMV is $2,000 - insurance = $1,500 Asset C: Change in FMV is 900 - 0 = 900. The lesser of ATB or decline is $700 - insurance = $500. Total: $2,300 Less: Statutory floor (100) Less: AGI limitation (10% × $18,000) (1,800) Casualty loss deduction 400 Remember the deduction is the lesser of ATB, or decline in FMV, less insurance proceeds. Reminder, ONLY Federally declared disasters are eligible for casualty loss treatment after 12/31/17.

Ted cashed in his life insurance policy when he found out he had a terminal illness. He had paid $15,000 in premiums and collected $50,000 from the insurance company. Ted is required to include the proceeds in gross income. A; True B: False

Solution: The correct answer is A. Because Ted cashed in (surrendered) his policy, rather than electing the nontaxable accelerated death benefit he will receive the insurance proceeds of $50,000 as taxable income.

Which of the following best describes depreciation cost recovery? A- It is periodic expensing of tangible property, including real and personal property used in business. B- It is periodic expensing of the cost of intangible and tangible assets. C- It is the periodic expensing of natural resources, tangible and intangible assets, as they are being used up. D- It is an expense that fluctuates with the actual taking of the resource from the land.

Solution: The correct answer is A. Cost recovery is a periodic expensing of tangible property, including real and personal property used in business. Amortization is a periodic expensing of the cost of intangible assets. Depletion is the expensing of natural resources as they are being used up. This expense will fluctuate with the actual taking of the resource from the land.

Peyton has a warehouse used in his business. He exchanges it for a storage building owned by Eli. (Peyton and Eli are unrelated). The basis of Peyton's asset is $40,000 and he gives Eli $20,000 cash plus the asset in exchange for Eli's asset, which is worth $36,000. Eli's basis in his original asset is $10,000. What is Eli's gain or loss? A- $20,000 gain recognized. B- $26,000 gain realized and recognized. C- $0 gain recognized. D- $0 loss recognized.

Solution: The correct answer is A. Eli must recognize gain to the extent of the boot paid to him ($20,000) limited by his potential gain. Note: only real property is eligible for like kind treatment. You need to calculate Peyton's FMV. You can do this because you know that the values exchanged must be equal. Peyton gives Eli $20k + property in exchange for a property worth $36,000: $20,000 + x = $36,000 x = $16,000

Reagan purchased stock in XYZ Corporation three years ago for $110,000. Today it is worth $100,000. Reagan exchanges (sells) the stock to a local charity for $10,000. What is Reagan's capital loss on the transaction? A- $0 B- $10,000 C- $100,000 D- $3,000

Solution: The correct answer is A. For tax purposes, bargain sale transactions (selling to an unrelated party at a price well below FMV) cannot generate capital losses. Answer B, C, & D are incorrect because this is a bargain sale, and no amount of loss is permitted.

Debra is transferred by her employer from Denver to Seattle. Her expenses are not reimbursed and are as follows: Cost of moving household furniture = $2,400 Transportation = $400 Meals = $420 Lodging = $350 Her qualified moving expenses are: A- $0 B- $2,800 C- $3,150 D- $3,570

Solution: The correct answer is A. For years after 12/31/17, qualified moving costs are not deductible and if reimbursed by employer, they must be included in income.

Frank's automobile, which is used exclusively in his business, was damaged in an accident. The adjusted basis prior to the accident was $8,000. The fair market value before the accident was $10,000 and the fair market value after the accident is $500. Insurance proceeds of $10,000 less his $2,000 deductible are received by Frank. What can Frank deduct as a result of the accident? A- $0 B- $1,500 C- $8,000 D- $9,500

Solution: The correct answer is A. Frank is not entitled to any "deduction" since the net insurance proceeds were equal to his adjusted taxable basis. For the tax years 2018 - 2026, deductions for personal casualty and theft losses are suspended by the TCJA of 2017. Business deductions for casualty and theft have remained (to the extent they exceed insurance reimbursement).

Noah has been working part-time through college and earned $20,000 last year with a total federal income tax liability of $1,200. This year he will earn $100,000 with an expected income tax liability of $15,000. What is the lowest amount of tax withholding Noah should have to meet the safe harbor rules? A- $1,200 B- $12,000 C- $13,500 D- $15,000

Solution: The correct answer is A. He has two choices: 100% of last year's tax liability or 90% of this year's tax liability. Last year's income tax liability was $1,200 and 90% of this year is $13,500. Therefore, the lowest amount that of tax withholding that needs to be met is $1,200. 110% for prior year tax safe harbor only applies if AGI is over $150,000.

Kevin owns a modified endowment contract. Kevin recently reassessed his insurance needs and decided that he would like to exchange his current modified endowment contract for a different insurance product. Which of the following transactions might result in gain realization and recognition? A- If Kevin trades his modified endowment contract for a life insurance policy. B- If Kevin trades his modified endowment contract for a different modified endowment contract. C- If Kevin trades his modified endowment contract for an annuity. D- None of the above transactions would result in the realization and recognition of gain.

Solution: The correct answer is A. If Kevin trades his modified endowment contract for a life insurance policy, the transaction will not be eligible for the deferral of gain under Section 1035 of the Internal Revenue Code.

All of the following statements about the Alternative Minimum Tax (AMT) are correct, EXCEPT: A- A taxpayer may elect to pay tax based on the AMT calculation if it produces a lower tax result. B- The AMT is designed primarily to change the timing of tax payments to more current tax payments. C- The AMT frustrates efforts by taxpayers to participate in activities that reduce or eliminate their current tax liability. D- Some adjustments made for AMT purposes result in a permanent increase in tax.

Solution: The correct answer is A. If the calculated tax due is greater under the AMT, the taxpayer must pay the higher amount. The AMT is not a voluntary alternative to the regular tax system. It is a mandatory alternative, and applies only when the AMT tax exceeds the regular tax imposed on the taxpayer.

Abby and Brock are divorced. They have one daughter, Caroline, who spends exactly six months of the year living with her mother and exactly six months of the year living with her father. They split all expenses evenly. Abby has an AGI of $200,000 and Brock has an AGI of $150,000. Caroline is a qualifying child of both Abby and Brock. Who can claim Caroline as a dependent? A- Abby. B- Brock. C- Both Abby and Brock. D- Neither Abby nor Brock.

Solution: The correct answer is A. In this case, both parents are eligible to claim Caroline as a dependent because she is a qualifying child for both of them. However, an individual cannot be claimed as a dependent by more than one person. Because Caroline lives with each parent for the same amount of time, the parent with the higher AGI is allowed to claim the dependency exemption. In this case, Abby has a higher AGI than Brock, so she is allowed to claim a dependency exemption for Caroline. This rule can be changed by signed agreement. For those over analyzing this, 365 days divided by two is 182.5 days, they switch at noon on the half day.

If an individual who may otherwise qualify as a dependent does not spend funds that he or she has received (i.e., social security, wages), what is the IRS position regarding these unexpended amounts in terms of their application to the support test and their inclusion in being applied to the gross income test? A- Income received but not spent is applicable to the gross income test but not the support test. B- Income received but not spent is not applicable to the gross income test nor to the support test. C- Income received but not spent is applicable to the gross income test and to the support test. D- Income received but not spent is not applicable to the gross income test but is applicable to the support test.

Solution: The correct answer is A. Income received but not spent is applicable to the gross income test but not the support test. To claim someone as a dependent you have to provide at least 50% of their support. If the child has income that just goes into savings or spent on miscellaneous fun items instead of being used toward paying their bills (housing, clothing, food, etc.), while you are paying their bills, then you are supporting them. The income counts as gross income for the child, but it is not support unless they use it to support themselves.

Franco and Giada are trying to calculate their gross income for the current year. Which of the following items should they exclude from their gross income? I- $60,000 in cash inherited by Giada from her mother. II- $20,000 borrowed by Franco and Giada from First City Bank. III- $12,000 gain from the sale of Franco and Giada's boat. IV- $400 of interest earned on a loan made by Franco to his cousin Vinnie. A- I and II only. B- III and IV only. C- I, II and III only. D- I, II and IV only.

Solution: The correct answer is A. Inherited cash or property is excluded from gross income; therefore, option "I" is correct. Borrowed money is also excluded from gross income; therefore, option "II" is also correct. Gain on the sale of assets (option "III") and interest income (option "IV") are both included in gross income.

During the current tax year, Sam Malone had $10,000 of passive income from a publicly traded limited partnership. He also has a non-publicly traded limited partnership which generated a $10,000 passive loss. How much of the passive loss is deductible by Sam during the current tax year? A- $0 B- $1,000 C- $3,000 D- $10,000

Solution: The correct answer is A. Sam will not be able to deduct the loss in the current tax year. Passive Income from a publicly traded limited partnership may not be offset by any other passive losses. The Passive Income from a publicly traded LP would need to be offset by a prior suspended loss from the publicly traded LP.

Sammy owned a home in south Florida that was severely damaged by a hurricane. Sammy had purchased the home for $150,000, and the fair market value of the home prior to the hurricane was $500,000. His homeowners insurance policy had lapsed one month before the hurricane hit and Sammy had not obtained any other insurance. After the hurricane, the property had a fair market value of $100,000. Assuming that Sammy's AGI was $115,000 this year, what is Sammy's casualty loss deduction? A- 0 B- $38,100 C- $138,400 D- $400,000.

Solution: The correct answer is A. Sammy's had a personal casualty loss, under TCJA personal casualty losses are not deductible unless the area was declared a federal disaster area.

Early this year, Monique took a trip from Boston to Rome. She was away from home for 15 days. She spent five days vacationing and ten days on business for her clothing line (including the two travel days.) Her expenses are as follows: Airfare = $2,100; Lodging (15 days) = $3,150; Meals (15 days) = $2,400; Valet service (cleaning of business suits) = $60. Monique's business travel expenses deduction is: A- $4,360 B- $5,060 C- $5,160 D- $5,860

Solution: The correct answer is A. She owns her business and can take a business deduction. If she was an employee of the company it would be zero under the TCJA suspension of itemized deduction subject to the 2% floor. Airfare Deductibility Rules: DOMESTIC If primarily business then deduct all airfare. Prorata meals and lodging. FOREIGN Prorata meals and lodging. Prorata airfare unless (then you can deduct all): < 7 days <25% on personal No control Vacation not a deciding factor Airfare = $2,100 × (10 ÷ 15) = $1,400 Lodging = $3,150 × (10 ÷ 15) = $2,100 Meals = $2,400 × (10 ÷ 15) = $1,600 ÷ 2 = $800 (meals are only 50% deductible) Valet = $60 Total = $4,360 Note: NOW EXPIRED.... for tax years 2021 and 2022, meals are 100% deductible.

John owns a rental home in Arizona. He decided that he would like to acquire a rental home in Washington. Ted who lives in Washington has a rental home. For health purposes, Ted must relocate to Arizona. John and Ted decide to exchange properties under section 1031 of the code. The other facts pertaining to the exchange are: Ted's Basis = $100,000 John's Basis = $75,000 Ted and John exchange the two properties, but Ted has to give John an additional $25,000 in cash. The fair market value of Ted's property is $100,000, and the fair market value of John's property is $125,000. What is John's basis in the property received in the exchange? A- $75,000 B- $50,000 C- $25,000 D- $100,000

Solution: The correct answer is A. Since John received boot and "traded down" his recognized gain equals the lesser of the realized gain or boot received, which is $25,000. John's basis will be his original basis ($75,000) less the boot received ($25,000) plus the gain recognized ($25,000) or $75,000. Ted's basis will increase by $25,000 because he is paying $25,000 in boot.

Which of the following statements regarding depreciation is/are correct? I- Real estate used for residential rental purposes is depreciated on a straight-line basis over 27 1/2 years. II- Real estate used for commercial purposes is depreciated on a straight-line basis over 39 years. III- Both the land and the value of the improvements on the land can be depreciated. IV- A mid-year convention is used in the depreciation of real property. A- I and II only. B- I, II and III only. C- I, II, and IV only. D- I, II, III, and IV.

Solution: The correct answer is A. Statement "III" is incorrect because only the value of the improvements on the land can be depreciated. The value of the land itself cannot be depreciated. Statement "IV" is incorrect because a mid-month, not a mid-year convention is used in the depreciation of real property.

Expenses incurred in a trade or business are deductible for AGI. A: True B:False

Solution: The correct answer is A. Such expenses are deductible for AGI.

For the purposes of Earned Income Credit, which loss/losses are disregarded? I- Net losses from estate and trusts. II- Net losses from non-business rents. III- Net capital losses IV- Taxable interest on U.S. government issue securities. A- I, II and III only. B- I, II, III and IV. C- II and III only. D- IV only.

Solution: The correct answer is A. Tax has already been paid on federally issued securities, while the rest are all disregarded.

Mrs. Eagleton is in the 35% tax bracket and her child just started stocking shelves at the local grocery store on the weekend. His total earnings from the grocery store are $2,200 for the taxable year. What is the amount of tax owed on the son's income? A- None B- $95.00 C- $332.50 D- $770.00

Solution: The correct answer is A. The $2,200 is earned income. Earned income is not subject to kiddie-tax. His standard deduction is the greater of $1,250 or his earned income plus $400, not to exceed $13,850 in 2023. Therefore his standard deduction is $2,600, which eliminates his income so he owes no tax.

Under the terms of a divorce agreement dated 1/3/18, Larry is required to pay his wife Joyce $2,100 in support per month for a minimum period of 10 years and $300 per month in child support. For a twelve-month period, Larry can deduct from gross income (and Joyce must include in gross income): A- $0 B- $25,200 C- $3,600 D- $28,800

Solution: The correct answer is A. The $300 per month for child support is not deductible by Larry. Child support payments are not deductible to the payor nor includable to the payee. Larry's $300 per month in child support will remain part of his gross income. The $2,100 is not alimony since it could extend beyond her death as the required payment is for a minimum of 10 years. As a requirement for alimony, it can't extend beyond the life of the payor or payee. Here, it stated is a minimum of 10 years, but what if the payor or payee died in year 4, then the agreement would still require payment, rendering it non-deductible. Note: As planners, your client may state a payment is alimony, it is up to you to know what qualifies and what does not qualify as alimony to properly advise your clients.

Veronica has determined that she needs to make a 4th quarter federal estimated income tax payment. When is this payment due? A- January 15th of next year. B- December 31st of this year. C- December 15th of this year. D- September 15th of this year.

Solution: The correct answer is A. The 4th quarter federal income tax estimated payment is due by January 15th of the year following the year the payment is being made for.

MyLei, who is a sole proprietor, is going to give her best customers gifts as a token of her appreciation. If MyLei wants to deduct the full cost of the gifts, what is the maximum amount she may spend on each customer's gift? A- $25 B- $50 C- $100 D- $250

Solution: The correct answer is A. The maximum allowable expense for business related gifts is $25 per customer.

Howard is 53 years old and has decided to purchase Long-term Care Insurance. Which of the following most accurately describes the tax benefits of premiums paid on a long term care policy? I- The policy must be guaranteed renewable or non-cancelable for the premiums to be deductible. II- Since Howard is less than age 62, only 7.5% of premiums paid are deductible. III- Premiums paid are deductible but limited based upon age. IV- The long term care insurance deduction is for AGI. A- I and III only. B- III only. C- II and III only. D- I, II, III and IV.

Solution: The correct answer is A. The IRS provides guidelines for the amount of premiums that are deductible based upon the insured's age. The amount of premiums paid is included in the medical expense deduction for total expenditures exceeding 7.5% of AGI and is from AGI. The policy must be guaranteed renewable or non-cancelable to be qualified.

James, a cash basis taxpayer has the following compensation and fringe benefits from his employer in the current year: Salary = $60,000 Bonus for services = $8,000 Premium for disability income protection = $2,000 Premium for group hospitalization insurance = $2,500 The bonus will be paid in January of next year. The wage continuation insurance would pay James three-fourths of his regular salary if he became disabled. The insurance benefits are provided by the employer to all full-time employees. What is James' gross income in the current year? A- $60,000 B- $62,000 C- $62,500 D- $68,000

Solution: The correct answer is A. The premiums for disability insurance and group hospitalization insurance are not included in the taxpayers income. Because James is a cash basis taxpayer, income is recognized when received. So the salary is $60,000 in the current year and though he earned his bonus in the current year, he will not recognize it until next year when it is paid.

Five years ago, Bailey purchased 200 shares of Circus Clowns, Inc. for $4,000. Bailey recently gifted those shares of stock to his son Barnum. He paid gift tax of $700. The value of the 200 shares of stock on the date of the gift was $2,000. Which of the following statements is correct? A- If Barnum subsequently sells the shares of Circus Clowns, Inc. for $4,750, the basis used to calculate his gain or loss will be $4,000. B- If Barnum subsequently sells the shares of Circus Clowns, Inc. for $4,750, the basis used to calculate his gain or loss will be $1,000. C- If Barnum subsequently sells the shares of Circus Clowns, Inc. for $1,600, the basis used to calculate his gain or loss will be $4,000. D- If Barnum subsequently sells the shares of Circus Clowns, Inc. for $1,600, he will not have any gain or loss.

Solution: The correct answer is A. This is a gift of loss property. Gift tax is only allocated to appreciation. Barnum will have a double basis in the stock, determined as follows: First establish basis. Since the sale price is above the donor's basis, that become Barnum's basis. Next step in all tax calculations is determine gain or loss from basis. In this case Barnum has a gain.

Britney owned an office building in Los Angeles that she rented out to several production companies. The building was destroyed by a fire and was a complete loss. Britney received a settlement from her insurance company and would like to reinvest in a new property. Britney wants to make sure that she is eligible for nontaxable exchange treatment. Which of the following is not correct regarding the requirements for nontaxable exchange treatment on Britney's transaction? A- Because Britney rented out the building instead of using the property directly, the replacement property must meet the functional use test. B- Britney must invest the proceeds in a replacement property that has a similar use to the property that was destroyed in the fire. C- Britney must reinvest the insurance proceeds within two years from the end of the year in which she received the insurance proceeds. D- Since Britney received cash as a result of the involuntary conversion, nonrecognition treatment is not mandatory even if she meets all of the requirements.

Solution: The correct answer is A. The replacement property must meet the taxpayer use test, not the functional use test, since Britney did not use the property directly. The taxpayer use test requires replacement property to be used by the taxpayer in an activity which is treated the same for tax purposes in order to qualify for nontaxable exchange treatment. All of the other statements regarding the nontaxable exchange treatment of Britney's transaction are correct.

Ridge is the manager of a motel. As a condition of his employment, Ridge is required to live in a room on the premises so that he would be there in case of emergencies. Ridge considered this a fringe benefit, since he would otherwise be required to pay $600 per month rent. The room that Ridge occupied normally rented for $60 per night, or $1,500 per month. On the average, 90% of the motel rooms were occupied. As a result of this rent-free use of a room, Ridge is required to include in gross income: A: $0 B: $600 per month C: $1,500 per month D: $1,350 ($1,500 × .90 = $1,350)

Solution: The correct answer is A. The room qualifies for the § 119 lodging exclusion.

Diane purchased a hotel on November 15, five and 1/4 years ago for $5,000,000. Determine the cost recovery deduction for one month. A- $10,683 B- $15,152 C- $21,368 D- $128,200

Solution: The correct answer is A. To depreciate real property, the mid-month convention is used. In addition, a hotel is not considered residential real property and is therefore depreciated using straight line depreciation over 39 years 1 ÷ 39 = .02564 $5,000,000 × 2.564% = $128,200 of annual depreciation expense $128,200 ÷ 12 = $10,683 for one month's depreciation in the current year

Toby, age 15, qualifies as a dependent of his grandmother. During 2022, Toby had interest income in the amount of $200 and earnings from a part-time job of $750. Toby's taxable income is: A: $0 B: $100 C: $650 D: $850

Solution: The correct answer is A. Toby's standard deduction of $1,150 ($750 + $400*) completely negates his gross income of $950 ($750 earned income + $200 interest income). * 2022 Tax law change for kiddie tax - standard deduction is either $1,150 or earned income plus $400 (not to exceed the standard deduction for a single tax filer).

Sammy owned a home in south Florida that was severely damaged by a small hurricane, no Federal disaster was declared. Sammy had purchased the home for $150,000, and the fair market value of the home prior to the hurricane was $500,000. His homeowners insurance policy had lapsed one month before the hurricane hit and Sammy had not obtained any other insurance. After the hurricane, the property had a fair market value of $0. Assuming that Sammy's AGI was $115,000 this year, what is Sammy's casualty loss deduction? A: $0 B: $138,400 C: $138,500 D: $388,400

Solution: The correct answer is A. Under TCJA, personal casualty losses unless located in a federally declared disaster area, are not deductible. Business casualty losses are unchanged. Personal casualty losses in federally declared disaster areas are still subject to the calculation below unless the government grants relief. Prior to the TCJA: Sammy's casualty loss is valued at $150,000 which is his adjusted basis less insurance proceeds received (insurance proceeds in this case are zero). His economic loss (the fair market value before the event, $500,000 less the fair market value after the event, $0) is $500,000. Since Sammy had never paid tax on the $350,000 gain in the property, however, he cannot take a tax deduction for the economic loss. If Sammy had the property fully insured, he would have received the full $500,000 (less his deductible) from the insurance company. Sammy's casualty loss of $150,000 must be reduced by $100 and the result is only deductible to the extent it exceeds 10% of AGI. The deductible portion of Sammy's casualty loss is $138,400 ($150,000 - $100 - $11,500 [10% of AGI]).

Roger, age 19, is a full-time student at State College and a candidate for a bachelor's degree. During the year, he received the following payments: State scholarship for ten months (tuition and books) $3,600 Loan from college financial aid office $1,500 Cash support from parents $3,000 Interest on CDs $1,700 Cash prize awarded in contest $500 $10,300 What is Roger's adjusted gross income for this year? A: $1,700 B: $2,200 C: $5,800 D: $10,300

Solution: The correct answer is B. $2,200 ($1,700 interest + $500 prize).

In the current year Harold had a Section 1231 gain of $13,000. In the prior years, Harold had the following Section 1231 transactions: YearNet Section 1231 Transaction2022$4, 000 Loss2021$2,000 Loss2020None2019None2018None2017$8,000 Gain2016$2,000 Gain2015$2,000 Gain How will Harold's Section 1231 gain be taxed in the current year? A- $12,000 will be taxed as ordinary income. B- $6,000 will be taxed as ordinary income and $7,000 will be taxed as a Section 1231 capital gain. C- $12,000 will be taxed as a Section 1231 capital gain. D- None of the above.

Solution: The correct answer is B. 5-year Look-back Rule: Harold would have to recognize $6,000 of his 2023 gain as ordinary income since in 2022 and 2021 he had Section 1231 losses. The remaining $7,000 of Harold's Section 1231 gain ($13,000 section 1231 gain from the fact pattern - $6,000 from the 5-year look back) would be treated as a Section 1231 capital gain.

All of the following entities may be appropriate for a business expected to incur losses initially EXCEPT: A- S Corporation. B- C Corporation. C- Partnership. D- LLC.

Solution: The correct answer is B. A C corporation would NOT be appropriate, because the losses do not flow through to the owners. All of the other entities listed may be appropriate.

Mr. and Mrs. Golden are married and have one preschool-aged child. During the year, Mrs. Golden is employed and earns $60,000 while Mr. Golden is a full-time student for the entire year. The amount paid for child care is $3,200. On a joint income tax return, their credit for child and dependent care expenses is: A: $0 B: $600 C: $640 D: $3,000

Solution: The correct answer is B. A spouse who is a full-time student is deemed to have earned income of $250 per month for one qualifying individual ($250 x 12 = $3,000). The eligible employment-related expenses cannot exceed the earned income of the taxpayer, or for married taxpayers, the earned income of the spouse with the lesser amount of earned income (in this case Mr. Golden). Consequently, $600 (20% × $3,000) is the available credit. The maximum expense eligible for the child and dependent care credit is $3,000 per child.

Amy and Steve are married. They sell their home in Texas because Amy was offered a job in Washington DC. Amy owned and used the home for 5 years. Steve moved in 1 year ago. If the basis in the home is $300,000 and the sale price is $600,000, what is the maximum exclusion they can take on this transaction? A: $250,000 B: $300,000 C: $375,000 D: $500,000

Solution: The correct answer is B. Amy meets the two year use rule, and Steve has only lived there a year, so his portion is reduced by 1/2. (½ × $250) + 250 = 375; limited to 300, which is the gain on the sale

Andrea, who lives in Ohio, volunteered to travel to Arizona in February to work on a home-building project for Habitat for Humanity (a qualified charitable organization). She was in Arizona for three weeks. She normally makes $600 per week as a carpenter's assistant and plans to deduct $1,800 as a charitable contribution. In addition, she incurred $700 for transportation in connection with the trip. What is Andrea's deduction associated with this charitable activity? A: $600 B: $700 C: $1,800 D: $2,500

Solution: The correct answer is B. Andrea cannot deduct the estimated value of $1,800 of her contributed services. However, she can deduct transportation costs of $700.

Heather had the following transactions for the 2023 tax year: Alimony received from 2017 divorce = $2,400; Salary earned = $38,000; Cash dividends received on stock investments = $1,000; Child support payments received = $12,000; Heather's AGI for this tax year is: A- $39,000 B- $41,400 C- $51,000 D- $53,400

Solution: The correct answer is B. Child support payments are neither deductible from nor includible in income. Note: alimony received from a contract dated by or prior to 12/31/18 remains includible income for the payee and deductible for the payor. Divorces finalized after 12/31/2018 will follow the new rules (not deductible or includable in income).

Prior to TCJA, the underlying rationale for the alimony rule is that: A- The income should be taxed to the person with a claim of right to the income. B- The income should be taxed to the person who enjoys the benefits of the income. C- The fruit and tree metaphor. D- Alimony is a payment for the taxpayer's property obligation and therefore, it is taxed to the recipient.

Solution: The correct answer is B. As a cash basis taxpayer, the person who receives the alimony (the one who benefits from it) will find the alimony received as taxable income. Alimony will be taxable even if the person entitled to it did not in fact receive it (e.g., payor paid expenses of recipient directly to third parties).

In the past, the IRS was known to cite parties who were owner-employers of businesses structured as C-corporations as receiving too much salary compensation. Amounts were disallowed and recharacterized as dividends. This practice is expected to reduce during the period while Job Growth Tax Relief Reconciliation Act is in force. Why might one expect this? A- Reduction of top marginal brackets means less money in income tax revenue. B- Reduction of dividend tax rates means that dividends, which were once taxed at ordinary tax rates, may net less tax to the government than compensation. C- Reduction of marginal tax rates and dividend rates, which are still not a deductible expense to the corporation, means less corporate income tax revenue. D- Reduction of top marginal brackets indicates an IRS softening of the hard-line position once taken in seeking out revenue.

Solution: The correct answer is B. At one time, the higher salaries an owner/employer took, the less profit for the corporation. However, with the reduction of the dividend rate to 15% for most taxpayers (20% for high income taxpayers), the IRS has more incentive to pursue revenue collection from salaries of these individuals.

After 1989, Donna purchased series EE savings bonds for $2,500 at the age of 25. This year she redeemed the bonds for $5,000 and paid qualified higher education expenses for her daughter in the amount of $3,000. How much interest will Donna be required to include in her gross income this year? A- $0. B- $1,000. C- $1,500. D- $2,500.

Solution: The correct answer is B. Because Donna did not use all of the proceeds from the bond redemption to pay for qualified education expenses, she will be required to include part of the interest income from the bonds in her gross income. The formula is a ratio utilizing the given information in the question fact pattern. She redeemed $5,000 worth of bonds, and used $3,000 of that for education so that's the beginning of the formula 3,000 / 5,000. When EE government bonds are used for education, the interest is free of income taxes. Take the ratio calculated (3,000/5,000 = .60) times the $2,500 of gain, and that is the amount that is excluded from income. Donna may exclude $1,500 of interest income from her gross income [($3,000 / $5,000) × $2,500 = $1,500]. Therefore, Donna must include $1,000 in her gross income ($2,500 - $1,500).

On February 12 of last year, Jason gifted stock in Retro Corp to his brother Aaron. Jason's basis in Retro's stock was $12,000 and it had a $9,000 fair market value on the date it was gifted. On August 31 of the following year, Aaron sold the Retro stock to Amy (an unrelated party) for $10,000. What is the tax treatment of these transactions to Aaron and/or Jason? A- Jason recognized a loss of $3,000 last year and Aaron recognized a gain of $1,000 in the following year. B- Neither Jason nor Aaron has a recognized gain or loss in either year. C- Aaron had a recognized gain of $1,000 in the following year. D- Jason had a recognized loss of $3,000 last year.

Solution: The correct answer is B. Because of the double basis rule, Aaron's basis is $12,000 for gain and $9,000 for loss. So he has no gain or loss. Because Jason gifted the shares to a related party, he has no recognizable loss. Double basis simply refers to the two transactions, high and low. The donee uses the FMV of the date of gift ($9,000) as the 'loss basis,' and the donor's adjusted taxable basis ($12,000) as the 'gain basis.' When the shares are sold between the FMV (date of gift) $9,000, and the adjusted taxable basis $12,000, the donee uses that value ($10,000) as his adjusted taxable basis.

Brady is starting a new business where he will be the sole owner. He would like to have limited liability, but he would prefer flow-through taxation because he expects to have losses in the first few years. He is not concerned about incurring self-employment taxes. Which of the following entities would best suit Brady's needs? A- A General or Limited Partnership. B- A Single-member LLC. C- A C corporation. D- A Proprietorship.

Solution: The correct answer is B. Both the proprietorship and the single-member LLC would provide the flow-through taxation that Brady is looking for, but only the LLC would also provide limited liability.

For purposes of tax deductibility what is the minimum deductible that a family can have on a high deductible health policy in the current year? A- $1,500 B- $3,000 C- $3,850 D- $7,750

Solution: The correct answer is B. HDHP for family is $3,000 for 2023. $1,500 is for a single person.

Which of the following statements is correct? A: The purpose of the AMT is to replace the regular income tax. B: Adjustments can either increase AMTI or decrease AMTI. C: Tax preferences can either increase AMTI or decrease AMTI. D: Tax preferences can only decrease AMTI.

Solution: The correct answer is B. Choice A is incorrect because the purpose of the AMT is to back up the regular income tax. Choices C and D are incorrect because tax preferences can only increase AMTI.

Valerie sued her former employer for a physical injury she sustained at work. Valerie was awarded $100,000 to compensate her for the injury and $25,000 in punitive damages. Valerie must include how much in her gross income: A- $0 B- $25,000 C- $100,000 D- $125,000

Solution: The correct answer is B. Compensation for damages for personal bodily injury are not included in taxable income. Punitive damages are included as taxable income. IRC § 104(a)(2) excludes from gross income "the amount of any damages (other than punitive damages) received (whether by suit or agreement and whether as lump sums or as periodic payments) on account of personal physical injuries or physical sickness."

Which of the following taxes generates the largest percentage of gross collections for the Internal Revenue Service? A- The Corporate income tax. B- The Individual income tax. C- The Estate tax. D- The Employment tax.

Solution: The correct answer is B. Individual income taxes make up nearly 50% of the gross collections by the Internal Revenue Service.

Tom operates an illegal drug operation and incurred the following expenses: Salaries = $50,000; Illegal kickbacks = $20,000; Bribes to border guards = $25,000; Cost of goods sold = $150,000; Rent = $8,000; Interest = $10,000; Taxable income = $400,000. How much is his taxable income reduced by, based on the above expenses? A- $-0- B- $150,000 C- $218,000 D- $263,000

Solution: The correct answer is B. Cost of goods sold is the only deduction allowed for illegal drug operation activities. This is an actual IRS rule and has popped up on the CFP Board exam from time to time.

Blair Corporation is in the turkey processing and marketing business. The company gives all of its employees a turkey each Christmas. The value of the turkey: A- May be excluded from the employees' income as a working condition fringe. B- May be excluded from income for the employees as a de minimis fringe benefit. C- May be excluded from income for the employee as a no-additional-cost fringe benefit. D- Must be included in the employees' gross income and reported on the W-2.

Solution: The correct answer is B. De minimus (negligible costs) items provided to employees are not considered taxable fringe benefits. It would be De minims since it is the product the company process for sale.

Which of the following events would produce a deductible income tax loss? A- Erosion of personal use land due to rain or wind not compensated for by insurance. B- Your new business car is stolen and is only insured for bodily injury and property liability. C- Damages to personal automobile resulting from the taxpayer's willful negligence. D- A misplaced personal use item which you have not yet found.

Solution: The correct answer is B. Deductible losses do not result from willful or negligent loss. Erosion of land is a normal event which is not an allowable loss. Your new business car being stolen would constitute a business casualty loss if uninsured for comprehensive. A personal casualty loss would not be deductible.

Which of the following is not an ordinary income asset? A- Literary compositions in the hands of the author. B- Depreciable property or real property used in a trade or business. C- Notes receivable from a trade or business. D- Stock in trade held for sale to customers in the ordinary course of business.

Solution: The correct answer is B. Depreciable property or real property used in a trade or business is a Section 1231 asset, not an ordinary income asset. All of the other options are ordinary income assets. Instructor note: You and I holding stock in our brokerage account is a capital asset, but here it says it is being held by a dealer to sell to customers, i.e inventory.

Gift property (disregarding any adjustment for gift tax paid by the donor): A: Gifted property has no basis to the donee because he or she did not pay anything for the property. B: Has the same basis to the donee as the donor's adjusted basis if the donee disposes of the property at a gain. C: Has the same basis to the donee as the donor's adjusted basis if the donee disposes of the property at a loss, and the fair market value on the date of gift was less than the donor's adjusted basis. D: Has no basis to the donee if the fair market value on the date of gift is less than the donor's adjusted basis.

Solution: The correct answer is B. Disregarding any adjustment for gift tax paid by the donor, the donee's gain basis for the property received is the same as that of the donor. The donee's loss basis is the lower of the donor's adjusted basis or fair market value on the date of the gift. Thus creating a dual basis.

Which of the below options correctly names the following definition: When income is readily available to the taxpayer, and that income is not subject to substantial limitations or restrictions, that income should be subject to tax. A- Accrual based accounting. B- Doctrine of constructive receipt. C- Cash based accounting. D- Qualifying transaction.

Solution: The correct answer is B. Doctrine of constructive receipt applies to many areas, including tax, employee benefits and retirement. For example, an employee is granted an employee bonus plan to be payable upon their retirement from the company. It is not taxable to the employee while they are working for the company. Once the employee reaches retirement age, the restrictions have been lifted and the employee can freely access the funds, they will become taxable.

Which of the following statements regarding the earned income credit is correct? A: In order to qualify, the taxpayer must have a qualifying child in his or her household. B: The amount of the credit varies depending on whether the taxpayer has two or three children. C: The credit is available regardless of the amount of the taxpayer's income. D: The employer may never issue an advance payment of the credit to the taxpayer

Solution: The correct answer is B. Earned Income credit is intended to motivate lower-income taxpayers to earn income. The credit increases as earned income increases up to a maximum level. It is slightly more generous for those filing MFJ and/or have one or more children than for those that do not.

Jason has three capital transactions for the current year: Short-term capital loss of $5,000 Short-term capital gain of $3,000 Long-term capital loss of $2,000 What is the net effect on Jason's taxes if he is in the 35% tax bracket? A- $1,400 tax reduction B- $1,050 tax reduction C- $850 tax reduction D- $450 tax reduction

Solution: The correct answer is B. First, tackle the short-term gains and losses. Short-term capital loss of $5,000 Short-term capital gain of $3,000 Net the STCG and STCL = $2,000 STCL. Then, net the long-term gains and losses (if you have both) Lastly, net them together. The $2,000 LTCL plus the $2,000 STCL = Total Loss of $4,000. You end up with a loss of both long and short gains. Since there is no other capital gains to offset, you can use up to $3,000 against ordinary income. The $3,000 will offset ordinary income at 35% = $3,000 × 0.35 = $1,050. The remaining $1,000 is a long-term capital loss carryover. Keep in mind, short-term gains and ordinary income use the same tax table. Only long-term gets the preferential tax treatment. Please visit the Getting Started Section, Additional Resources and view the Infographic on Netting Capital Gains for a visual review of this topic.

A single-life annuity contract was purchased 12 years ago by Kelly. The premiums were $1,200 per year. The annuity payments are $1,000 per year to be paid on February 2nd. Her life expectancy is 20 years. How much taxable income must Kelly recognize this first year from the annuity contract? A- None B- $280 C- $720 D- $1,000

Solution: The correct answer is B. Her basis in the annuity contract is $1,200 premium per year × 12 years she owned the contract = $14,400. The expected return is $1,000 of income per year × 20 year income stream = $20,000. The exclusion ratio is $14,400 ÷ $20,000 = 72% The inclusion ratio (percent that is taxable) is 100% - 72% = 28% The taxable amount each year is therefore $1,000 × 28% = $280

In Year 1, Lena invests $80,000 for a 20% partnership interest in an activity in which she is a material participant. The partnership reports losses of $300,000 in Year 1 and $150,000 in Year 2. Lena's share of the partnership's losses is $60,000 in Year 1 and $30,000 in Year 2. How much of the losses can Lena deduct? A: $60,000 in Year 1 and $30,000 in Year 2 B: $60,000 in Year 1 and $20,000 in Year 2 C: $0 in Year 1 and $0 in Year 2 D: $60,000 in Year 1 and $0 in Year 2

Solution: The correct answer is B. Lena's losses are not subject to the passive loss limitations because she is a material participant in the activity. However, the at-risk rules limit her losses to $80,000, the amount she invested in the partnership interest.

Jack and Marge, both 71, got married on July 1 of this year. Immediately following their honeymoon, Jack succumbed to congestive heart failure and died. The autopsy showed that Jack's death was the result of severe cardiac overexertion resulting from relentless amounts of cardiovascular stress. Jack and Marge had no dependents. What filing status should Marge use for this year's tax return? A- Marge should file single or Married Filing Separately. B- Marge should file as Married Filing Jointly. C- Marge should file as Head of Household. D- Marge cannot file as Married Filing Jointly, but she will be able to claim Jack as an exemption.

Solution: The correct answer is B. Marge may file as Married Filing Jointly as long as she would have qualified to file jointly had Jack survived.

When do the recapture rules for Section 179 apply: I- When the asset is sold before it would have been fully depreciated. II- When the business use drops below 50%. III- When the Section 179 deduction taken in one year exceeds the allowable maximum. IV- When there is sufficient income in one tax year to support the deduction taken. V- When the deduction causes the tax liability to become negative. A- III and IV only. B- I and II only. C- V only. D- All of the above.

Solution: The correct answer is B. Section 179 recapture rules apply when the business use of an asset drops below 50% for a given year or when the asset is disposed of before it would have been fully depreciated.

Ashton, age 29, purchased stock in a real estate investment trust (REIT) with a dividend yield of 9%. He was concerned that the real estate market may have been hurt in the recent market turmoil. However, they kept paying the stated dividend but his 1099-DIV only lists 4% as ordinary dividends. How can this be possible? A- The REIT was paying 5% in qualified dividends. B- The REIT was making nondividend distributions. C- The REIT was selling his shares. D- The REIT already paid tax on the other 5% so Ashton does not have to.

Solution: The correct answer is B. Nondividend distributions are a return of capital and are not included for gross income. (A) is incorrect because REITs cannot pay qualified dividends. (C) is incorrect because the REIT cannot sell an owner's shares. (D) is incorrect because the REIT cannot pay the tax for the shareholder.

Peyton has a warehouse used in his business. He exchanges it for a storage building owned by Eli. (Peyton and Eli are unrelated). The basis of Peyton's asset is $40,000 and he gives Eli $20,000 cash plus the asset in exchange for Eli's asset, which is worth $36,000. Eli's basis in his original asset is $10,000. What is Eli's new basis? A- $0 B- $10,000 C- $20,000 D- $30,000

Solution: The correct answer is B. Note: Only real property is eligible for like kind treatment. You need to calculate Peyton's FMV. You can do this because you know that the values exchanged must be equal. Peyton gives Eli $20k + property in exchange for a property worth $36,000: $20,000 + x = $36,000 x = $16,000

Which of the following statements about a S corporation is true? A- S corporation status is automatic if there are fewer than 75 shareholders. B- S corporations are prohibited from having more than one class of stock. C- S corporations are prohibited from earning passive income. D- S corporations may have non-resident aliens as shareholders.

Solution: The correct answer is B. Option "A" - S status must be applied for, it is never "automatic." Option "C" - S corporations are allowed to earn passive income. Option "D" - S corporations may NOT have non-resident aliens as shareholders.

The best source for gathering information about the intent of recent changes in the tax law might be: A- RIA Federal Tax Coordinator. B- Congressional Committee Reports. C- Treasury Regulations. D- Tax Court Cases.

Solution: The correct answer is B. Option "A" is incorrect because RIA provides plain language interpretation of tax law. Option "C" is incorrect because it is the highest level of tax regulations, but does not indicate the intent of Congress in enacting tax law. Option "D" is incorrect because it provides ruling of the U.S. Tax Court in the form of case law. Option "B" is correct because the Congressional Committee Reports (sometimes known as the Blue Book) provides congressional reasoning for enacting tax law.

Danny is starting a new business. He is concerned about liability. He would like to have flow-through taxation. At some point, he would like to be able to easily sell interests in the business, but he does not expect to have more than 20 investors. Danny does not want to pay self-employment taxes on all income. Which of the following entities would best suit Danny's needs? A- A Proprietorship. B- A S corporation. C- A C corporation. D- A Partnership.

Solution: The correct answer is B. Option "A" is not correct because a proprietorship does not provide limited liability and Danny would need to change entities to take on future investors. Option "C" is not correct because a C coporation would not provide flow-through taxation. Option "D" is not correct because a partnership would not provide limited liability and Danny would have to pay self-employment taxes on the business net income. Therefore, only option "B" meets all of Danny's requirements.

During the year, Myrna furnished more than 50% of the support for the following persons: Butch, Myrna's husband, who has no income and does not file a return. Wallace, Myrna's cousin, who does not live with her. Brad, Myrna's father-in-law, who does not live with her. Dawn, Myrna's 18 year old daughter who is a full time student. Presuming all other requirements for qualified dependent are met, on a separate return, how many qualified dependent credits may Myrna claim? A- One. B- Two. C- Three. D-Four.

Solution: The correct answer is B. Qualified dependents 1. The fact that the father-in-law does not live with Myrna does not automatically disqualify him a dependent (qualified relative does not have to live in the taxpayers home), if he meets all the other qualifying relative requirements, as stated, Myrna can claim the qualified dependent credit for him. 2. Myrna's daughter is over 17 and ineligible for the qualifying child credit of $2,000 but, as a full time student, she will qualify as a qualified relative for the $500 credit. The rules for age change depending on what is tax advantage is being asked for. Qualifying for kiddie tax is under 19, qualifying for child tax credit is under 18, and qualifying for dependent care is under age 13. Does not qualify as a dependent Butch, the husband, cannot be a qualified dependent for tax purposes. Cousins do not meet the relationship test.

A scholarship recipient who is a candidate for a degree may exclude from gross income the scholarship proceeds received for: A- Dorm Room. B- Tuition. C- Board (meals). D- Research costs and travel related to studies.

Solution: The correct answer is B. Scholarships received are not included as income when used for tuition and books as long as the recipient is a candidate for a degree. Monies received for living expenses including meals, housing, and stipend are all included in gross income.

Rudy bought 10 shares of Fat Cat, Inc. stock on January 1 of the current year. Rudy paid $20 for each share. At first, it appeared that Rudy had made a good investment, as the price of Fat Cat stock rose to $50 per share on March 1 of the current year. However, rumors of corporate wrongdoing soon started to circulate and the price of Fat Cat began to fall. On August 1 of the current year, Fat Cat, Inc. declared bankruptcy and announced that the stockholders should not expect to receive anything on the liquidation of the corporation. What type of loss does Rudy have in the current year if any? A- Rudy does not have a loss because he did not sell the stock. B- Short-term capital loss of $200. C- Short-term capital loss of $500. D- Long-term capital loss of $200.

Solution: The correct answer is B. Section 165 creates an artificial sale date of December 31 for worthless securities. Therefore, Rudy is deemed to have sold the stock for $0 on December 31. Because Rudy's holding period was less than one year and one day, his loss will be a short-term capital loss. The amount of Rudy's loss is equal to his basis in the investment.

Rudy bought 10 shares of Fat Cat, Inc. stock on January 1, Year 1. Rudy paid $20 for each share. At first, it appeared that Rudy had made a good investment, as the price of Fat Cat stock rose to $50 per share on March 1, Year 1. However, rumors of corporate wrongdoing soon started to circulate and the price of Fat Cat began to fall. On August 1, Year 1, Fat Cat, Inc. declared bankruptcy and announced that the stockholders should not expect to receive anything on the liquidation of the corporation. What type of loss does Rudy have in Year 1, if any? A: Rudy does not have a loss because he did not sell the stock. B: Short-term capital loss of $200 C: Short-term capital loss of $500 D: Long-term capital loss of $200

Solution: The correct answer is B. Section 165 creates an artificial sale date of December 31 for worthless securities. Therefore, Rudy is deemed to have sold the stock for $0 on December 31. Because Rudy's holding period was less than one year and one day, his loss will be a short-term capital loss. The amount of Rudy's loss is equal to his basis in the investment.

Elaine incurred $26,000 of margin interest on her $600,000 investment portfolio. Her portfolio income consists of $10,000 in interest, $15,000 in qualified dividends, $3,000 in ordinary dividends, $6,000 in short-term capital gains, and $11,000 in long-term capital gains. How much of the margin interest is deductible on Elaine's tax return assuming no special elections? A- $17,000 B- $19,000 C- $24,000 D- $26,000

Solution: The correct answer is B. She can deduct up to her net investment income which = $10,000 + 3,000 + 6,000 = $19,000 without making a special election. $10,000 in interest, taxed at ordinary income rates $15,000 in qualified dividends, taxed at capital gains rates $3,000 in ordinary dividends, taxed at ordinary income rates $6,000 in short-term capital gains, taxed at ordinary income rates $11,000 in long-term capital gains, taxed at capital gains rate If she elected to treat the long-term capital gains and qualified dividends as ordinary income she could deduct it all. However, DO NOT assume that the election is made. The question would have to specify that information or ask "what is the maximum she can deduct?"

During the current year, Justin, a self-employed individual, paid the following amounts: Federal income tax = $5,000; State income tax = $3,000; Real estate taxes on land in a neighboring state = $8,000; State sales taxes = $600; State occupational license fee = $400. What amount can Justin deduct as taxes by itemizing his deductions? A- $3,600 B- $10,000 C- $11,000 D- $11,600

Solution: The correct answer is B. State income or sales tax (whichever is higher) and real estate taxes are deductible as itemized deductions but for years after 12/31/17, they are capped at $10,000 (total). Federal income taxes paid are deductible from the tax liability, but are not an itemized deduction. Occupational license fees are deductible as a direct business cost in which they are deducted for AGI not as itemized deductions.

The Qualified Dependent Test includes which of the following tests: I- Gross income. II- Support. III- Member of household or family. IV- Citizenship or residence. V- Joint return. A- I and II only. B- I, II, IV and V only. C- I, II and IV only. D- All of the above.

Solution: The correct answer is D. The five elements of the Qualified Dependent Test are: Gross income, support, member of household or family, citizenship or residence, and joint return.

An exchange of Plum, Inc., stock for Orange, Inc., stock qualifies as a like-kind exchange under § 1031. A: True B: False

Solution: The correct answer is B. Stock and other securities are not eligible for like-kind exchange treatment under § 1031. § 1031 provides like-kind exchange treatment for real property assets held for trade or business and for real property assets held for production of income.

Alisha Syrmos, a CFP® Professional and fee-only financial planner, has assisted Bob Martin, a self-employed physician in tax and investment planning during the year. Identify the schedule(s) on which Alisha's fee may be deductible by Bob on his federal income tax return. I- Schedule A - itemized deductions. II- Schedule C - profit or loss from business. III- Schedule D - capital gains and losses. A- I only. B- II only. C- I and II only. D- I, II and III.

Solution: The correct answer is B. Tax planning fees may no longer be deducted against a taxpayer's itemized deduction, Schedule A. However, because the taxpayer is self-employed, the portion of services related to the business and not personal may be taken as a deductible expense on the taxpayer's Schedule C.

Your client is contemplating the sale of some of her holdings in her employer's stock. The stock was acquired in four separate purchases as follows: Stock #1 - purchased 200 shares for $2,000 ($10 per share) on June 1, 2006. Stock #2 - purchased 200 shares for $3,600 ($18 per share) on June 1, 2007. Stock #3 - purchased 200 shares for $2,400 ($12 per share) on June 1, 2008. Stock #4 - purchased 200 shares for $4,000 ($20 per share) on June 1, 2009. She wants to know the least amount of gain she would be required to report if she sold 500 shares for $12,500. Compute this gain, assuming she has previously sold no shares. A- $500 B- $3,700 C- $4,300 D- $5,000

Solution: The correct answer is B. The 500 shares would be sold as follows: 200 shares from Stock #4; 200 shares from Stock #2; and 100 shares from Stock #3. These would total $4,000 + $3,600 + $1,200 = $8,800 basis. This subtracted from $12,500 - $8,800 = $3,700. The taxpayer can choose one method of sale as long as they stay with it. LIFO, FIFO, average shares, or identifying shares.

Which of the following credits are fully refundable? I- The Earned Income credit. II- The American Opportunity credit. III- Lifetime Learning credit. IV- Child Tax credit. V- Adoption credit. A- I, IV and V only. B- I only. C- I and V only. D- None of the choices.

Solution: The correct answer is B. The Earned Income credit is refundable; able to create a negative tax liability. The American Opportunity credit and Child tax credit may be partially refundable.

Under the First in First Out (FIFO) inventory system: A- The first good purchased is the first good sold. B- The cost of goods sold is based on the costs of the first goods purchased. C- FIFO reduces the probability of scrap or obsolescence. D- All of the above.

Solution: The correct answer is B. The FIFO method is concerned with movement of costs through inventory, not goods. The cost of the first units purchased are the first costs to be transferred to cost of goods sold when the goods are sold.

Kari owns depreciable residential rental real estate which has accumulated depreciation (all from straight-line) of $45,000. If Kari sold the property, she would have a $33,000 gain. The initial characterization of the gain would be: A: Section 1250 gain B: Section 1231 gain C: Section 1239 gain D: Section 179 gain

Solution: The correct answer is B. The gain is § 1231 gain. Since straight-line depreciation was used, there is no § 1250 recapture. Also, since Kari is an individual, there is no "ordinary gain adjustment" under § 291. Section 1239 would not apply because there is no reason to conclude that the property would be sold to a related taxpayer. Once an asset has been categorized as a Section 1231 asset, it is afforded special tax treatment. Gains on Section 1231 assets are treated as capital gains, and losses on Section 1231 assets are treated as ordinary losses. Section 1231 gives the taxpayer the best of capital and ordinary income asset treatment. Since straight-line depreciation was used, there is no § 1250 recapture. Also, since Kari is an individual, there is no "ordinary gain adjustment" under § 291. Section 1239 would not apply because there is no reason to conclude that the property would be sold to a related taxpayer. Instructor note: 1250 is for depreciation taken in excess of straight line. There was no excess depreciation taken. Gains from the sale of income property is 1231 gain.

If a taxpayer incurred $7,000 in capital losses and $2,000 in capital gains in the same year, they would: A- Apply 100% of the losses against the capital gains and deduct the remainder against ordinary income. B- Apply 100% of the losses against capital gains, deduct $3,000 and carryover the remaining $2,000 to the future. C- Apply $2,000 to offset the capital gains and carry over the remainder to tax years with capital gains for an unlimited time. D- Apply $2,000 to offset the capital gains the taxpayer loses the rest of the losses.

Solution: The correct answer is B. The maximum capital loss that a taxpayer can apply is up to the extent of any capital gains for that year plus an additional $3,000. Any excess losses are carried forward and applied until used. In this case, the $2,000 would be offset with an additional $3,000 taken for the current year. That would leave a carryover loss of $2,000 to the future.

Tad is eligible for a qualified dependent credit for his 70-year-old mother. In calculating her taxes, his mother may not claim an additional standard deduction for her age. A: True B; False

Solution: The correct answer is B. The mother can claim the additional standard deduction for her age.

Alice owns land "A" with an adjusted basis of $250,000, subject to a mortgage of $50,000. On July 1st, Alice exchanges land "A" and its mortgage for $300,000 in cash, a promissory note for $300,000, and property "B" that has a fair market value of $75,000 with Betty. What is the amount realized by Alice? A: $675,000 B: $725,000 C: $925,000 D: $975,000

Solution: The correct answer is B. The realized amount not only includes the monies and fair market value of property "B" received (and any indebtedness the buyer has to the seller), but also any liabilities for the seller is relieved. In this case, the seller received $675,000 in cash, property, and notes (buyers indebtedness to the seller) as well as relief from $50,000 in mortgage. The total amount realized is $725,000. The question is tricky because of the vocabulary. The "amount realized" is the total that she receives.

Andres owns a ski cabin with an adjusted basis of $350,000, subject to a mortgage of $50,000. On July 1st, Andres exchanges the cabin and its mortgage for $300,000 in cash, a promissory note for $300,000, and a beach house that has a fair market value of $75,000 with Bart. What is the amount realized by Andres? A- $675,000 B- $725,000 C- $925,000 D- $975,000

Solution: The correct answer is B. The realized amount not only includes the monies and fair market value of the beach house received (and any indebtedness the buyer has to the seller), but also any liabilities for the seller is relieved. In this case, the seller received $675,000 in cash, property, and notes (buyers indebtedness to the seller) as well as relief from $50,000 in mortgage. The total amount realized is $725,000. Summary: When dealing with an exchange, there are two things to note: realized and recognized. Realized is a transaction happening. Recognized is when a realized transaction has not met an exception and must have tax calculated. A like kind exchange is an exception to realized transactions to not be recognized. This question asked the realized amount which is the value received plus debt relief totaling 725k. There is no recognized amount, which is what you were trying to calculate.

Beau would like to invest in bonds and is considering either a taxable bond with an interest rate of 5% or a tax-exempt municipal bond of comparable risk and quality with an interest rate of 3%. Beau's marginal tax rate is 25%. In order to help Beau compare these two bonds, compute the equivalent tax-free rate for the taxable bond. A- 3%. B- 3.75%. C- 4.25%. D- 5%.

Solution: The correct answer is B. When you are given the taxable and need to calculate the tax-free rate, you use the algebraic equivalent of the TEY formula. Stupid trick to remember: If I am given the taXable rate, I need to multiple (X) in the formula. The equivalent tax free rate for the taxable bond is [0.05 × (1 - 0.25)] = 3.75%

Suppose a company agrees to pay an executive employee $150,000 per year plus 20% of profits for the coming year. The company has a banner year and the employee is paid compensation of $750,000 as a result. Would this likely be deemed unreasonable, and therefore, non-deductible? A- Yes, because the amount is unreasonably high whether it was a good year or not for the company. B- No, because the reasonableness of salaries is tested according to the circumstances under which the agreement was made. C- Yes, because the reasonableness of salaries is tested according to the circumstances when the compensation is actually paid. D- No, because the employee will have to pay tax on the compensation anyway.

Solution: The correct answer is B. While $750,000 in compensation may appear unusually large, in this instance it can be considered reasonable because the employee took some risk in accepting this arrangement. The compensation also was deemed appropriate at the time at which it was made. Therefore, the $750,000 that may otherwise be deemed unreasonable is (in this case) within the limits of reasonableness.

Paul (age 35) and his wife Stacey (age 33) are married with three young children. They both work outside the home. Paul is a corporate executive with Wellstar and Stacey is an executive assistant with a small local company. Paul fully participates in his company's qualified retirement plan by contributing $22,500 (2023) of his salary, which is matched 100% up to 3% of compensation. Stacey's employer does not offer a retirement plan. In addition, during the year they had the following items of income and expense: Paul's gross salary: $150,000 Stacey's gross salary: $32,000 Stacey's cash gift to her mother: $7,000 Interest from a joint savings account: $100 Federal income taxes withheld from paychecks: $30,000 State and local income taxes: $12,000 Charitable contributions made: $5,500 Mortgage interest for home: $11,100 Real Estate Taxes on home: $6,000 Contribution to Paul's traditional IRA: $6,500 Contribution to Stacey's traditional IRA: $6,500 Medical expense due to a car accident: $16,000 (their max out of pocket) What is Paul and Stacey's taxable income? A- $115,482.50 B- $121,982.50 C- $125,400.00 D- $131,900.00

Solution: The correct answer is B. You need to know and understand the flow of the tax formula for the exam: *Paul uses the 116k-136k phase out, Stacey uses the spousal 218k-228k phase out Itemized deductions equal $31,117.50 versus the standard deduction of $27,700 in 2023.

Wilbur had the following transactions for the most recent business tax year: Alimony paid (2017 divorce) = $3,600; Commissions earned as a salesperson = $54,000; Capital loss on stock investment = $1,000; Gift received from mother = $2,000; Qualified residential interest paid on home mortgage = $6,000. Wilbur's AGI for that year is: A- $43,400 B- $45,500 C- $49,400 D- $50,400

Solution: The correct answer is C. $54,000 (commission earned) - $3,600 (alimony paid) - $1,000 (capital loss) = $49,400. The gift is not taxable, and the interest on the home mortgage is a deduction from AGI. Please note: only alimony contracts signed after 12/31/18 will be subject to the new law and result in no deduction for alimony. Net capital losses of individuals are deductible FOR AGI to the extent of $3,000 per year. • Excess capital losses are carried over to the next tax year indefinitely.

Michelle's husband passed away in January this year. She does not remarry and still maintains a residence for herself and her son who is 10 years old. When she is filing her tax return for this year she may file as: I- Single II- Married filing jointly III- Married filing separately IV- Qualifying widower A- I only B- IV only C- II and III only D- II, III, and IV only

Solution: The correct answer is C. (I) Since Michelle's spouse died during the year she is considered married for the year. (II) and (III) Since Michelle is considered married for the full year (she was married but her spouse died during the year and she did not remarry) she may file MFS or MFJ. (IV) She does not currently qualify for filing qualifying widower since this status applies for the 2 years following the year of a spouse's death.

Ron wants to make a gift to a state university. Which of the following gifts would provide Ron with the largest charitable deduction for the current tax year if his AGI is $100,000 and he has not made any prior gifts? A- A gift of Long-term publicly traded stock valued at $40,000 with a basis of $10,000. B- A gift of current year inventory valued at $30,000 with a basis of $25,000. C- A cash gift of $55,000. D- Real estate held for years valued at $45,000 with a basis of $10,000.

Solution: The correct answer is C. A cash gift to a public charity such as the state university provides for a deduction of up to 60% of the donors AGI. The cash gift allows Ron to deduct $55,000. Choice A is incorrect because this stock gift is limited to 30% of AGI. This allows Ron to deduct only $30,000 because it is long term gain property. Choice B is incorrect because the gift of inventory only provides Ron with a $25,000 deduction, limited to basis. This is ordinary income property. Choice D is incorrect because the real estate only allows Ron to deduct $30,000 since it is limited to 30% of his AGI because like A, it is long term capital gain property.

Which of the following events would produce a deductible loss? A: Erosion of personal use land due to rain or wind. B: Termite infestation of a personal residence over a several year period. C: A delivery van used for business and destroyed in an auto accident. D: A stolen diamond ring.

Solution: The correct answer is C. A casualty loss may be taken for business assets.

In which of the following venues is a jury trial available for tax controversies? A- The U.S. Tax Court. B- The U. S. Tax Court, Small Claims Division. C- A U. S. District Court. D- The U.S. Court of Federal Claims. E- All of the above.

Solution: The correct answer is C. A jury trial is only available in tax controversies adjudicated by the U.S. District Court. Only bench trials are available in the other venues.

The holding period of property acquired by gift may begin on: A- The date the property was acquired by the donor only. B- The date of gift only. C- Either the date the property was acquired by the donor or the date of the gift. D- Some other date.

Solution: The correct answer is C. A person receiving a gift has a holding period of the donor plus the donee if at disposition he uses the gain basis. However, the holding period for a gift utilizing the loss basis (where a double basis rule applies) starts the holding period at the date of the gift.

Which of the following is not a requirement for the deferral of gain in a Non simultaneous exchange under Section 1031? A- The replacement property must be like-kind property with respect to the original property. B- The proceeds from the sale of the original property must be held by an escrow agent. C- A replacement property must be identified within 90 days of the sale of the original property. D- The closing on the replacement property must take place by the earlier of 180 days from the sale of the original property or the due date (including extensions) of the tax return for the year the original property was sold.

Solution: The correct answer is C. A replacement property must be identified within 45 days, not 90 days, of the sale of the original property.

Under which of the following circumstances is a trip outside the United States considered to be purely for business? I- The taxpayer does not have control over the timing or arrangements for the trip. II- The trip outside the United States lasts for less than seven days. III- Less than 50 percent of the time spent on the trip was personal. IV- Vacation was not a primary consideration for the trip. A- I only. B- II and III only. C- I, II and IV only. D- I, II, III, and IV

Solution: The correct answer is C. A trip outside the United States is considered to be purely for business when less than 25 percent of the time spent on the trip was personal. All of the other statements regarding travel outside the United States are correct.

Under which of the following circumstances is a trip outside the United States considered to be purely for business? I: The taxpayer does not have control over the timing or arrangements for the trip. II: The trip outside the United States lasts for less than seven days. III: Less than 50 percent of the time spent on the trip was personal. IV: Vacation was not a primary consideration for the trip. A: 1 only B: 2 and 3 C: 1, 2, and 4 D: 1, 2, 3, and 4

Solution: The correct answer is C. A trip outside the United States is considered to be purely for business when less than 25 percent of the time spent on the trip was personal. All of the other statements regarding travel outside the United States are correct.

Mackenzie has two apartment units that are occupied by tenants all year long. In December, the tenants in unit 2 paid him in advance for the next January's rent. The regular rent is $1,000 per month for each of the units. How much rental income must Mackenzie include in taxable income this year? A- $12,000 B- $24,000 C- $25,000 D- It depends on which accounting method he uses

Solution: The correct answer is C. According to Publication 17, "Advance rent is any amount you receive before the period that it covers. Include advance rent in your rental income in the year you receive it regardless of the period covered or the method of accounting you use." The math: Two units, $1,000 each per month. It is December and the tenet in unit 2 pre-pays January. 12,000 for unit 1, and 13,000 for unit 2. Total of $25,000.

Kathy operates a gym. Her customers pay $480 for 48 aerobic classes that can be taken any time in a 24-month period. If Kathy sells one of the agreements to a customer on June 30 of this year, and the customer takes six lessons during the year, how much of the income is Kathy required to recognize as income on this year's taxes? A- $480 if she is a cash or accrual basis taxpayer. B- $240 if she is an accrual basis taxpayer. C- $60 if she is an accrual basis taxpayer. D- $120 is she is a cash basis taxpayer.

Solution: The correct answer is C. Accrual taxpayers who receive prepaid revenues do not recognize taxable income until the revenue is actually earned. Kathy has earned 6 out of 48 classes or 12.5% (6 / 48) of the revenue collected; 12.5% × $480 = $60.

Dudley and Belle were divorced in 2008. Dudley's ex-spouse (Belle) has custody of their 10-year old daughter. During the current year, Dudley made alimony payments of $12,000 to contractors and maintenance workers for upkeep of Belle's house at Belle's direction. In addition, Dudley paid child support payments of $6,000 to Belle. Dudley's adjusted gross income before any deductions for the above-listed expenses was $75,000. What is the appropriate amount of the deduction on Dudley's federal income tax return this year for the above transactions? A- $0 B- $6,000 C- $12,000 D- $18,000

Solution: The correct answer is C. Alimony is deductible even when paid to a third party on behalf of the receiving ex-spouse as long as agreed. Identifiable child support is never included in income or deductible from income of the payor ex-spouse. Divorces finalized on or before December 31, 2018 follow the old rules (deductible for payor and included in income for payee). Divorces finalized after December 31, 2018 follow the new rules (neither deductible or included in income).

Which of the following statements is/are NOT representative of the Federal income tax on individuals? A- Federal Income tax rates are progressive. B- The tax base for the application of income tax rates is taxable income. C- Persons who have earned income other than wages are subject to withholdings. D- All of the above are correct.

Solution: The correct answer is C. All persons with taxable income are subject to the "pay-as-you-go" payment procedure but not withholdings. They must make estimated payments or face interest and penalties.

Which of the following does not illustrate one of the four basic tax planning principles? A- Buying investments that pay qualified dividends rather than nonqualified dividends. B- Taking advantage of employer-provided health insurance. C- Frequent buying and selling of investments in order to take advantage of the short-term capital gains rates. D- Investing in a Roth IRA.

Solution: The correct answer is C. Answer "C" does not illustrate one of the four basic tax planning principles. Long-term capital gains rates are more advantageous than short-term capital gains rates; therefore, taxpayers are better off holding investments for at least one year rather than buying and selling on a frequent basis.

Which of the following are preference items or adjustments for purposes of the individual alternative minimum tax? I- Qualified private-activity municipal bond interest. II- The excess of percentage depletion over the property's adjusted basis. III- Investment interest in excess of net investment income. IV- Qualified housing interest. A- I only. B- II and IV only. C- I and II only. D- II and III only.

Solution: The correct answer is C. By definition, investment interest expense in excess of net investment income and qualified housing interest are not preference items or adjustments for purposes of the alternative minimum tax.

How would the realization requirement influence an investor's decision to purchase stocks expected to appreciate in value but not paying dividends versus stocks paying dividends but not expected to appreciate in value? I- Dividends are recognized income in the year they are received. II- Appreciation in value is taxed in the year in which it occurs. III- Stock dividends are not recognized until the stock is sold. IV- Realization is an economic concept and recognition is a tax concept. A- I and IV only. B- II and III only. C- I, III, and IV only. D- I, II, and III only.

Solution: The correct answer is C. Dividends paid in cash are recognized as income in the year the dividends were paid. A dividend paid in shares of stock is not taxed upon receipt and will appreciate in value until the stock is sold. A cash based taxpayer recognizes income when received either actually or constructively. Realization is an economic concept, recognition is a tax concept.

Under the cash basis method of accounting the seller recognizes income when: A- The check is drawn by the payor (buyer). B- The check is deposited by the seller. C- The check is received by the seller. D- The receipt must be in currency; checks are not recognized as cash receipts.

Solution: The correct answer is C. Cash accounting recognizes income upon either actual or constructive receipt. Actual receipt is where the taxpayer has received the cash directly. Constructive receipt occurs when, though not having received the money in hand, the taxpayer has immediate access to the money (i.e., lock box receipt).

Junior is 8 years old and has an UGMA account that has been funded with various bonds by Senior. Junior's interest income is $5,500 for the current tax year. How much of the interest will be subject to the parent's tax rate? A- $1,250 B- $2,500 C- $3,000 D- $4,250

Solution: The correct answer is C. For dependent children, unearned income over $2,500 is subject to tax at the parent's tax rates. 5,500-2,500 = 3,000 Please view the pre-study lecture on Kiddie Tax if you need additional information on this topic.

Which of the following is not a requirement for a deductible business-related expense? A- The expense must be ordinary. B- The expense must be necessary. C- The expense must be capitalized. D- The expense must be reasonable.

Solution: The correct answer is C. Deductible business expenses are usually not capital in nature. Capitalized expenses are recovered over the life of the asset through depreciation deductions, rather than through a deduction for a business expense.

Fiona is a highly compensated employee of GreatWorks, Inc. Which of the following fringe benefits would be taxable to Fiona? A- Health insurance provided by GreatWorks to all employees. B- Group term life insurance in the amount of $40,000 paid for by GreatWorks. C- Dependent care assistance for the highly compensated employees of GreatWorks. D- On-premises athletic facilities that may only be used by the managers and vice-presidents of GreatWorks.

Solution: The correct answer is C. Dependent care assistance can only be excluded from a highly compensated employee's gross income if it is provided on a nondiscriminatory basis. Answer "D" is not correct because access to athletic facilities can be provided on a discriminatory basis without causing inclusion in the employee's gross income.

Dakota qualifies as a dependent of his parents. This year, he earned $500 from a part-time job and $1,500 in interest from a savings account. Dakota's taxable income for this year is: A- $2,000 B- $1,250 C- $750 D- $600

Solution: The correct answer is C. First determine the standard deduction. The greater of $1,250 or earned income plus $400. Earned income of $500 plus $400 = $900, the standard deduction of $1,250 is greater. The calculation is as follows: Step 1 - unearned income. $1,500 (interest) - $1,250 (standard deduction) $250 taxed at the child's rate Step 2 - earned income $500 (wage) all taxed at the child's rate. The full standard deduction was already used. Total taxed at the child's rate is $250 + $500 = $750

Georgia recently rolled over her traditional IRA from one institution to another. When gathering information for Georgia's tax return for the year, which combination of IRS forms would be used by her CPA to obtain necessary information relating to the rollover? A- Form 8606 and Form 1040. B- Form 8606 and Form 5498. C- Form 1099-R and Form 5498. D- Form 1099-R and Form 1040.

Solution: The correct answer is C. Form 1099-R is used to report the amount of the rollover. Form 5498 reflects contributions to the IRA. This fact pattern states they are preparing to file, not actively filing. Form 5498 should be on the list of documents needed, as it will complete the file once it is available. A is incorrect. Form 8606 is used to determine the tax consequences of certain taxable distributions from IRAs. B is incorrect. Form 8606 is used to determine the tax consequences of certain taxable distributions from IRAs. D is incorrect. Form 1040 would be used to report the rollover but not to gather information relating to the rollover.

Gordon, an employee, is provided $125,000 of group term life insurance coverage this year. The plan is nondiscriminatory. Since Gordon paid nothing towards the cost, he must include in his gross income this year which of the following amounts? A: The amount the company pays for Gordon's insurance. B: The amount that Gordon would have to pay if he purchased a $125,000 insurance policy. C: The cost (based on IRS tables) of $75,000 of group term life insurance. D: The cost (based on IRS tables) of $25,000 of group term life insurance.

Solution: The correct answer is C. Gross income would be reported for the cost (based on IRS tables) paid by an employer on any amount of group term insurance above $50,000 of coverage.

For the current year, Harry reported salary and taxable interest income of $50,000. His capital asset transactions during the year were as follows: Long-term capital loss = ($5,000) Long-term capital gain = $1,000 For the current year, what amount should Harry report as adjusted gross income? A- $45,000 B- $46,000 C- $47,000 D- $50,000

Solution: The correct answer is C. Harry has a net long-term capital loss of $4,000 [($5,000) LTCL + $1,000 LTCG]. First net all gains and losses, any excess loss up to $3,000 can be taken against ordinary income, remaining loss will carry forward. $50,000 salary and interest income - $3,000 capital loss against ordinary income = $47,000 AGI

Which of the following personal income tax planning techniques are used to defer (postpone) taxation? I- Itemizing deductions. II- Contributing to an individual retirement account. III- Using the child care credit. IV- Owning cash value life insurance. A- I and II only. B- II only. C- II and IV only. D- III and IV only.

Solution: The correct answer is C. IRAs and cash value insurance both involve deferral techniques. The other options are avoidance or elimination techniques. Earnings on the cash value of an insurance policy grow tax-deferred. The earnings on cash value would be taxable if withdrawn or the policy was canceled. An inforce policy that pays to the beneficiary upon death of the insured would be tax free.

Trevor (age 66) and Susan (age 67 and blind) file a joint return for 2023. What is their standard deduction? A- $27,700 B- $30,700 C- $32,200 D- $33,250

Solution: The correct answer is C. In 2023 the standard deduction for a married couple is $27,700. Since they are both 65 years of age or older they may also take an additional standard deduction of $1,500 each. Lastly, since Susan is blind, she may take an additional standard deduction of $1,500 upon filing. $27,700 + ($1,500 × 3) = $32,200. Note: The additional standard deduction is $1,500 for MFJ/Qualifying Widow(er)/MFS and $1,850 if single or head of household.

Steven and Julie's children have the following for this year: - Brian, age 12, earns $2,800 in salary mowing lawns. - Courtney, age 19, earns $2,600 in dividends and capital gains. - Derek, age 16, earns $2,700 in dividends and interest. - Danny, age 10, earns $900 in dividends and interest. Whose income is subject to the tax at their parent's tax rate? A- Brian. B- Courtney and Derek. C- Derek. D- Brian, Courtney and Derek.

Solution: The correct answer is C. In 2023, a person under the age of 19 or under the age of 24 and a full-time student at the end of the tax year, would pay tax at their parent's tax rate if they had more than $2,500 in unearned income. Thus, $200 of Derek's unearned income is taxed at his parent's tax rate. Brian's income is earned and the kiddie tax applies only to unearned income. Earned income is taxed at the child's rate (single tax filer rate). Courtney is 19, and kiddie tax would only apply if the fact pattern stated she was a full-time student. Since she is not, all income is taxed to her. Danny avoids taxation at the parent's rate as $900 is under the standard deduction.

John, Jay and Jeff each have an ownership interest in Three Guys Burgers, Inc. Based on the following information, which of them is/are considered to have materially participated in the conduct of the Three Guys Burgers business this year? I- John dedicated more than 500 hours this year to Three Guys Burgers. II- Jay devoted 150 hours to Three Guys Burgers this year. III- Jeff devoted 115 hours to Three Guys Burgers this year, but also devoted more than 100 hours to several other similar activities, for a total of 520 hours in all of the activities combined. A- I only. B- II and III only. C- I and III only. D- I, II and III.

Solution: The correct answer is C. Jay has not materially participated. Although Jay devoted more than 100 hours to the activity, he did not devote more hours than anyone else because John worked at Three Guys Burgers for more than 500 hours. Jeff is also a material participant because he devoted more than 100 hours to the activity and also devoted more than 100 hours to several other similar activities, for a total of more than 500 hours in all of the activities combined. Note: To be considered material we have to meet a specific set of rules among them is 1) the Taxpayer dedicated more than 500 hours to the activity or 2) the Taxpayer dedicated more than 100 hours and the most of anyone. Part of the rules for PAL allows a Taxpayer to make an annual election to join similar activities to achieve the > than 500 hours for materiality, which is what III is outlining.

Explain the reason for the inclusion amount with respect to leased passenger automobiles. A- The inclusion amount estimates the difference between business and personal use. B- The inclusion amount is the result of the formula that determines whether it is best to lease or to buy an automobile. C- The inclusion amount is designed to help to level out the lease expense vs. the depreciation expense. D- The inclusion amount is the commuting miles that are included as part of the standard deduction on a leased auto, which cannot be deducted on an owned auto.

Solution: The correct answer is C. The inclusion amount is designed to help to smooth the lease expense vs. the depreciation expense. The lease is front loaded.

Ima Clipper, a well-known artist, donated one of her original bronze creations to a local charity, which auctioned the piece for $3,000. Ima totaled her costs as follows: Bronze = $425 Other materials = $150 Pro-rata overhead = $125 Furnace/casting fees = $200 Artistic contribution = $2,100 Assuming this is Ima's only charitable contribution and based on an annual income of $150,000, what is the maximum amount of charitable income tax deduction available to her? A- $3,000 B- $2,100 C- $775 D- $900

Solution: The correct answer is C. Only materials and expenses are deductible, not artistic contribution or time. No deduction is allowed for use of property; therefore, the pro-rata overhead would likely not be allowed. Bronze $425 + Other materials $150 + Furnace/casting fees $200 = $775.

Quin owns a house in Connecticut and an apartment in New Orleans. Quin spends most of her time in Connecticut, so she sometimes rents out the apartment in New Orleans when she is not there. This year, Quin rented out the apartment for thirty days and personally used the apartment for forty days. How will Quin's rental activity be classified for tax purposes and why? A- Nontaxable activity because Quin used the apartment personally more than she rented it out. B- Mixed-use activity because Quin both rented out the apartment and used it personally. C- Mixed-use activity because Quin rented out the apartment for more than 14 days and personally used the apartment for the greater of 14 days or 10% of the rental days. D- Rental activity because Quin rented out the apartment for more than 14 days.

Solution: The correct answer is C. Option "A" is incorrect; this would be a nontaxable activity if Quin had rented the property for less than 15 days. However, Quin rented the property for 30 days, so it is not a nontaxable activity. Option "B" is incorrect because an activity will not be classified as a mixed-use activity just because the owner both rents the property and uses it personally. Option "C" is correct; even though Quin did rent out the apartment for more than 14 days, she also used it personally for the greater of 14 days or 10% of the rental days. Therefore, Quin's rental activity is classified as a mixed-use activity.

Which of the following is applicable on a case-by-case basis? A- A revenue ruling. B- The Internal Revenue Code. C- A private letter ruling D- A Supreme Court decision.

Solution: The correct answer is C. Option "C" is for the case the private letter ruling is written for only. All other Options impact all taxpayers.

Claude and Daphne are trying to calculate their gross income for the current year. Which of the following items should they include in gross income? I- Child support payments in the amount of $15,000 received by Daphne from her ex-husband for the support of Daphne's minor child Emile. II- $1,200 in dividends received by Claude and Daphne from Mudbugs, Inc., a corporation in which they own 200 shares of stock. III- Unemployment benefits in the amount of $800 received by Claude from the state of Louisiana. IV- $3,000 that Daphne earned selling her homemade andouille sausage. A- IV only. B- I and II only. C- II, III, and IV only. D- I, II, III and IV.

Solution: The correct answer is C. Option "I" is not correct because child support is not includible in gross income. All of the other options are included in gross income (dividend income, unemployment compensation benefits, and gross income from self-employment).

Quin owns a house in Connecticut and an apartment in New Orleans. Quin spends most of her time in Connecticut, so she sometimes rents out the apartment in New Orleans when she is not there. This year, Quin rented out the apartment for thirty days and personally used the apartment for forty days. How will Quin's rental activity be classified for tax purposes and why? A: Nontaxable activity because Quin used the apartment personally more than she rented it out. B: Mixed-use activity because Quin both rented out the apartment and used it personally. C: Mixed-use activity because Quin rented out the apartment for more than 14 days and personally used the apartment for the greater of 14 days or 10% of the rental days. D: Rental activity because Quin rented out the apartment for more than 14 days.

Solution: The correct answer is C. Option A is incorrect; this would only be a nontaxable activity if Quin had rented the property for less than 15 days. However, Quin rented the property for 30 days, so it is not a nontaxable activity. Option B is incorrect because an activity will not be classified as a mixed-use activity just because the owner both rents the property and uses it personally. Option D is incorrect; even though Quin did rent out the apartment for more than 14 days, she also used it personally for the greater of 14 days or 10% of the rental days. Therefore, Quin's rental activity is classified as a mixed-use activity.

Which of the following would be required to make federal estimated tax payments, assuming each had earnings in excess of $200,000? I- Estate. II- Partnership. III- C corporation. IV- Sole proprietor. A- I and II. B- III and IV. C- I, III, and IV. D- II and III.

Solution: The correct answer is C. Partnerships are flow-through entities that never pay income tax. Therefore, partnerships would never be required to pay estimated taxes. A partnership is reported on form 1065 (an informational tax form) that flows through to the owner's 1040. A sole prop is reported directly on the 1040 (schedule C), making the individual and the business one and the same. Instructor Note: the $200,000 here is just a random number to indicate a tax liability that would need to use estimates, it could have been $100k or $300k here as well. CFP Board will often provide excess information as a distractor when testing concepts.

Punitive damages received as a result of personal injury: A- Are not taxable because the damages are intended to punish the other party. B- Will be non-taxable if the payments were received on account of a physical personal injury. C-Are taxable. D- Are not taxable if the personal injury was to one's business reputation.

Solution: The correct answer is C. Punitive damages are always included in income. They may be excludible if they are the only recovery in a wrongful death case (The Alabama Rule).

Leona is 68 years old and single. What is the lowest adjusted gross income that will require Leona to file a tax return for 2023? A- $8,900 B- $13,755 C- $15,755 D- $16,750

Solution: The correct answer is C. Read the question and rationale carefully. Leona must file a tax return if her adjusted gross income is $15,700 or more for the current year ($13,850 basic standard deduction + $1,850 additional standard deduction for age). Any amount GREATER THAN $15,700 will cause taxes to be filed. **Keep in mind this question is looking for the LOWEST amount that would cause her to file.** Answer choice A is below the single standard deduction and single additional standard deduction for age. Answer choice B is below the single standard deduction and single additional standard deduction for age. Answer choice C is $55 above the single standard deduction and single additional standard deduction for age. Answer choice D is $1,050 above the single standard deduction and single additional standard deduction for age. But the question is looking for the LOWEST amount that would cause her to file.

Which of the following properties has a 7-year MACRS depreciation life? A- Computers B- Residential real property C- Office furniture D- Autos

Solution: The correct answer is C. Remember "CAT-CORN" Computers, Autos, and Trucks are 5 year. Office furniture is 7 year. Residential real property (rental houses) is 27.5 year and Nonresidential real property (commercial buildings) is 39 year.

Abner owned bonds that paid $750 of interest on the first day of January each year. Exactly one-third of the way through the current year, Abner gave the bonds to his brother, Brody. When Brody receives the $750 of interest on the first day of January next year, what amount will be included in Brody's gross income next year? A: $0 B: $250 C: $500 D: $750

Solution: The correct answer is C. Remember, interest is paid after it is earned, so the amount received by Brody includes interest earned while the bonds were owed by Abner. Brody owned the bonds for two-thirds of the year. Therefore, he must report two-thirds of the interest in his gross income for the year in which the interest is received.

Kal has taxable income this year of $6 million. He purchased $2,900,000 worth of depreciable property this year and is trying to calculate his §179 deduction. What is the correct amount? A- $10,000 B- $1,300,000 C- $1,150,000 D- $1,160,000

Solution: The correct answer is C. Since he placed into service more assets than allowed under the limitation you must calculate the phase-out. $2,900,000 placed into service less $2,890,000 placed into service limit = $10,000 $1,160,000 - $10,000 = $1,150,000. Since this is below the net income of the business there are no further limitations.

Jessie, an unmarried taxpayer using the single filing status, received $16,000 of Social Security retirement benefits this year. Jessie also received $5,000 of interest income and $45,000 of income from her retirement plan during the year. How much of Jessie's Social Security benefits must be included in her gross income? A: $0 B: $8,000 C: $13,600 D: $16,000

Solution: The correct answer is C. Since her MAGI ($50,000) plus one-half of her Social Security benefits (0.5 × $16,000 = $8,000) exceeds her adjusted base amount ($34,000), she must calculate her includible Social Security benefits using the formula 3 or 4 below. 3. 0.85 × $16,000 = $13,600 4. 0.85 × [$50,000 + (0.50 × $16,000) - $34,000] = $20,400 plus the lesser of the amount calculated using 1 and 2 below: 1. 0.50 × $16,000 = $8,000 2. 0.50 × [$50,000 + (0.50 × $16,000) - $25,000] = $16,500 The lesser amount is $8,000 The formula 4 total is $28,400 ($20,400 + $8,000) The lesser of the formula 3 or 4 amounts is $13,600. Therefore, $13,600 of the Social Security benefits must be included in Jessie's gross income.

Sources of "substantial authority" available for tax research include: I- Internal Revenue Code. II- Congressional Committee Reports (Blue Book). III- Treasury Regulations. IV- Private Letter Rulings. A- I and II only. B- I, II, and III only. C- I, II, III and IV. D- I, III and IV only.

Solution: The correct answer is C. Substantial authority is official words and rulings which can be relied on to support a tax opinion or position. All of these can be relied on by someone. Instructor note: The question is asking what represents substantial authority, not to whom. Private Letter Rulings's are substantial authority for who they are written to.

Which of the following is a tax credit that reduces the tax due on taxable income? I- Qualified dependent credit. II- Child tax credit. III- Earned income credit. IV- Credit for estimated tax payments. A- I, II and III only. B- II and III only. C- I, II, III and IV only. D- III, and IV.

Solution: The correct answer is C. The "Qualified dependent credit" is new under TCJA and applies to qualified dependents and/or qualifying children 17 and over. It is limited to $500. The "child tax credit" applies to qualifying children under age 17 and was expanded under TCJA to $2,000 per child, with the possibility of up to $1,600 per child being refundable. The "earned income credit" is a credit against the calculated tax, available to those with very low income, predominantly from earnings (wages) and it is a refundable credit. The CFP® exam considers the prepayment of tax, through withholding and/or estimated tax payments, as credits as well since they also reduce the balance due.

Edgar is looking for as many above the line deductions as he can find. His recent raise, while very welcome, put him into a higher tax bracket. He provides you with the following list of items he hopes will help reduce his gross income. Which one can you tell him will be an above the line deduction? A- Edgar paid $8,000 in moving costs for the position that came with the raise. B- Edgar paid $1,000 to fix some water damage from a water pipe he hit while moving items out of his basement. C- Edgar made a $1,000 contribution to an HSA to help with his deductible medical plan. D- Edgar made a $500 donation to his favorite animal shelter before he moved.

Solution: The correct answer is C. The HSA is an above the line deduction. A is incorrect. TCJA suspended moving expense deductions for 2018 - 2025, unless you are active military changing stations. B is incorrect. Casualty losses are a below the line deduction and are not currently deductible under TCJA (2018 - 2025) unless they are due to federal declared disasters. D is incorrect. Charitable deductions are below the line if you are itemizing your taxes.

The classifications of income are: I- Active Income. II- Earned Income. III- Unearned Income. IV- Portfolio Income. V- Passive Income. A- II, III, IV and V only. B- I, II and III only. C- I, IV and V only. D- All of the above.

Solution: The correct answer is C. The IRS has three classifications of income; active, passive and portfolio. Earned income is a subset of active income while unearned income may be either passive or portfolio income.

Under the Last In First Out (LIFO) inventory system: A- The last goods purchased are the first goods sold. B- LIFO means that the oldest goods will remain in inventory until sold. C- The cost of goods is assigned the most current inventory costs. D- LIFO ensures that the newest goods are sold.

Solution: The correct answer is C. The LIFO method is concerned with movement of costs through inventory, not goods. The cost of the last units purchased will be the first costs to be transferred to cost of goods sold when the goods are sold.

Alice and Bob have been married for 10 years and file a joint tax return. Bob is a spender and Alice is a saver. On Bob's last overspending spree, he accumulated a large amount of debt but managed to pay the balance down to $10,000. In an attempt to hide the ramifications of this spending spree from Alice, this year Bob participated in a debt forgiveness program under which 45% of the debt was forgiven and the remaining debt was to be repaid over a term of 5 years. What amount will be included as income on Alice and Bob's joint tax return this year as a result of this arrangement? A- $10,000 will be included in income on their 1040. B- $6,500 will be included in income on their 1040. C- $4,500 will be included in income on their 1040. D- $0 will be included in income on their 1040.

Solution: The correct answer is C. The debt relief agency will send IRS Form 1099-C to Bob, indicating the amount of debt that was forgiven. This amount is then reported on the Form 1040 as taxable income.

What is the maximum capital loss that an individual taxpayer can deduct in any one year? A- None. B- No more than $3,000. C- No more than $3,000 in addition to the total capital gain recognized in that year. D- No more than $3,000 for that year plus any other losses carried forward from prior years.

Solution: The correct answer is C. The maximum capital loss that a taxpayer can apply is up to the extent of any capital gains for that year plus an additional $3,000 against income from other sources. Any excess losses are carried forward and applied as discussed above until used.

Which of the following is not generally one of the four main categories of itemized deductions? A- State income or sales tax, and property taxes. B- Charitable contributions. C- Personal casualty losses. D- Qualified personal residence property.

Solution: The correct answer is C. The statement is incorrect since you can no longer deduct personal casualty losses. A personal casualty loss that stems from a Federal Declared Disaster, would be deductible. A Federal Declared Disaster would be stated if it applied to the question. A Qualified personal residence is a QPRT, and the grantor of the trust would still be able to deduct the property taxes. Medical Expenses, Taxes, Interest, Charitable Contributions. TCJA suspended miscellaneous itemized deductions (subject to the 2% floor) and personal casualty losses (other than Federal Disasters).

Blake is a CFP® professional and prepares tax returns for his clients. He prepared his brother's income tax return for $1,000 and he willfully neglects to include $30,000 of income since his brother did not receive a 1099 for consulting work. Blake is aware that his brother earned the $30,000 but fails to report it since he doesn't believe the IRS will catch the understatement of income. The additional tax on this $30,000 of income would have been $7,500. How much of a penalty may Blake be subject to for the understatement of income? A- None, but his brother will be subject to penalties. B- $3,750 C- $5,000 D- $7,500

Solution: The correct answer is C. The preparer penalty for willful or reckless conduct is the greater of $5,000 or 50% of the income derived by the preparer for the return. In this case, he charged his brother $1,000.

Which of the following is INCORRECT? A- Alimony paid during 2018 is a deduction for AGI. B- The expenses paid associated with royalty property are a deduction for AGI. C- Contributions to an IRA are a deduction from AGI. D- Medical expenses paid are a deduction from AGI.

Solution: The correct answer is C. The question is asking you to identify the false statement. Deduction are either FOR AGI (above the line) or FROM AGI (below the line). IRA contributions (when deductible) are deducted FOR AGI and an adjustment to gross income (not FROM AGI as listed, making statement C false). If alimony is paid in 2018, the divorced was finalized prior to 12/31/18 and will follow the old rules for deductibility and inclusion in income. Alimony is an above the line (FOR AGI) deduction. Expenses associated with royalties are an above the line deduction (FOR AGI). Medical expenses are a below the line deduction for the amount exceeding 7.5% of AGI (FROM AGI).

Which of the following statements regarding deductible expenses is not true? A- Above-the-line deductions are sometimes referred to as adjustments. B- Above-the-line deductions are usually considered to be more favorable to the taxpayer than below-the-line deductions. C- Below-the-line deductions are usually considered to be more favorable to the taxpayer than above-the-line deductions. D- Taxpayers may take the greater of their itemized deductions or the standard deduction.

Solution: The correct answer is C. The question was looking for the false statement. While statement C is false, you should note that above the line deductions (for AGI) are more favorable since many phaseouts are based on AGI. A lower AGI may result in additional deductions or tax credits.

Pedro's child attends a school operated by the church the family attends. Pedro made a donation of $500 to the church in lieu of the normal registration fee of $200. In addition, Pedro paid the regular tuition of $8,000 to the school. Based on this information, what is Pedro's charitable contribution? A: $0 B: $200 C: $300 D: $8,300

Solution: The correct answer is C. The taxpayer's donation of $500 in lieu of the normal $200 registration fee would be deductible to the extent of $300 [$500 - $200 benefit received (the registration fee)]. The tuition of $8,000 is a personal expense which cannot be deducted as a charitable contribution.

Which of the following is not a requirement for an alimony deduction, if applicable? A: The payments must be in cash. B: The payments must cease upon the death of the payee. C: The payee must have a dependent child. D: The payor and payee must not live in the same household at the time of the payments.

Solution: The correct answer is C. There is no such requirement for payments to be classified as alimony. Alimony is support for an ex-spouse. Alimony is deductible for any divorces finalized by 12/31/2018.

Karen and Tom are married filing jointly taxpayers with 3 children. Their MAGI is $195,000. What is the maximum amount of the child tax credit that could be refundable to Karen and Tom? A- $0. B- $2,000. C- $4,800. D- $6,000.

Solution: The correct answer is C. They have 3 children. The child tax credit is $2,000 per child against their tax obligation, up to $1,600 ($4,800 for the three children) per child can be refundable if there is no tax obligation due.

Victor and Vivian have a very diverse family. Which of the following children would not be a qualifying child for the purpose of claiming the child tax credit in the current year? A- Vivian's granddaughter, who turned 4 in the current year and lives with Victor and Vivian for more than half of the year. B- Victor's brother, who turned 16 in the current year and lives with Victor and Vivian for more than half of the year. C- Victor and Vivian's daughter, who turned 17 in the current year and does not provide more than half of her support. D- Victor and Vivian's son, who was born on October 21 of the current year.

Solution: The correct answer is C. To qualify for the child tax credit, the child must be under 17. Not to be confused with the age to be a qualifying dependent, which is under 18. Individuals who reach the age of 17 during the tax year are not eligible to be qualifying children for the purpose of the child tax credit. Victor and Vivian will not be able to claim a child tax credit for their daughter who turned 17. All of the other children are eligible to be qualifying children.

Payments for employment-related care that are made to relatives of the taxpayer may qualify for the credit for child and dependent care expenses. Which of the following payments does not qualify? A- Payments for employment-related care made to the taxpayer's aunt. B- Payments for employment-related care made to the taxpayer's 21-year-old daughter (who is not a dependent of the taxpayer). C- Payments for employment-related care made to a dependent of the taxpayer. D- Payments for employment-related care made to taxpayer's 17-year-old niece.

Solution: The correct answer is C. To qualify for the dependent care credit, both parents (or one if single) must be paying child care expense for them to work. They cannot use the child care expense, for the purposes of the credit, if they are a stay at home care giver that needs a break a couple hours a day or week. Paying a dependent of the taxpayer to care for children while the taxpayer is at work does not qualify towards the credit for child and dependent care expenses. You cannot pay your 15 year old child to watch your 8 year old child and have it be a qualified expense. You could pay a (non-dependent) 21 year old child to watch the children while you are at work and that will count as a qualified expense. All of the other options are qualifying payments.

Peyton has a piece of equipment used in his business. He exchanges it for a like-kind asset owned by Eli. (Peyton and Eli are unrelated). The basis of Peyton's asset is $40,000 and he gives Eli $20,000 cash plus the asset in exchange for Eli's asset, which is worth $36,000. Eli's basis in his original asset is $10,000. What is Peyton's recognized gain or loss? A- $0 B- $4,000 loss C- $24,000 loss D- $6,000 gain

Solution: The correct answer is C. While losses are not recognized in like kind exchanges; this is not a like kind exchange. Only property is eligible to like kind treatment under the new law. As a result, Peyton has a realized/recognized loss as follows. Value received (new property) $36,000 - Basis $40,000 - cash $20,000 = $24,000 loss. His new basis is the value he "paid" $36,000. TCJA 2017 adjusted the 1031 rules to only qualify for real property. Equipment is no longer a qualified 1031 exchange.

Wilma is married to Herb, who abandoned her five years ago. She has not seen or communicated with him since June of that year. She maintains a household in which she and her two young dependent children live. Which of the following statements about Wilma's filing status in this year is correct? A- Wilma can use the rates for single taxpayers for this year. B- Wilma can file a joint return with Herb for this year. C- Wilma can file as a head of household for this year. D- Any of the above.

Solution: The correct answer is C. You are able to file as a head of household if you are considered to be abandoned by your spouse and are claiming dependent children. An individual is required to live apart from his or her spouse for the entire last six months of the tax year to file under abandoned spouse status. Wilma meets the "abandoned spouse" rules. Therefore, she can file as a head of household. Otherwise, her filing status would be married, filing separately. Head of Household required that the taxpayer pay for more than 50% of the upkeep of the home in which the qualifying individuals reside. The qualifying individuals need not be dependents of the taxpayer.

Peyton has a warehouse used in his business. He exchanges it for a like-kind asset owned by Eli. (Peyton and Eli are unrelated). The basis of Peyton's asset is $40,000 and he gives Eli $20,000 cash plus the asset in exchange for Eli's asset, which is worth $36,000. Eli's basis in his original asset is $10,000. What is Peyton's new basis? A-$20,000 B- $40,000 C- $60,000 D- $80,000

Solution: The correct answer is C. You are expected to calculate his FMV. You can do this because you know that the values exchanged must be equal. Peyton gives Eli $20k + property in exchange for a property worth $36,000: $20,000 + x = $36,000 x = $16,000

Karen is single, blind, and 72 years old. What is her total standard deduction, assuming she has filed her taxes for the 2023 tax year? A- $13,850 B- $15,350 C- $15,700 D- $17,550

Solution: The correct answer is D. Item Amount Standard deduction for a single tax filer $13,850Additional standard deduction for a single tax filer age 65+$1,850Additional standard deduction for a single tax that is blind (upon filing)$1,850 Total Standard Deduction for Karen $17,550 Note: a married tax filer would receive a $1,500 additional standard deduction for age 65+ and/or blindness.

Under which of the following circumstances will a taxpayer be subject to an accuracy-related penalty? I: If the taxpayer files an incorrect return and has failed to make a good faith effort to comply with the tax law. II: If the taxpayer understates his tax liability by more than 5 percent of the correct tax liability. III: If the taxpayer makes a substantial understatement associated with an estate or gift tax valuation. A; 1 only B: 1 and 2 C; 2 and 3 D: 1 and 3

Solution: The correct answer is D. A taxpayer will be subject to an accuracy-related penalty if he makes a substantial understatement of his tax liability, generally more than 10 percent of the correct tax liability and at least a $5,000 deficiency.

Under what circumstances will the child of divorced parents be treated as the qualifying child of the noncustodial parent? I- The parents are legally divorced and not living in the same household. II- The child receives over one-half of his support for the year from his parents. III- The child is in the custody of the parent for more than half the year. IV- The custodial parent signs a statement that he will not claim the child as a dependent for the year and the noncustodial parent attaches the statement to his return. A- I only. B- I, II and III only. C- II, III, and IV only. D- I, II, III, and IV only.

Solution: The correct answer is D. All of the options are requirements that must be met in order for a child of divorced parents to be treated as a qualifying child of the noncustodial parent.

Which of the following imposed the first constitutional federal income tax? A- Revenue Act of 1861. B- 16th Amendment. C- Revenue Act of 1916. D- None of the above.

Solution: The correct answer is D. Answer "A" is incorrect because although the Revenue Act of 1861 did impose a federal income tax, it was later found to be unconstitutional because Congress did not have the power to levy an individual income tax at that time. Answer "B" is incorrect because the 16th Amendment gave Congress the power to impose an individual income tax, but did not itself impose that tax. Answer "C" is incorrect because the Revenue Act of 1916 raised the rates previously imposed under the Revenue Act of 1913. Therefore, answer "D" is correct because the Revenue Act of 1913 imposed the first constitutional income tax.

What characteristics of an automobile lease might cause the lease to be treated as a purchase for tax purposes? I- Intent of the parties to the transaction. II- Whether any equity results from the arrangement. III- Whether any interest is paid. IV- Whether the fair market value of the car is less than the "lease payment" or option when the option to buy is exercised. A- I and IV only. B- I, III and IV only. C- I and III only. D- Any of the above.

Solution: The correct answer is D. Any sign of ownership, including that of an installment sale will cause a leased vehicle to be treated for tax purposes as a purchased vehicle. Rather than deducting the lease payments, the taxpayer will be required to elect either standard mileage or to take depreciation plus actual expenses.

A minority non-employee shareholder in an S corporation: I- Receives income when the corporation declares and pays a dividend. II- Votes for the Board of Directors at the annual shareholders' meeting. III- Receives a K-1 annually in order to prepare a personal income tax return. IV- Reports on a personal income tax return a pro rata share of the corporate profit or loss. A- I, II and III only. B- I and III only. C- II and IV only. D- I, II, III and IV.

Solution: The correct answer is D. As a shareholder (employee or non-employee), you are entitled to dividends and voting your pro rata share of ownership. The K-1 will provide all shareholders with their share of corporate income or loss.

Walter is a cash basis taxpayer. Which of the following items must be included in his adjusted gross income calculation for the current year. A- Earnings from Series EE bonds. B- Business income that was earned on Dec. 15 of the current year. The client mailed a check on Dec. 29 of the current year. The check arrived in Daniel's mailbox on Jan. 2 of the following year. C- A business sale to a customer made on Oct. 15. Daniel extended the payment due date, and the client has yet to pay the bill. D- Original Issue Discount on the bonds for the current year.

Solution: The correct answer is D. Earnings on EE bonds can be delayed until redeemed or matured. Since he is a cash basis taxpayer, Walter does not have to include the business income until the year in which the check is received. The account receivable will not be included until paid. The original issue discount must be included in the current year's income regardless of receipt.

Fred's AGI is $150,000. He contributed $90,000 in cash to a public charity. What is Fred's charitable contribution deduction for AMT purposes? A: $27,000 B: $45,000 C: $75,000 D: $90,000

Solution: The correct answer is D. Fred's charitable contribution deduction for AMT purposes is the same as for regular taxes, limited to 60% of AGI ($150,000 AGI × .60 = $90,000). Charitable deductions are not an adjustment for AMT. For Tax years 2020 and 2021 only; Fred can deduct 100% of AGI cash contributions to qualified charities. 2022 tax year returns to 60% of AGI cash contributions.

Frank's automobile, which he uses exclusively in his trade or business, was damaged in an accident. The adjusted basis of the automobile prior to the accident was $8,000. The fair market value of the automobile before the accident was $10,000 and the fair market value of the automobile after the accident is $500. Insurance proceeds of $9,500 were received. What are Frank's income tax consequences of this transaction? A- $0 B- $1,500 loss C- $500 loss D- $1,500 gain

Solution: The correct answer is D. His gain is $9,500 - $8,000 because he has already taken depreciation down to $8,000, and he received compensation in excess of his depreciable basis.

Can money paid for child support be structured in a divorce as to be deductible to the payor spouse for divorces prior to 2019? A- Yes, if the decree stipulates such. B- No, unless the taxpayer gets a letter ruling from the IRS. C- No, child support payments can never be made deductible. D- Yes, if the money to be considered as child support is included in deductible alimony

Solution: The correct answer is D. If an agreement is reached (prior to 12/31/18) between former spouses where the decreed amount of alimony is increased to include child support, then the additional alimony would be taxable to the recipient and deductible to the payor. The additional money cannot be based on any contingency such as with the child reaching the age of majority or death.

Which of the following statements regarding the deduction of costs associated with investigating the purchase of a new line of business is not correct? A- If the new line of business is not purchased, no deduction is permissible. B- If the new line of business is purchased and it is in the same line of business as the current trade or business operation, the cost of investigating the new business is fully deductible. C- The ability to deduct the cost of investigating a new line of business is often overlooked by taxpayers. D- If the new line of business is purchased and it is in a different line of business as the current trade or business operation, there is no way to recoup the costs of investigation.

Solution: The correct answer is D. If the new line of business is purchased and it is in a different line of business as the current trade or business operation, the costs of investigation are recouped by capitalizing the expenses and amortizing it ratably over a 60-month period.

Which of the following statements concerning hobby activities is correct? A- Any activity which does not generate a profit within three years must be treated for income tax purposes as a hobby activity. B- The IRS must prove that the taxpayer does not have a profit motive to treat an activity as a hobby activity. C- Expenses associated with the hobby activity can offset, without limitation, the income generated from the activity. D- Hobby income is included in gross income above the line, while hobby expenses are not deductible.

Solution: The correct answer is D. Income generated from a hobby activity is included in gross income, and expenses associated with the hobby are no longer deductible per TCJA for years after 12/31/17. If the activity earns a profit in three out of five years, the IRS has the burden of proof of showing that there is no profit motive, but if there has not been a profit in three out of the last five years, the taxpayer has the burden of proof. There is no bright line test that requires an activity to be treated as a hobby activity based on the income trend of an activity.

Which of the following is an itemized deduction? A- Job related moving expenses. B- Tax return preparation fee C- Unreimbursed employee expenses D- Nonprescription insulin.

Solution: The correct answer is D. Medical expenses including nonprescription insulin are itemized and deductible in excess of 7.5% of AGI . The other choices are no longer deductible under TCJA.

Which of the following are requirements for satisfying the bona fide resident test necessary for excluding foreign earned income? I- The taxpayer must establish permanent quarters in the foreign country for himself and his family. II- The taxpayer may not return to the United States during the year. III- The taxpayer must intend to work in the foreign country for an indefinite period of time. A- I only. B- I and II only. C- II and III only. D- I and III only.

Solution: The correct answer is D. Occasional trips back to the United States for vacation or other purposes will not usually prevent a taxpayer from meeting the bona fide resident test.

How much of the self-employment tax can be deducted from gross income? A- None. B- 25% C- 40% D- 50%

Solution: The correct answer is D. One half (50%) of self-employment tax is deductible from the taxpayer's gross income.

David is a wealthy attorney in the highest marginal income tax bracket. He is interested in purchasing a franchise in a fast-growing food chain with some of his associates. After reviewing the proposal, you have determined that apart from a substantial up-front investment, the business will NOT need to retain income and any income generated in subsequent years will be paid out to the investors. David wants to be assured the business would NOT be disrupted if one of his associates lost interest or encountered personal financial reversals. What form of business structure would make the most sense? A- A Limited Partnership. B- A General Partnership. C- A C Corporation. D- A S Corporation.

Solution: The correct answer is D. Options "A" and "B" are incorrect in that they would dissolve in the event of a 50% turnover in ownership in a 12-month period. Option "C" is incorrect because a C corporation generally retains income. Option "D" is correct as an S corporation holds no retained earnings since all profits or losses are passed through to its owners.

Which of the following decreases a taxpayer's at-risk amount? A- Cash and the adjusted basis of property contributed to the activity. B- Amounts borrowed for use in the activity for which the taxpayer is personally liable or has pledged as security property not used in the activity. C- The taxpayer's share of amounts borrowed for use in the activity that is qualified non-recourse financing. D- Passive losses which are used against passive income from another source.

Solution: The correct answer is D. Options "A," "B" and "C" all increase the at risk amounts. Remember, when taking passive losses, the first step is looking at the at risk amount. If there are $75,000 in loss and only $50,000 at risk, $25,000 is suspended and all the at risk amount was used up. Unless there is an increase to the at risk amount, it is reduced to zero.

Which of the following factors should be considered in determining whether an activity is treated as an appropriate economic unit for the grouping of passive activities? A- The similarities and differences in types of business. B- The extent of common control. C- The extent of common ownership. D- All of the above.

Solution: The correct answer is D. Options "A," "B" and "C" are all considerations in determining the appropriate treatment of an economic unit.

A shareholder in an S corporation: I- Must be a US citizen. II- Votes for the Board of Directors at the annual shareholders' meeting. III- Receives a K-1 annually in order to prepare a personal income tax return. IV- Reports on a personal income tax return a pro-rata share of corporate profit or loss. A- I, II and III only. B- I and III only. C- II and IV only. D- II, III, and IV only.

Solution: The correct answer is D. Ownership of S corporation stock is restricted to individuals who are either US citizens OR US residents, estates, certain trusts, and charitable organizations. A shareholder in an S corporation may vote to retain or revoke S corporate status, votes, receives an annual K-1 and reports their pro rata share of profit or loss on their personal income tax return.

Brett owns a rental duplex located in Tucson, Arizona. He exchanges it for single family rental located in a tourist area. In which of the following locations may the other investment land be located and enable Brett to qualify for § 1031 like-kind exchange treatment? A: Mexico City, Mexico B: Toronto, Canada C: Paris, France D: None of the above

Solution: The correct answer is D. Real property located in the United States exchanged for foreign real property (and vice versa) does not qualify as like-kind property.

Tab owns an apartment building and a DVD rental business. He participates for approximately 600 hours in the apartment building operations and approximately 1,000 hours in the DVD rental activity. Which of the following statements is correct? A- Both the apartment building and the DVD rental businesses are passive activities. B- Neither the apartment building nor the DVD rental business is a passive activity. C- The DVD rental business is a passive activity, but the apartment building is not. D- The apartment building is a passive activity, but the DVD rental business is not.

Solution: The correct answer is D. Rental real estate is by definition a passive activity, therefore, the apartment rental business is passive. The DVD rental business is not a passive activity in that DVD rentals are short-term. To be considered as passive activities, other rental activity of goods and equipment must be long-term. According to code, short-term is defined as less than seven days.

Jermaine sold his entire block of 100 shares of DEF Company stock on February 10 of the current year, resulting in a long-term capital gain in the amount of $15,000. On November 11 of the current year, he sold his entire block of 100 shares of ABC Company stock, resulting in a capital loss of $15,000. On December 1 of the current year, he purchased 100 shares of ABC Company stock at $90 per share. What is the amount of the net capital gain or loss recognized by Jermaine in the current tax year? A- $0 net gain or loss. B- $6,000 net capital loss. C- $15,000 net capital loss. D- $15,000 net capital gain.

Solution: The correct answer is D. Since Jermaine sold the ABC Company stock at a loss, and then subsequently purchased shares of ABC Company stock within 30 days, the wash sale rule will apply. Therefore, the $15,000 loss on the sale will not be deductible in the current year. Jermaine will have a capital gain of $15,000, resulting from the sale of the DEF Company stock.

Greg just received his student loan statement that indicates that he has paid $3,000 of interest on his student loan during this tax year. How much of the interest may he deduct? A- None of the interest is deductible since it is consumer debt. B- $3,000 as an itemized deduction. C- $2,500 as an itemized deduction. D- $2,500 as an adjustment to income.

Solution: The correct answer is D. Student loan interest is an "above-the-line" deduction. The amount that can be taken is limited to $2,500 of interest paid.

Which of the following are deductions for AGI? A- Self-employed health insurance B- $6,000 qualified residential interest on vacation home C- Alimony paid on a divorce agreement date 2/1/16 D- A and C only.

Solution: The correct answer is D. The alimony and self-employed health insurance are deductions for AGI. Personal mortgage interest is not deductible for but rather as an itemized deduction.

Iris, a widow, elected to receive the proceeds of a $100,000 face value life insurance policy on the life of her deceased husband in annual installments of $12,500 over the remainder of her life, estimated to be 10 years. Which of the following is true? A: None of the payments received are included in gross income because their source is the life insurance policy. B: All of the payments are included in Iris's gross income because she paid nothing for the right to receive the payments. C: Iris will not recognize income until the 9th year, after she has recovered her investment. D: Iris must include $2,500 in gross income each year for the first 10 years she collects on the policy.

Solution: The correct answer is D. The income portion of the first annuity payment received is $2,500 ($12,500 - $10,000 exclusion). The exclusion is calculated as follows: ($100,000/125,000) × $12,500 = $10,000

Which of the following is a form on which individual taxpayers can report their income, deductions, exemptions, and other information required to calculate their federal tax liability? A- Form 1040IND B- Form 1040A. C- Form 1040EZ. D- Form 1040.

Solution: The correct answer is D. There is no Form 1040IND. 1040A and 1040EZ were eliminated with the tax reform under TCJA 2017.

Which of the following is a condition for receiving a dependent care credit? A- The taxpayer must provide over 1/2 cost of maintaining the household, which is also the principal residence of the child. B- The child must be a dependent. C- If married, both parents must work or go to school. D- All of the above.

Solution: The correct answer is D. These are all conditions for claiming the dependent care credit.

John owns a rental home in Arizona. He decided that he would like to acquire a rental home in Washington. Ted who lives in Washington has a rental home. For health purposes, Ted must relocate to Arizona. John and Ted decide to exchange properties under section 1031 of the code. The other facts pertaining to the exchange are: Ted's Basis = $100,000 John's Basis = $75,000 Ted and John exchange the two properties, but Ted has to give John an additional $25,000 in cash. The fair market value of Ted's property is $100,000, and the fair market value of John's property is $125,000. What is Ted's recognized gain or loss? A- $0 B- $25,000 C- $50,000 D- $75,000

Solution: The correct answer is A. John Ted FMV 125100 Basis75100Deferred Gain500 Boot250Recognized250Deferred250 New Asset100125New Basis75125Deferred Gain250

The Tax Reform Act of 1986 was roughly revenue neutral because: I- It was supported by both Republicans and Democrats. II- It was not intended to raise or lower taxes. III- It divided the tax burden evenly between individuals and businesses. IV- It made the tax rates equal across all tax brackets. A- II only. B- I and III only. C- II and III only. D- I, II, and IV only.

Solution: The correct answer is A. A piece of tax legislation is considered revenue neutral when it is expected to neither raise nor lower the total amount of taxes to be collected.

Under current rules, the Section 179 deduction for expensing qualifying business property in 2023 is: A- $510,000 B- $1,160,000 C- $2,030,000 D- $2,890,000

Solution: The correct answer is B. Section 179 deduction is $1,160,000 for 2023. However, the amount is reduced dollar for dollar for acquisition amounts above $2,890,000 placed into service during 2023.

What is the effective income tax rate? A- The highest marginal rate paid by a taxpayer based on income. B- The highest rate that is paid by an individual. C- The average rate a taxpayer pays based on taxable income. D- The tax rate considered for tax planning purposes.

Solution: The correct answer is C. The effective tax rate is the average tax an individual pays. The rate is determined by dividing the tax liability by the taxable income. The tax rate for planning purposes may be the effective or the highest marginal rate depending on the issue.


Conjuntos de estudio relacionados

CC Math: Cycle 2 Multiplication 1-15

View Set

InQuizitive Chapter 21: The New Deal, 1932-1940

View Set

Chapter 19 - Legal descriptions and deeds

View Set